Download as pdf or txt
Download as pdf or txt
You are on page 1of 83

Verbal Ability – 1 P-1(BS)

Synonyms, Antonyms, Fill in the Blanks, Analogy and Odd Word Out
Answers and Explanations

1 c 2 d 3 c 4 d 5 a 6 c 7 a 8 b 9 c 10 a
11 d 12 b 13 c 14 a 15 c 16 d 17 a 18 a 19 b 20 c
21 a 22 c 23 d 24 d 25 c 26 c 27 c 28 d 29 d 30 c
31 a 32 c 33 b 34 b 35 b 36 b 37 d 38 b 39 d 40 b
41 c 42 a 43 a 44 b 45 c 46 c 47 d 48 b 49 b 50 c
51 a 52 c 53 d 54 a 55 b 56 c 57 b 58 d 59 d 60 a
Practice Questions
1 c 2 d 3 c 4 b 5 c 6 c 7 b 8 c 9 b 10 c
11 b 12 c 13 b 14 c 15 b 16 c 17 c 18 d 19 c 20 d
21 d 22 c 23 c 24 d 25 b 26 d 27 d 28 b 29 b 30 d
31 c 32 d 33 b 34 a 35 d 36 b 37 d 38 c 39 c 40 c
41 c

1. c 'Prim' is an adjective which means to be stiffly formal 8. b ‘Paradigm’ is something that serves as a model or
and proper. Hence, the appropriate synonym would example. Therefore, option (b) is its perfect synonym.
be 'groomed'. Option (d), ‘paranormal’, means something which is
beyond the range of normal experience or scientific
2. d To be 'keen' is to be eager or enthusiastic about explanation.
something. Therefore, the correct answer is option (d).
Option (b), ‘rogue’, refers to someone who is dishonest 9. c ‘To assuage’ is to have a soothing effect; to pacify or
or immoral. calm. Hence, option (c) is the correct answer. Option
(b), ‘prohibit’, means to prevent someone from doing
3. c ‘Haughty' is an adjective and is used to describe something.
someone who is scornfully and condescendingly
proud. Therefore, 'arrogant' is its suitable synonym. 10. a ‘Innocuous’ is something which has little or no adverse
Option (d), ‘subservient’, means too willing to obey effect. Therefore, option (a), ‘harmless’, would be its
people. correct synonym.

4. d ‘Contemptuous’ means feeling or showing hatred or 11. d 'Compliant' means willing to do whatever you are asked
disapproval. Its synonym is ‘disdain’, which means a or ordered to do; ready and willing to comply. 'Yielding'
feeling of disapproval for something you do not feel means tending to do or willing to do what other people
deserves respect. Hence, option (d) is the correct want you to do. Hence, option (d) is the correct
answer. answer.

5. a 'Sympathy' is the feeling that you care about and are 12. b To ‘abominate’ is to abhor or dislike intensely. Hence,
sorry about someone else’s trouble, grief misfortune the appropriate word to replace it would be ‘detest’.
etc. Hence, the correct answer is option (a),
‘compassion’. 13. c To ‘fabricate’ is to fake or concoct. Hence, ‘forged’
would be suitable to replace it.
6. c ‘Tenebrous’ means shut off from the light; dark.
Therefore, option (c), ‘gloomy’, is its correct synonym. 14. a ‘Abject’ means terrible and without hope. Hence, option
(a) correctly replaces it.
7. a ‘Morbid’ means something unhealthy or characterized
by disease. Hence, option (a), ‘unwholesome’, is its 15. c ‘Pandemonium’ is a state of noisy disorder. Hence, its
appropriate synonym. suitable synonym would be ‘commotion’.

VA / Exercise - 1 CEX-5321/P1BS/17 / Page 1


16. d An ‘erudite’ person is very knowledgeable. Hence, its 31. a ‘Adept’ means skilled; ‘adapt’ means to adjust, ‘adopt’
antonym would be ‘ignorant’. 'Uncouth' means rude or means to take as your own, and ‘aloft’ means in the
socially unacceptable. Hence, option (d) is the correct air. Hence, option (a) is correct.
answer.
32. c ‘Ensure’ means to make sure, ‘assure’ means to
17. a ‘Craven’ means cowardly. So its antonym is ‘brave’. guarantee, ‘insure’ means to protect against loss or
Hence, option (a) is the correct answer. damage, and ‘abjure’ is to reject formally. Hence, option
(c) is correct.
18. a To be ‘laconic’ is to be very precise and curt, ‘verbose’
is to use too many words. Hence, option (a) is the 33. b ‘Boar’ is a wild pig, ‘bore’ is a tiresome person, ‘boor’ is
correct answer. a rude or insensitive person, and ‘burr’ is something
that is prickly and irregularly spherical. Only option (b)
19. b ‘To exculpate’ is to free from blame or guilt. So, the logically fits in the given sentence.
opposite would be ‘incriminate’, which means to charge
with or show evidence of involvement in a crime. 34. b A gang of thieves cannot be respectful. So, options
Hence, option (b) is the correct answer. (a) and (c) are negated. ‘Distinguished’ is used for a
person who seem important and worth respect. So,
20. c ‘Boost’ is to encourage while its opposite is option (d) is also incorrect. ‘Notorious’, which means
‘discourage’. ‘Bolster’ is a long pillow or cushion. well know for something bad, correctly fills in the
Hence, option (c) is the correct answer. blank.

21. a A ‘doltish’ person is a stupid or slow-witted person. 35. b ‘Meet’ means to encounter, ‘mete’ means to measure,
So, the correct antonym should be option (a), ‘clever’. ‘meat’ refers to flesh as food, and ‘meek’ means not
violent or strong. Hence, option (b) is the correct
22. c To ‘castigate’ is to rebuke or criticize in a severe manner. answer.
Its opposite is ‘praise’. Therefore, option (c) is the
correct answer. 36. b It is clearly hinted in the sentence that the concerned
person is feeling bored because of isolation and so it
23. d ‘Dull’ means lacking in force, intensity or sharpness. can lead to the development of negative emotions
So, its appropriate antonym is ‘sharp’. Hence, option within her. Option (b) is correct because ‘ennui’ means
(d) is the correct answer. 'Azure' refers to the blue ‘boredom’ and ‘discontentment’ means ‘dissatisfied’ or
color of the sky. ‘not content’.

24. d To be ‘pertinent’ is to have a clear decisive relevance 37. d Refer to the later part of the sentence which states
to the matter at hand. Hence, the opposite should be that the ‘bite has proved dangerous’. Hence the word
option (d), ‘irrelevant’. which will fit in the first blank is ‘venomous’. Similarly
‘The Copperhead’ is a singular subject and the correct
25. c ‘Execrable’ is something that is hateful. Hence, its pronoun which can replace it is ‘its’. Hence, the correct
opposite has to be option (c), ‘admirable’, which means answer is option (d).
something great, marvelous. Hence, option (c) is the
correct answer. 38. b ‘Plain Jane’ refers to someone who is plain. So a ‘flashy’
ring will be no use to him. Similarly the usage of phrases
26. c To ‘ratify’ a treaty is to make it legal, whereas to ‘annul’ such as ‘Kangra painting’ and ‘Mithila painting’ indicate
a treaty is to make it void or to cancel the validity. that the word in the second blank would be ‘antique’.
Hence, option (c) is the correct answer.
39. d Since the sentence talks about speed and
27. c ‘Adhere’ means to stick to, while detach is just the understanding, option (d) is the best answer. Moreover,
opposite. Hence, option (c) is the correct answer. in the later part of the sentence a direct hint is given
(comprehending whatever you are reading).
28. d To ‘refute’ an idea is to contradict it and fight, to
‘endorse’ an idea is to support it. Hence, option (d) is 40. b We are looking for a positive word for the first blank.
the correct answer. So ‘toughness’ and ‘well being’ both fit. However, for
the second blank only ‘vulnerability’ would fit. ‘Prone’
29. d The word ‘delicate’ means fragile or breakable. Hence, is incorrect as the blank requires a noun while it is an
its correct opposite is option (d), ‘rugged’, which means adjective. Hence, option (b) is the correct answer.
strongly built or constituted.
41. c ‘Fuel’ is processed by the ‘engine’ to provide energy
30. c ‘Slender’ is to be slim, ‘stout’ is to be heavyset. Hence, for the functioning of automobiles. Similarly ‘food’ is
option (c) is the correct answer. processed by the ‘stomach’ to provide energy for the
functioning of the body.

CEX-5321/P1BS/17 / Page 2 VA / Exercise - 1


42. a Just like a ‘cartographer’ makes ‘maps’, an ‘architect’ 59. d Options (a), (b) and (c) refer to something that is short-
makes ‘designs’. lived. Option (d) is their opposite.

43. a Just like ‘insecticides’ protect crops from ‘insects’, 60. a Option (a) is not related to Geography like the rest of
‘antiseptic’ protects wounds from ‘germs’. the choices. ‘Chorography’ is the science or art of
describing and mapping a region or district and
44. b ‘Abrogate’ and ‘annul’ are synonyms. They mean to ‘topography’ is the detailed mapping or charting of the
end in an official way. Similarly, ‘abdicate’ and features of a relatively small area, district, or locality.
‘renounce’ are synonyms. They mean to formally give
up something. ‘Acerbate’ means irritate. Practice Questions
45. c Just like ‘tennis’ is played in the ‘court’, a ‘drama’ is 1. c The correct answer is head waiter.
performed on a ‘stage’.
2. d Option (a) and (c) are irrelevant. Option (b) is the
46. c Just as a ‘bouquet’ is made of ‘flowers’, a ‘book’ is meaning of FIDUCIAL. Hence, the correct answer is
made of ‘pages’. option (d).

47. d Just as ‘study of birds’, is known as ‘ornithology’, 3. c Options (a), (b) and (d) are irrelevant. Option (c) is the
‘study of plants’ is known as ‘botany’. correct answer. The literal meaning of the phrase is
end of a cycle.
48. b Just as ‘plutocracy’ is the government by the wealthy,
‘democracy’ is the government by the people. 4. b Options (a), (c) and (d) are irrelevant. Cornucopia
‘Oligarchy’ refers to the government or control by a means ‘horn of plenty’. Hence, option (b) is the correct
small group of people. answer.

49. b Just as a ‘clue’ (plays the role of a helping device) 5. c Options (a) and (b) are irrelevant. Option (d) is the
helps in solving a mystery, a ‘key’(plays the role of a commercial name of a bicycle. Hence, the correct
helping device) helps in opening a lock. answer is option (c).

50. c Just like ‘hiss’ is the sound of a ‘snake’, ‘trumpet’ is the 6. c The correct answer is option (c). Other options are all
sound of an ‘elephant’. wrong.

51. a Options (b), (c) and (d) are used for identification and 7. b The correct answer is option (b).
indication of a particular thing. However, 'call' implies
'sound'. 8. c The correct answer is option (c). Often used to
describe a young lady.
52. c Options (a), (b) and (d) refer to showing good or
outstanding judgment or understanding. However, 9. b The correct term for option (a) is ‘hosier’ options (c)
option (c) is the antonym of the rest of the words. and (d) are irrelevant. The correct answer is option
‘Superficial’ means ‘affecting only the surface’. (b).

53. d ‘Thrilling’ is extremely exciting. ‘Humiliating’, 10. c The correct answer is option (c).
‘shameful’ and ‘upsetting’ are its antonyms, which mean
‘embarrassing’ or ‘annoying’. 11. b The correct answer is option (b).

54. a Options (b), (c) and (d) refer to a clumsy, stupid person. 12. c The word prelate means a priest, generally of a high
However, option (a) is the antonym of the rest. rank. Hence, option (c) is the correct answer.

55. b Options (a), (c) and (d) refer to enthusiastic approval 13. b The word portend means to indicate in advance future
or acclaim. However, option (b) is the antonym of the events. Hence, option (b) is the correct answer.
rest.
56. c Options (a), (b) and (d) refer to bulky appearance. 14. c Options (b) and (d) are irrelevant. One kills if one stabs
However, option (c) is the antonym of the rest. with a knife. Hence option (a) is incorrect. The correct
answer is option (c).
57. b Options (a), (c) and (d) refer to a feeling of anxiety.
Option (b) is the antonym of the rest. 15. b Omen means a sign of events to come. Hence, option
(b) is the correct answer.
58. d Options (a), (b) and (c) refer to rationality in behaviour
or decision. However, option (d) is the antonym of the 16. c A maelstrom is huge whirlpool occurring in an ocean.
rest. Hence, option (c) is the correct answer.

VA / Exercise - 1 CEX-5321/P1BS/17 / Page 3


17. c Dudgeon means resentment. Hence, the opposite is 29. b Options (a), (c) and (d) are nouns. Only option (b) is
delight. verb.
30. d Options (a), (b) and (c) pertain to food grains. Option
18. d Options (a) and (b) are synonyms. Option (c) is
(d) is the correct answer.
irrelevant. Hence, the correct answer is option (d).
31. c Options (a), (b) and (d) pertain to energy. Option (c)
19. c Options (b) and (d) are almost synonyms of spartan. which pertains to life is the correct answer.
Option (a) is the literal meaning. Hence, the correct
answer is option (c). 32. d Options (a), (b) and (c) are verbs. Option (d) can be
used only as an adjective and hence is the correct
20. d Options (a), (b) and (c) are synonyms. The correct answer.
answer is option (d).
33. b Options (a), (c) and (d) are collective nouns. Option
(b) is the correct answer.
21. d Options (a) and (b) are synonymous. Option (c) is an
old meaning. The correct answer is option (d). 34. a Options (b), (c) and (d) are adjectives. Option (a) is a
noun and hence the correct choice.
22. c Options (a), (b) and (d) are synonyms. Option (c) is
the correct answer. 35. d The emu is a flightless bird and hence option (d) is the
correct choice.
23. c Laconic means using very few words. Options (a)
and (b) mean the same. Option (d) has nothing to do 36. b Options (a), (c) and (d) are verbs. Option (b) is a noun
with words. Hence, option (c) is the correct answer and therefore the correct answer.
which means verbose.
37. d Options (a), (b) and (c) represent clothing. Option (d)
24. d Inveterate means ‘having a firmly established habit’. represents a weapon and hence is the correct
Hence, options (a), (b) and (c) are irrelevant. Thus answer.
option (d) is the correct answer.
38. c Masons, Bricklayers and Joiners work on buildings.
25. b Options (a), (c) and (d) are similar. Option (b) being Carpenters make furniture. Hence option (c) is the
antonymous is the correct answer. correct answer.

26. d Philander means to dangle after women. Hence option 39. c Option (a), (b) and (d) are local winds. Option (c)
(d) which means one who hates women, is the stands for a cyclone and is the correct answer.
correct answer.
40. c Options (a), (b) and (d) can be used as verbs. Option
27. d Options (a), (b) and (c) all pertain to men’s clothes. (c) can be used only as a noun. Hence, the right
Option (d), which means one who provides armour is answer.
the correct answer.
41. c Options (a), (b) and (d) refer to opinion. Option (c)
28. b Options (a), (c) and (d) are similar. Hence, option (b) is refers to a climb. Hence option (c) is the correct
the right answer. answer.

CEX-5321/P1BS/17 / Page 4 VA / Exercise - 1


Verbal Ability – 2 P-1(BS)

Cloze Test, One Word Substitution and Spellings


Answers and Explanations

1 d 2 c 3 b 4 c 5 b 6 c 7 b 8 c 9 a 10 b
11 c 12 d 13 a 14 a 15 b 16 b 17 c 18 b 19 d 20 d
21 b 22 c 23 a 24 b 25 b 26 a 27 d 28 d 29 b 30 d
31 a 32 c 33 d 34 a 35 d 36 d 37 a 38 b 39 c 40 c
41 b 42 b 43 c 44 c 45 d 46 a 47 c 48 b 49 c 50 d
51 a 52 c 53 a 54 a 55 d
Practice Questions
1 d 2 c 3 c 4 b 5 b 6 c 7 b 8 d 9 c 10 a
11 d 12 c 13 b 14 d 15 b 16 b 17 b 18 a 19 c 20 c
21 d 22 b 23 c 24 d 25 a 26 c 27 d 28 b 29 c 30 a
31 b 32 d 33 a 34 c 35 d 36 b 37 d 38 a 39 c 40 c
41 d 42 b 43 c 44 b 45 c 46 d 47 a 48 c 49 d 50 b
51 d

1. d The purpose of brand ambassadors is to show India 9. a Refer to the later part of the sentence which talks
in the best light. This means that they ‘project’ India in about a popular poll. Only the word ‘select’ is in sync
the best possible way. Hence, option (d) is the correct with ‘poll’. Hence, option (a) is the correct answer.
answer.
10. b In the previous sentence, the author talks about a ‘poll’
2. c Options (a), (b) and (d) can be ruled out since they for selecting a brand ambassador. Here he goes on to
render an incorrect meaning to the sentence. It is India’s asking a question that if such polls are conducted,
‘uniqueness’ that it is a cultural-soft superpower. who will you vote for. Hence, option (a) is the correct
Hence, option (c) is the correct answer. answer.
3. b Option (b) is the correct answer. Responsibility can 11. c ‘Descend’ means to be inherited or transmitted, as
be ‘important’. It cannot be ‘reluctant’ or ‘hesitant’. through succeeding generations of a family. Hence, it
‘Matured’ is incorrect because it is in the past tense. fits well in the blank.
4. c Only option (c) ‘power’, fits in grammatically. Hence, it 12. d ‘Breakthrough’ refers to a significant advance,
is the correct answer. Options (a), (b) and (d) will development, achievement in a particular field or
render the sentence incorrect. situation. In the given sentence it is used for the
advancement in information about the descent of
5. b It is clear from the passage that the Indian icons are humans.
gaining popularity, and therefore, we can say that
they are becoming global icons. Thus, option (b) is the 13. a The passage talks about the commonality between
correct answer. sea sponges and humans. So, the blank will take
‘connection’ to make the sentence grammatically
6. c The sentence gives examples of two well-known correct.
Indians and hence, option (c), ‘popular’, is the most
appropriate word to fill in the blank. 14. a ‘Lineage’ refers to lineal descent from an ancestor.
Hence, it fits the blank.
7. b Since the brand ambassadors touched the hearts of
15. b The sentence refers to the line of descent that was
the people across continents, the approach has to be
beyond the expectations of scientists. Hence,
‘simple’ and not ‘casual’, ‘royal’ or ‘rustic’. Hence, option
‘expected’ fits the blank.
(b) is the correct answer.
16. b ‘Pour’ is the correct option. ‘bestow into’ is
8. c The sentence means that the image the brand
grammatically wrong. ‘Fell’ (to cause to fall by striking;
ambassadors created for themselves has lived more
cut or knock down) does not fit in the meaning of the
than the brand ambassadors themselves. Hence, the
sentence. ‘Downturn’ means ‘a tendency downward,
correct answer is option (c).
especially in business or economic activity.’

VA / Exercise - 2 CEX-5322/P1BS/17 / Page 1


17. c From the passage it can be inferred that the fish are 25. b 'Gather' means to come together, or bring people
becoming resistant to the poison rotenone. Since the together, in one place to form a group. Hence, option
passage states that ‘The gods seem to be on the side (b) is the correct answer. 'Recipe' refers to a set of
of the fish’, it’s implied that the fish now survive. instructions that tells you how to cook something and
‘Indifferent’ is out of place. The fish cannot be the ingredients you need for it. 'Mortal' refers to
indifferent to a poison. ‘Garrulous’ means ‘given to something that cannot live for ever and must die. 'Portal'
constant and frivolous chatter; loquacious; talkative’. means a large, impressive gate or entrance to a building.
‘Vulnerable’ (susceptible to injury) is incorrect. It also means a website that is used as a point of entry
to the Internet, where information has been collected
18. b ‘Brink’ means ‘the extreme edge or margin; a border’.
that will be useful to a person interested in particular
Humans are hunting species to the brink of extinction.
kinds of things.
‘Blink’ means to close and open one or both of the
eyes rapidly. ‘Rink’ means an area surfaced with 26. a 'Larceny' means the crime of stealing something from
smooth ice for skating, hockey etc. somebody. Hence, option (a) is the correct answer.
'Peevish' means feeling or showing irritation. 'Impulse'
19. d ‘Wipe out’ means to destroy completely; eradicate.
means a sudden strong desire to do something.
According to the passage, many species living today
'Embroil' means to involve somebody/yourself in an
are likely to be destroyed completely over the next
argument or a difficult situation.
century or two. ‘Sopped’ means to dip, soak, or drench
in a liquid; saturate. ‘Ferret out’ means to uncover and 27. d 'Impunity' means exemption or freedom from
bring to light by searching. punishment, harm, or loss. Hence, option (d) is the
correct answer. 'Perjury' means the crime of telling a
20. d ‘Onslaught’ means a violent attack. ’Infatuation’ means
lie in a court of law after promising to tell the truth.
a foolish, unreasoning, or extravagant passion or
'Periphery' refers to the outside edge of an area; the
attraction. ‘Gratuity’ means a favor or gift, usually in
area that surrounds a place or thing. 'Imbroglio' refers
the form of money, given in return for service.
to a complicated situation that causes confusion or
‘Lividness’ means discoloration due to bruising.
embarrassment, especially one that is political.
21. b 'Numismatics' refers to the study or act of collecting
28. d 'Callow' refers to a young person who does not have
coins, paper money, and medals. Hence, option (b) is
much experience and does not know how to behave
the correct answer. 'Oblique' means not stated directly.
the way adults behave. Hence, option (d) is the
'Philately' is the hobby of collecting stamps. 'Economics'
correct answer. 'Hallow' means to respect greatly.
means the study of how a society organizes its money,
'Laggard' means a slow and lazy person, organization,
trade and industry.
etc. 'Bellow' means to shout in a loud deep voice,
22. c 'Hallucinate' means to see or hear things that are not especially because you are angry.
really there. Hence, option (c) is the correct answer.
'Incriminate (somebody)' means to make it seem as if 29. b 'Stratagem' means a trick or plan that you use to gain
somebody has done something wrong or illegal. an advantage or to trick an opponent. Hence, option
'Improvise' means to make or do something using (b) is the correct answer. 'Blaze' means to burn brightly
whatever is available, usually because you do not and strongly. 'Poignant' means having a strong effect
have what you really need. 'Percolate' means to move on your feelings, especially in a way that makes you
gradually through a surface that has very small holes feel sad. 'Willful' means something done deliberately,
or spaces in it. although the person doing it knows that it is wrong.

23. a 'Oology', or 'oölogy' is the branch of zoology that deals 30. d 'Autocrat' means a person (as a monarch) ruling with
with the study of eggs, especially birds' eggs. Hence, unlimited authority. Hence, option (d) is the correct
option (a) is the correct answer. 'Sanctuary' means answer. 'Pauper' means a very poor person. 'Misfit'
an area where wild birds or animals are protected refers to a person who is not accepted by a particular
and encouraged to breed. 'Morphology' is the form group of people, especially because his behaviour or
and structure of animals and plants, studied as a ideas are very different. 'Overt' means done in an
science. 'Embryology' is the scientific study of the open way and not secretly.
development of embryos.
31. a ‘Graphology’ refers to the study of handwriting, for
example as a way of learning more about somebody’s
24. b 'Larder' means a cupboard/closet or small room in a character. Hence, option (a) is the correct answer.
house, used for storing food. Hence, option (b) is the ‘Calligraphy’ refers to the art of making beautiful
correct answer. 'Lard' means a firm white substance handwriting. ‘Aesthetic’ means concerned with beauty
made from the melted fat of pigs that is used in cooking. and art and the understanding of beautiful things.
'Ladle' means a large deep spoon with a long handle, ‘Ambidexterity’ means the quality or state of being
used especially for serving soup. 'Loiter' means to ambidextrous, which means able to use the left hand
stand or wait somewhere especially with no obvious and the right hand equally well.
reason.

CEX-5322/P1BS/17 / Page 2 VA / Exercise - 2


32. c ‘Hedonist’ refers to a person who believes that 50. d None of the spellings are correct. The correct spellings
pleasure is the most important thing in life. Hence, are ‘bedouin’, ‘reprisal’, ‘indeterminable’ and
option (c) is the correct answer. ‘Fascist’ refers to a ‘cadaverous’.
person who supports fascism. Fascism is an extreme
51. a The correct spelling is ‘chauffeur’.
right-wing political system or attitude which is in favour
of strong central government and which does not 52. c The correct spelling is ‘speculative’.
allow any opposition.
53. a The correct spelling is ‘bankruptcy’.
33. d ‘Epitaph’ refers to something written or said in memory
of a dead person; especially words written on a 54. a The correct spelling is ‘unanimous’.
gravestone. Hence, option (d) is the correct answer. 55. d The correct spelling is ‘veterinary’.
‘Epigraph’ means a quotation set at the beginning of a
literary work or one of its divisions to suggest its
theme. ‘Fresco’ refers to the art of painting on wet
Practice Questions
plaster.
1. d The whistleblowers did not gain any advantages or
34. a ‘Theology’ refers to the study of religious faith, practice, benefits. So options (a) and (b) are eliminated like
and experience: the study of God and God’s relation wise they did not acquire any privileges and hence
to the world. Hence, option (a) is the correct answer. option (c) is eliminated. Hence, option (d) is the correct
‘Religion’ means the belief in a god or in a group of answer.
gods. ‘Ontology’ refers to a branch of metaphysics
concerned with the nature and relations of being. 2. c The whistleblower was not a spy. Refused and
‘Doctrine’ refers to a set of ideas or beliefs that are declined in this context are grammatically and
taught or are believed to be true. semantically wrong. ‘Challenged’ is completely
meaningless. Hence, options (a), (b) and (d) are wrong.
35. d ‘Transaction’ refers to an exchange or transfer of The correct answer is option (c).
goods, services, or funds. Hence, option (d) is the
correct answer. ‘Remuneration’ means an amount of 3. c ‘Started’, ‘fired’ and ‘facilitated’ can all fit in. But they
money paid to someone for the work that person has lack the intensity of ‘triggered’. When you pull a trigger
done. ‘Alimony’ refers to money that a court orders you can set off anything from a small pistol to a huge
someone to pay regularly to a former wife or husband bomb. The debate set off by the leak is huge. Hence,
after a divorce. option (c) is the right answer.
36. d ‘Acquiesce’ is the correct spelling. 4. b Stories are not ‘announced’, ‘exposed’ or ‘disclosed’ in
newspapers. They are published. Hence, option (b) is
37. a ‘Philatelic’ is the correct spelling. the right answer.
38. b ‘Garrote’ is the correct spelling.
5. b ‘Consortium’ means ‘Association’ but is restricted, to
39. c ‘Imminent’ is the correct spelling. companies, banks etc. ‘Forum ‘and ‘Collaboration’ do
not fit in. Hence the correct answer is option (b).
40. c ‘Malfeasance’ is the correct spelling.
41. b ‘Crevasse’ and ‘scoundrel’ are correct. ‘Galeon’ should 6. c ‘Impasse’ means deadlock. Non of the other options
be spelt as ‘galleon’ and ‘pugnascious’ is spelt as mean deadlock. Hence, option (c) is the correct answer.
‘pugnacious’. 7. b We want a negative word here. Improve and brighten
42. b ‘Furtherance’ and ‘crotchet’ are correct. ‘Gaietey’ is being positive are ruled out. ‘Evaporate’ is much
spelt as ‘gaiety’ and ‘dominearing’ is spelt as stronger than ‘Weaken’. Hence, option (b) is the correct
‘domineering. answer.

43. c ‘Sebaceous’, ‘guillotine’ and ‘expatriate’ are correct. 8. d Option (a) is patently wrong. Option (c) is a grammatical
‘Heinuous’ is spelt as ‘heinous’. misfit. Plan is weaker than scheduled. Hence, option
(d) is the correct answer.
44. c ‘Varigated’ should be spelt as ‘variegate’.
9. c Option (b) is wrong in this context because most and
45. d None of the spellings are correct. The correct spellings
guarded do not go together. In view of the word
are ‘secession’, ‘valediction’, ‘compendium’ and
‘warning’ option (a) is wrong. The IMF chief cannot be
‘tenacious’.
cantankerous. Hence, option (c) which means
46. a The correctly spelt words are ‘assiduous’, ‘clientele’ undisguised is the right answer.
and 'baccalaureate'.
10. a Option (a) is the strongest of the four terms. Hence,
47. c ‘Gurilla’ is wrongly spelled. The correct spelling is option (a).
‘guerrilla’.
11. d Only option (d) fits in here.
48. b ‘Vigilance’ and ‘tautology’ are incorrectly spelt.
49. c ‘Instillation’ is incorrectly spelt.

VA / Exercise - 2 CEX-5322/P1BS/17 / Page 3


12. c Option (b) is meaningless. Options (a) and (c) can be 27. d Diatribe is abuse, fisticuffs is a fight passage-at-arms
used with target, but they are not relevant here. A is an armed fight. Hence, all of them are wrong. The
realistic target is more important than a tight or clear correct answer is option (d).
target. Hence, option (c) is the right answer.
28. b The correct collective noun for foxes is skulk. Hence,
13. b Strict measures and statistical measures are
option (b) is the correct answer. Husk is hares and
meaningless in this context. Hard nosed means realistic
siege is for herons.
and tough minded. They are not meaningless in this
context. However, credible or believable is more
29. c The correct collective noun here is kit. Hence option
relevant here than hard nosed. Hence option (b) is the
(c) is the correct answer.
right answer.
14. d The IMF chief is sorry and unhappy. Hence, option (d) 30. a Theophany is a visible manifestation of God to human
is the only one which fits in. kind. Cccophony is harsh sounds. Resurrection is the
return to life of a dead person (body). Hence, the
15. b Abortive means fruitless. Now of the measures taken
correct answer is option (a).
so far have borne fruit. Hence, option (b).
16. b Welfare and poverty cannot be developed. Power 31. b Phage means killer or eater of bacteria. Option (c)
cannot be developed though the power sector can be means cannibals. Option (a) is part of the human body.
developed. Hence, option (b) is the correct answer. Hence, the correct answer is option (b).
17. b Option (a), (c) and (d) can all be used. But none of
32. d Correct answer is option (d).
them is as strong as ‘Counter Productive’ which means
producing results opposite to those desired. Hence,
33. a Correct answer is option (a).
option (b).
18. a Existing alliances can be expanded or reinforced. New 34. c Correct answer is option (c).
alliances can be established. But establishment comes
only after exploration. Hence, option (a) is the correct 35. d Correct answer is option (d).
answer.
36. b Correct answer is option (b).
19. c Alienating means antagonizing and hence is the
strongest of all four options. Hence, the correct
37. d Correct answer is option (d).
answer is option (c).
20. c Liability being negative is ruled out. Options (b) and (d) 38. a Correct answer is option (a).
do not fit in grammatically. Hence, the correct answer
is option (c). 39. c Correct answer is option (c).

21. d A periscope is used by a submarine, a stethoscope 40. c Correct answer is option (c).
by a physician a stereoscope by a photographer. The
correct answer is option (d). 41. d Correct answer is option (d).
22. b Adulation is praise, eloquence is fluencey of speech,
recondite is something which is difficult to understand. 42. b Correct answer is option (b).
Hence, option (b) is the correct answer.
43. c Correct answer is option (c).
23. c Mare Clausum petains to a sea, confluence means the
flowing together of two or more rivers and riparian is 44. b Correct answer is option (b).
a legal term pertaining to rivers. Hence, option (c) is
the correct answer. 45. c Correct answer is option (c).
24. d Option (a) and (b) have a political connotation. Option
46. d Correct answer is option (d).
(c) is related to the Prophet. Hence, option (d) is the
correct answer.
47. a Correct answer is option (a).
25. a Ruritanian means pertaining to the imaginary country
called Ruritania. Pasto rate means the territory looked 48. c Correct answer is option (c).
after by a pastor. Bilious means hostile. Hence, the
correct answer is option (a). 49. d Correct answer is option (d).

26. c A sling is used for small stones. A rack is an instrument 50. b Correct answer is option (b).
of torture. A hammock is a cloth or net used as a bed
in a garden. Hence, the correct answer is option (c). 51. d Correct answer is option (d).

CEX-5322/P1BS/17 / Page 4 VA / Exercise - 2


Verbal Ability – 3 P-1(BS)

Idioms and Phrasal Verbs


Answers and Explanations
1 d 2 a 3 d 4 b 5 d 6 a 7 c 8 c 9 d 10 c
11 b 12 a 13 a 14 a 15 c 16 c 17 d 18 a 19 a 20 b
21 a 22 a 23 b 24 a 25 b 26 a 27 c 28 a 29 b 30 d
31 a 32 b 33 a 34 c 35 a 36 a 37 c 38 a 39 a 40 b
41 a 42 b 43 a 44 b 45 c 46 a 47 b 48 a 49 c 50 c
51 d 52 c 53 a 54 c 55 a 56 a 57 d 58 a 59 d 60 b
Practice Questions
1 c 2 c 3 d 4 c 5 d 6 c 7 d 8 d 9 d 10 d
11 c 12 c 13 d 14 c 15 d 16 c 17 d 18 d 19 c 20 d
21 d 22 d 23 c 24 d 25 c 26 c 27 d 28 c 29 d 30 c
31 d 32 d 33 c 34 d 35 d 36 d 37 d 38 c 39 d 40 c
41 d 42 d 43 d 44 c 45 d 46 b 47 d 48 d 49 b 50 b
51 a 52 b 53 a 54 d 55 b 56 c 57 d 58 c 59 b 60 c
61 c 62 a 63 d 64 b 65 c 66 c 67 d 68 b 69 c 70 d
71 c 72 b 73 c 74 d 75 c 76 a 77 c 78 d 79 b 80 c
81 d 82 c 83 c 84 d 85 d 86 c 87 d 88 c 89 c 90 d
91 d 92 d 93 c 94 d 95 d 96 b 97 d 98 c 99 b 100 d

1. d ‘Kick the bucket’ means to die. 12. a To describe a situation as a 'can of worms' means that
it is a situation that causes a lot of problems for you
2. a ‘Feather your own nest’ means to take advantage of when you start to deal with it.
the opportunities to get a lot of money for yourself.
13. a 'Half the battle' is used to refer to an important step
3. d ‘Go off the deep end’ means to give way immediately towards achieving something.
to an emotional outburst, especially of anger.
14. a 'Put the hard word on' means to ask a favour of
4. b ‘To call a spade a spade’ is to tell the truth about someone, especially a financial favour.
something, even if it is not polite or pleasant.
15. c 'Be chilled to the marrow' means to be extremely cold.
5. d ‘To sit on the fence’ is to be unable to take a decision.
16. c Wait on someone hand and foot: To serve someone
6. a ‘All Greek to me’ means something that one does not very well
understand. A heavy hand: Tyranny, persecution, or oppression
A free hand: Freedom to do as desired
7. c ‘Cut to the chase’ means to be specific and to the
From hand to mouth: Providing only bare essentials
point.
17. d Come again: Used as a request to repeat what was
8. c ‘Field day’ means an activity, or time spent in an activity, said.
that is enjoyable and fun. Come down on: To punish, oppose, or reprimand
severely and often with force
9. d To have a ‘gut feeling’ means to have a sense or a
Come a cropper: To fail utterly
certain feeling about a person or a situation, without
Come clean: To confess all
knowing the rational basis for such a feeling.
18. a Pick and choose: To select with great care.
10. c To ‘give someone the slip’ means to run away from Pick holes in: To seek and discover flaws or a flaw
problems or from any difficult situations. Pick (one’s) way: To find passage and make careful
progress through it
11. b If you 'have your back to the wall', you are in serious
Pick (someone) to pieces: To criticize sharply
difficulty.

VA / Exercise - 3 CEX-5323/P1BS/17 / Page 1


19. a Give (someone) a hard time: To make life difficult for; 38. a ‘To have a whale of time’ is to enjoy greatly.
harass
Give rise to: To be the cause or origin of; bring about 39. a ‘Change their tune’ means to alter approach or attitude.
Give or take: Plus or minus a small specified amount Hence, option (a) is the correct answer.
Give ground: To yield to a more powerful force; retreat
40. b 'Crank something out' means to produce something
20. b Bring home: To make perfectly clear quickly or carelessly; to make something in a casual
Bring home the bacon: To earn a living, especially for and mechanical way.
a family
41. a ‘Feeling under the weather’ means to be unwell.
Bring to bear: To exert; apply
Bring to light: To reveal or disclose 42. b 'Wishy-washy' means lacking in strength of character
or purpose; ineffective.
21. a ‘Dawned on’ is the correct idiomatic usage in the
sentence as ‘dawn on’ means to become suddenly 43. a A bull in a China shop’ means to behave clumsily or
clear to (a person). awkwardly.
22. a ‘Raking in’ is the correct idiomatic usage in the sentence 44. b 'Far-fetched' means not likely to happen or be true.
as it means to earn large sums of money.
45. c ‘Hot on heels’ means to follow someone close and try
23. b ‘What’s eating you’ is the correct phrase. It means to catch up.
what is worrying or annoying you?
46. a ‘Carry/take coals to Newcastle’ means to supply
24. a ‘Beeline for’ is the correct phrasal usage as it means something to a place or person that already has a lot
hurry directly to. The phrase, used in this context, of that particular thing.
describes the rush hour at the metro.
47. b ‘Fiddle while Rome burns’ means to work as normal
25. b ‘Cut above somebody/something’ is the correct idiomatic and neglect priorities during a crisis.
usage as it means to be better than other people or
things. 48. a ‘Salad days’ means the days of one’s youth when one
has little experience.
26. a The idiom ‘come what may’ means no matter what
happens. 49. c ‘Fly on the wall’ alludes to the position of being able to
freely observe a situation without being noticed.
27. c The explanation of the idiom ‘to cry over spilt milk’ is
incorrect as it means to cry or complain about 50. c ‘Out of the woods’ means a state of being free from
something that has already happened. difficulty or perils, to be safe.

28. a The explanation of the idiom ‘rule the roost’ is incorrect 51. d ‘To hide one’s light under the bushel’ means to conceal
as it means to be in charge. ‘To revel’ means to take one's talents or positive qualities.
great pleasure or delight (usually followed by in).
52. c ‘Busman’s holiday’ means a holiday where you do the
29. b The explanation of the idiom ‘to call in’ is incorrect as it same things you do at work.
means to summon for assistance or consultation.
53. a ‘Like a moth to a flame’ means to be irresistibly and
30. d The explanation of the idiom ‘to keep one’s eyes dangerously attracted to something or someone.
skinned’ is incorrect as it means to pay attention; be
watchful. 54. c ‘To throw caution to the wind’ means to become very
careless; to take a risk; to behave recklessly.
31. a ‘Have your hands full’ means to be very busy or too
busy to do something else. 55. a ‘To talk turkey’ means to have a frank discussion. It is
32. b ‘The long and (the) short of it’ is used when you are often used in a business context.
telling somebody the essential facts about something
56. a ‘Wheels within wheels’ implies complex interacting
or what effect it will have, without explaining all the
processes, agents, or motives.
details.

33. a ‘Lead up to’ means to make someone think something. 57. d ‘Ride a/the wave of something’ means to be very
successful for a limited period of time
34. c ‘To cut a poor figure’ means to appear or behave badly.
58. a ‘Nose round’ means to look around a place; to look for
35. a ‘The tip of the iceberg’ means only a small part of a something in a place.
much larger problem.
59. d ‘Rushed off one’s feet’ means to be very busy.
36. a ‘Talk out of’ means to dissuade from by talking

37. c 'Put up' means to provide lodgings for. 60. b ‘Cheek by jowl’ means very close together.

CEX-5323/P1BS/17 / Page 2 VA / Exercise - 3


Practice Questions 24. d Option (d) is the correct answer. If prospective soldiers
are of acceptable quality they pass muster and are
1. c Option (c) is the right answer. admitted to the army.
2. c Option (c) is the right answer. To have feet of clay 25. c Option (c) is the correct answer. Old horses past their
means to have faults which are not immediately prime were not killed but put out to pasture.
apparent.
26. c Option (c) is the correct answer. If you are not the
3. d Option (d) is the correct answer. When one is forced only pebble then there are other pebbles i.e. other
to work very hard or very fast one cannot always important people.
keep one’s feet firmly on the ground.
27. d Option (d) is the correct answer. A broad church admits
4. c Option (c) is the correct answer. It literally means people of all faiths.
displaying your flag (your affiliation) on the mast.
28. c Option (c) is the correct answer. Among animals there
5. d Option (d) is the correct answer. This idiom is used is an order of seniority according to which they may
humorously to suggest that one has come last in a approach the food and feed.
contest.
29. d Option (d) is the correct answer. If you have paid the
6. c Option (c) is the right answer. Clover is a plant that money your have the right to call the shots.
grows wild.
30. c Option (c) is the correct answer.
7. d “To coin” refers to creating a new phrase word.
31. d Option (d) is the correct answer. A ‘petard’ is a cracker
8. d Option (d) is the correct answer. The man at the coal or a bomb. Meaning of idiom is to be blown up by one’s
face does the actual work of hewing the coal. own bomb.
9. d “Clockwork precision” refers to something very 32. d Option (d) is the correct answer.
organized.
33. c Option (c) is the correct answer.
10. d Option (d) is the right answer. The cloak of a priest is
referred to as cloth. 34. d Option (d) is the correct answer.

11. c Option (c) is the correct answer. 35. d Option (d) is the correct answer.
36. d Option (d) is the correct answer. The fur coat is the
12. c Option (c) is the correct answer. When one escapes
visible part of the dress.
a mishap by a whisker one refers to it as a close call.
37. d Option (d) is the correct answer.
13. d Option (d) is the correct answer. Mud is opaque. Nothing
is visible in mud. Hence, a sarcastic reference. 38. c Option (c) is the correct answer. Flit means to move
noiselessly. Moonlight represents night.
14. c Option (c) is the correct answer. In days gone by
during battle the surgeons sometimes had to perform 39. d Option (d) is the correct answer.
operations without paraphernalia. To ensure that the
40. c Option (c) is the correct answer.
patient did not hurt himself the surgeon made him bite
a bullet. 41. d Options (a), (b) and (c) are correctly matched. Option
(d) is incorrect. The correct meaning is to stop
15. d Option (d) is the correct answer.
preparing for war and use the money to improve
16. c Option (c) is the correct answer. people’s lives.

17. d Option (d) is the correct answer. A knife is used to 42. d Options (a), (b) and (c) are correctly matched. Option
hurt, maim or kill someone. (d) is wrong. The correct meaning is to tell someone
something in confidence i.e. in secrecy.
18. d Option (d) is the correct answer. If a king were
abducted a huge amount would be demanded by way 43. d Options (a), (b) and (c) are correctly matched. The
of ransom. meaning of option (d) is incorrect. The correct meaning
is written down or in writing. It might also mean plain
19. c Option (c) is the correct answer. In a coin operated and simple.
telephone no talk was possible until the coin dropped.
44. c Options (a), (b) and (d) are correctly matched. Option
20. d Option (d) is the correct answer. (c) is incorrect. The correct meaning is to keep
switching one’s opinions from positive to negative and
21. d Option (d) is the correct answer.
back to positive again.
22. d Option (d) is the correct answer. Low wages attract
45. d Options (a), (b) and (c) are correctly matched. Option
people of limited abilities.
(d) is wrong. The correct meaning is to punish
23. c Option (c) is the correct answer. someone.

VA / Exercise - 3 CEX-5323/P1BS/17 / Page 3


46. b Options (a), (c) and (d) are correctly matched. Option 68. b Option (b) is the correct answer. The correct meaning
(b) is wrong. The correct meaning is to make someone is to indicate sexual interest.
understand something much more clearly than they
did before especially something unpleasant. 69. c Option (c) is the correct answer. It means to arrange.

47. d Options (a), (b) and (c) are correctly matched. Option 70. d Option (d) is the correct answer. It means ‘attacked’.
(d) is wrong. it means more of the samething. A busman 71. c Options (c) is the correct answer. To teem down is to
drives along a route everyday. On a holiday he travels rain heavily.
the same route as a passenger.
72. b Option (b) is the correct answer. ‘Talk at’ means to go
48. d Options (a), (b) and (c) are correctly matched. Option on talking without listening.
(d) is wrong. The correct answer is to secretly love
someone who does not love you. 73. c Option (c) is the correct answer. Means a period of
time leading to an important event.
49. b Options (a), (c) and (d) are correctly matched. Option
(b) is wrong. The correct answer is ‘an unpleasant 74. d Option (d) is the correct answer. ‘A cut above’ means
event casts gloom over happy events.’ superior to.
50. b Options (a), (c) and (d) are correctly matched. Option 75. c Option (c) is the correct answer. It means admitting
(b) is wrong. The correct meaning is ‘a famous and everything truthfully.
often discussed event’. 76. a Option (a) is the correct answer. The meaning is to tell
51. a Options (b), (c) and (d) are correctly matched. Option someone something so that they can give their opinion.
(a) is wrong. The correct meaning is actions showing
77. c Option (c) is the correct answer. It means doing
an unwillingness to spend money.
something very fast.
52. b Options (a), (c) and (d) are correctly matched. Option
78. d Option (d) is the correct answer. The meaning is to
(b) is wrong. The correct answer is having to deal
make someone come out of the place where they are.
with people who criticise, attack or challenge you.
79. b Option (b) is the correct answer. It means to arrive in
53. a Option (a) is the right answer.
a manner which makes other people notice you.
54. d Option (d) is the correct answer. 80. c Option (c) is the correct answer. It means go to a
55. b Option (b) is the correct answer. place.

56. c Option (c) is the correct answer. 81. d Option (d) is the correct answer. The meaning is to get
a large amount of something.
57. d Option (d) is the correct answer.
82. c Option (c) is the correct answer. It means to tolerate.
58. c Options (c) is the correct answer. Options (b) and (d)
are grammatically incorrect. Option (a) is idiomatically 83. c Option (c) is the correct answer.
in correct. 84. d Option (d) is the correct answer.
59. b Option (b) is the correct answer. Options (a), (c) and 85. d Option (d) is the correct answer.
(d) are grammatically incorrect.
86. c Option (c) is the correct answer.
60. c Option (c) is the right answer. To bring around means 87. d Option (d) is the correct answer.
to revive or bring back to consciousness.
88. c Option (c) is the correct answer.
61. c Option (c) is the correct answer. To call back means
to return. 89. c Option (c) is the correct answer.
90. d Option (d) is the correct answer.
62. a Option (a) is the correct answer. To pick on means to
criticise or treat unfairly. 91. d Option (d) is the correct answer.
63. d Option (d) is the correct answer. Options (a), (b) and 92. d Option (d) is the correct answer.
(c) are meaningless. To make for means to depart in 93. c Option (c) is the correct answer.
the direction of.
94. d Option (d) is the correct answer.
64. b Option (b) is the correct answer and stuck up for
means defended. Defended is grammatically incorrect. 95. d Option (d) is the correct answer.
96. b Option (b) is the correct answer.
65. c Option (c) is the correct answer and means ‘spends’.
97. d Option (d) is the correct answer.
66. c Option (c) is the correct answer. The correct meaning
is spared no effort and left nothing to chance. 98. c Option (c) is the correct answer.

67. d Option (d) is the correct answer. The meaning is to 99. b Option (b) is the correct answer.
attack or criticise someone. 100. d Option (d) is the correct answer.

CEX-5323/P1BS/17 / Page 4 VA / Exercise - 3


Verbal Ability – 4 P-1(BS)

Vocabulary Revision Test


Answers and Explanations
1 c 2 d 3 d 4 b 5 c 6 b 7 a 8 c 9 c 10 d
11 b 12 b 13 d 14 d 15 b 16 d 17 a 18 b 19 a 20 b
21 b 22 c 23 d 24 c 25 d 26 b 27 a 28 a 29 c 30 c
31 d 32 b 33 a 34 a 35 c 36 c 37 a 38 d 39 a 40 d
41 b 42 c 43 a 44 b 45 a 46 c 47 b 48 a 49 c 50 a
51 c 52 a 53 d 54 b 55 c 56 a 57 d 58 b 59 c 60 a
61 c 62 a 63 c 64 c 65 d 66 d 67 b 68 b 69 c 70 b
71 a 72 c 73 a 74 d 75 b 76 a 77 c 78 b 79 d 80 a
81 b 82 d 83 c 84 d 85 d 86 a 87 c 88 b 89 c 90 d
91 b 92 a 93 c 94 d 95 b 96 c 97 a 98 c 99 b 100 b
101 d 102 a 103 c 104 b 105 b 106 b 107 b 108 c 109 d 110 a

1. c ‘Renounce’ and ‘abdicate’, both mean giving up 11. b An ‘aberration’ is something abnormal. Hence, it is the
something. exact opposite of ‘routine’ or ‘mundane’.
2. d ‘Abnegation’ means self-denial. 12. b A ‘hedonist’ is devoted to the pursuit of pleasure and
3. d ‘Placate’ and ‘soothe’ are synonyms. They mean to believes that pleasure is the only good. A ‘puritan’, on
cause someone to feel less energy about something. the other hand, believes that pleasure is wrong.

4. b 'Vociferous' refers to someone who expresses 13. d ‘Truncate’ means to make something shorter. ‘Lengthen’
feelings or opinions in a loud or forceful way. Hence, is the exact opposite of this.
option (b), loud, is the synonym of the word.
14. d A person who is rude or shows lack of respect is
5. c 'Fictional' refers to something that is not true. Hence, referred to as 'impertinent'. Its antonym will be
option (c), fanciful, is the correct answer. Options (a), 'courteous'. Options (a), (b) and (c) are the synonyms
(b) and (d) are the antonyms of the word. of the word.

6. b ‘Perfunctory’ means characterized by routine or 15. b 'Divulge' means to make someone known of some
superficiality; lacking in interest or enthusiasm. information or to publicize. 'Dissemble' means to hide
‘Uninterested’ is its synonym. true feelings, opinions etc.

7. a ‘Bucolic’ and ‘pastoral’ are synonymous and both are 16. d A ‘balm’ eases the pain. Therefore ‘irritant’ is its
related with rural life and countryside. antonym.

8. c ‘Anathema’ pertains to a person or thing that is detested 17. a ‘Spur’ means to boost. So, ‘disincentive’ is its antonym.
or loathed. Its synonym is ‘curse’.
18. b ‘Loquacious’ means talkative whereas someone who
9. c ‘Impecunious’ means having little or no money; is reticent is reserved.
penniless; poor. Option (c), ‘penurious’ means the
same. 19. a ‘Bestial’ means like a beast, so ‘angelic’ is its antonym.

10. d To ‘macerate’ is to leave food in a liquid so that it 20. b ‘Itinerant’ is someone migrating from one place to
absorbs the liquid and becomes soft or to become another. ‘Sedentary’ pertains to someone who stays
soft in this way. So, option (d) is its synonym. in one place.

VA / Exercise - 4 CEX-5324/P1BS/17 / Page 1


21. b Since the sentence refers to the games which are 35. c 'Palpable' is an adjective that means readily or plainly
called ‘mindless marathons’, only ‘tedious’, which seen, heard, perceived, etc. 'Obvious' and 'evident',
means long and tiresome, logically fits in the meaning options (b) and (c), are synonyms of 'palpable'.
of the sentence. Hence, option (b) is the correct 'Obscure' means not clear or plain; ambiguous, vague,
answer. ‘Scrupulous’ means very careful about doing or uncertain. It is an antonym of others words.
something correctly. ‘Invigorate’ means to give life and
energy to someone. 36. c 'Cataclysm' means flood. 'Deluge' refers to an extensive
flood. 'Inundate' means flood; cover or overspread
22. c Since the cat is feeling threatened, the best option to with water; deluge. 'Delude' means to mislead the mind
express this feeling would be option (c). or judgment of; deceive.
‘Consternation’ means a feeling of anxiety or dismay,
typically at something unexpected and ‘wariness’ 37. a 'Propitious' means indicative of favor; auspicious.
means caution about possible dangers or problems. 'Propensity', 'proclivity' and 'disposition' refer to a natural
inclination or tendency.
23. d The answer choices should be in contrast to each
other and should also be logically related to ‘courtship’. 38. d 'Obliterate', 'annihilate' and 'extirpate' mean to remove
Only option (d) fulfills both the conditions. ‘Fervor’ or destroy all traces of; do away with; destroy
means strong feeling of excitement and enthusiasm. completely. 'Aggrandize' means to widen in scope;
increase in size or intensity; enlarge; extend.
24. c The preposition ‘to’ is used in order to express
movement towards a place. Hence, option (c) is the 39. a 'Perturbation', 'delirium' and 'jitters' mean nervousness;
correct answer. a feeling of fright or uneasiness. 'Composure' refers
25. d The phrase ‘muddle up’ means to cause things to be to serene, self-controlled state of mind; calmness;
mixed in the wrong order. Hence, option (d) is the tranquility.
correct answer. 40. d 'Expedite' means to accelerate the process or progress
26. b The sentence suggests that Mamta can endure of: speed up, hasten or advance. 'Procrastinate'
something that her family cannot. So, the word in blank means to put off intentionally and habitually.
should go with ‘aloofness’. ‘Ostracism’ means to 'Procrastinate' is an antonym of the other words.
outcast. So, ‘ostracism’ best fits in the blank.
41. b 'Dull' is the antonym of 'vibrant'. Similarly, 'Bored' is the
27. a ‘Vacuous’ means lacking importance or substance. antonym of 'enthusiastic'. 'Limpid' means perfectly clear.
‘puerile’ means silly or childish especially in a way that
shows a lack of good judgment. ‘Ambivalent’ means 42. c 'Chocoholic' is someone who eats too much
having contradictory attitudes or feeling. ‘Litigious’ chocolates. Similarly, 'glutton' is a person who eats
means tending or likely to engage in lawsuits. Only too much food. Option (b) is incorrect because a
‘litigious’ fits in the meaning of the sentence and hence, 'sweet tooth' might not necessarily eat too many
is the answer. sweets. A 'teetotaler' is someone who abstains from
alcohol.
28. a Suffrage means the right vote in an election. Only
option (a) fits in the meaning of the sentence. 43. a Parent is the guardian of a child or ward.

29. c 'Candid' means expressing opinions and feelings in an 44. b House of state (Rajya Sabha) is a permanent house
honest and sincere way. When somebody is 'candid but, house of people is not a permanent house. Similar
and trustworthy, his statement can not be insincere. analogy can be drawn for the legislative assembly
and the legislative council in a state. Legislative council
30. c The sentence means that the paper intended to bring is the permanent house and legislative assembly is a
to light the forms of corruption and falsehood in public temporary house.
life. So, 'uncover' best fits in the blank.
45. a Just as a State is headed by a Governor. A Union
31. d 'Conforming' means behaving in a way that is accepted Territory is headed by a Lt. Governor.
by most people. Options (a), (b) and (c) are synonyms
of unconventional. 46. c A group of bats is called 'colony' whereas a group of
32. b 'Laconic' means using few words. Other options are baboons is called 'congress'.
used as adjectives to describe someone who is wordy 47. b 'Acrophobia' is the fear of 'heights' whereas
or use too many words. 'antlophobia' is the fear of floods.
33. a 'Crippled', 'handicapped', and 'retarded' have negative 48. a 'Sallow' means of a sickly yellowish hue or complexion
connotations and are no longer used because they while 'flushed' means having a healthy reddish color.
are considered offensive. Thus, 'sallow' and 'flushed' are antonyms.
'Concentrated' is the opposite of 'scattered'.
34. a 'Enervate' means lacking physical, mental, or moral
vigor. Options (b), (c) and (d) are its antonyms. 49. c 'Spirit' and 'ghost' are synonyms. Similarly, 'spirited'
and 'vigorous' are synonyms.

CEX-5324/P1BS/17 / Page 2 VA / Exercise - 4


50. a An 'invention' is protected through patent. Similarly, 64. c (a) Hand's turn : An act of manual labor; especially a
'literary work' is protected through a copyright. single usually small expenditure of effort
(b) High hand: An oppressive or dictatorial manner.
51. c If someone 'gatecrashes', they attend a private social
(c) The correct idiom is 'hand in hand'. It means
event without being invited.
together; jointly.
52. a If an idea, wish or promise is 'pie in the sky', it is good (d) Hand over fist: Very rapidly
but unrealistic and unlikely to be achieved.
65. d (a) Give birth to : To be the origin of
53. d If something takes a 'nosedive', it drops or decreases (b) Give rise to: To be the cause or origin of
in value very rapidly. (c) Give or take: Plus or minus a small insignificant
amount
54. b 'All singing, all dancing' means having a lot of advanced
(d) The correct idiom is 'give ground', which means to
technical features and therefore modern.
retreat.
55. c 'Apples and oranges' is used to describe a situation in 66. d 'Ballpark figure' is a rough or approximate number
which two people or things are completely different (guesstimate) to give a general idea of something.
from each other.
67. b If a thing is overly abundant or commonplace, we say
56. a A 'fly in the ointment' refers to a person or thing that it is 'a dime a dozen.'
spoils a situation.
68. b 'To keep up with the Joneses' means to make efforts
57. d 'Walking on air' means feeling very happy. to keep up with one's neighbors or peers in social
58. b 'To throw over' means to forsake or to abandon. In the status and prestige.
given sentence, because of certain issues in the party 69. c 'To rob Peter to pay Paul' means to take from one
the two members abandoned the party they merely to give to another; to discharge one debt by
themselves had founded. incurring another.
59. c If two people or groups are 'at loggerheads', they 70. b 'Lead someone a merry dance' means to cause
strongly disagree about something. The phrase is often someone a lot of trouble, especially by getting them to
used with the preposition 'with'. For example: They're do a lot of things that are not necessary.
constantly at loggerheads with the labor union. The
phrase is sometimes used with the preposition 'over'. 71. a The correct spelling is 'preceding'.
For example: The opposing parties are still at
loggerheads over the most crucial parts of the bill. 72. c The correct spelling is 'bailiwick', which means a
person's specific area of knowledge.
60. a 'By and large' means mostly or generally for the most
part. 73. a The correct spelling is 'consensus'.
61. c (a) A foot soldier : One who performs necessary but 74. d The correct spelling is 'supersede'.
basic, often mundane tasks.
(b) Pass muster : To measure up to the required 75. b The correct spelling is 'hypocrisy'.
standards.
(c) The correct idiom is 'head over heels'. It means 76. a The correct spelling is 'hieroglyphic'.
madly in love. 77. c The correct spelling is 'tantamount'.
(d) Off and on : Intermittently
62. a (a) The correct idiom to be used here is 'set about', 78. b The correct spelling is 'obstreperous'.
which means to make someone begin doing
something. 'Set something out' means to remove 79. d The correct spelling is 'espionage'.
something and place it so that it is available for
someone or some purpose. 80. a The correct spelling is 'defoliated'.
(b) Set the stage : To provide the underlying basis for
(c) Set in motion : To give impetus to 81. b 'Through' is the correct preposition here. 'Read through'
(d) On the square: Honestly and openly means to read all of something. Read about is incorrect;
the author doesn't want 'you' to read about the review
63. c (a) A square peg in a round hole: A misfit. of law firms but to read those very reviews. 'By' and
(b) Set straight: To make certain that someone 'for' will make the sentence grammatically incorrect.
understands something exactly
(c) The correct idiom is 'walk out on', which means to 82. d 'Give' doesn't complete the sentence. In the context of
desert or abandon. the passage, 'educate' fits in perfectly. The author
(d) Chip off the old block: A child whose appearance even mentions in the next sentence how one would
or character closely resembles that of one or the be able to give oneself a brief 'education'.
other parent.

VA / Exercise - 4 CEX-5324/P1BS/17 / Page 3


83. c 'Truth of the matter' is the correct phrase. It means to 'watch' are incorrect because one cannot 'think' or
introduce a fact which supports what you are saying 'watch' cause and effect but can observe.
or which is not widely known.
97. a The passage is in present tense so, maintaining the
84. d 'Cope' and 'deny' do not go well with 'yourself'. 'Acquit' tense consistency, 'ate' and 'had' can be negated. The
also doesn't make sense contextually. 'Defend' entire sentence and the overall passage tell us that it
perfectly fits in the context of the sentence. is eating the food that can upset health and not cooking.
Thus, the blank will take 'eat'.
85. d 'Which' is correct here as it points to some particular
questions that are important to ask. 98. c One can 'fall' sick, not 'turn' or 'become' sick. 'Were' is
incorrect because 'eating' is followed by 'sickness',
86. a 'Fragile' means something that is delicate and prone to
'were' gives the sense that one was sick even before
damage. 'Sensitive' suggests that the blank will take
eating, which is not the case here.
'fragile'. 'Weak' is used in comparison to the strength
of something else and here we have no such 99. b 'Caused' is the correct choice. If you read the whole
reference. 'Exploited' and 'tough' do not fit in with the sentence, it become clear that the meaning the author
context. wants to convey is that the food is the reason for the
sickness.
87. c 'Variety' and 'range' take the preposition 'of' after them.
'Assortment' refers to a miscellaneous collection of 100. b The correct phrasal verb is 'turns up' which means 'to
things or people, and hence not correct. Thus option happen unexpectedly'. 'Leads', 'holds' and 'causes'
(c), 'diversity', is the correct answer. are incorrect.
88. b The correct word here is 'considered'. "Hottest 101. d Only 'creates' fits in the blank. Others make the
hotspots" is a category or type of area for which sentence grammatically or logically incorrect.
Western Ghats have been considered. 'Treated',
102. a The correct verb here is 'are'. Since the sentence is in
'mentioned' and 'demarcated' are incorrect in this
simple present tense, 'were' and 'would' are incorrect.
context.
89. c 'Commits' is most in line with the context of the passage. 103. c One gets diagnosed 'with' an illness. 'For', 'of' and 'by'
denote an incorrect usage of the prepositions.
90. d The sentence talks about transmitting the ecological
sites to generations that will come later, and hence 104. b The sentence has to be in simple present tense, thus,
'future' is the correct option. Though 'next' also denotes 'becomes' is the correct answer.
a future generation, it is more specific and its scope is 105. b 'Dies' is the correct choice because the sentence is in
confined to the immediate 'next' generation. present tense. 'Told' and 'discussed' do not make sense
91. b From the context of the extract it is clear that protecting in the sentence.
the Western Ghats is neither an 'obstacle' nor a 'cause.'
106. b A person's fixed, permanent, and principal home for
'Challenge' is the most apt choice here given the later
legal purposes is called 'domicile'.
details about the area that needs to be protected.
'Work' doesn't fit in grammatically. 107. b 'Breviary' is a book of the prayers, hymns, psalms,
92. a The expert panel has not 'found', 'detected' or 'exposed' and readings for the canonical hours. The place where
the area. Rather, the area has been 'assessed' by the dead bodies are kept is called a 'mortuary'. 'Lapidary'
panel for its geographic spread and ecological is the art of cutting gems. 'Calamari' is squid used as
importance. food.

93. c 'Developed' and 'similar' do not fit in with the context of 108. c 'Interpellate' is to question (as a foreign minister)
the passage. 'Peculiar' means strange. It fits in the formally concerning an official action or policy or
meaning of the sentence. personal conduct. 'Interject' is to interrupt what
someone else is saying with (a comment, remark,
94. d One 'tables' a report or a finding, not an area. Thus, etc.).'Interpolate' means to put (something) between
option (a) is incorrect. 'Chosen' is incorrect as there is other things or parts.
no question of choice, as the paragraph does not talk
about other contenders for the special status from 109. d The scientific study of vocal sounds is called
amongst whom it has been chosen. Option (c), 'phonology'. 'Pomology' is the study of fruits.
'manipulated' is incorrect contextually. 'Treated' is the 'Phytopathology' is the study of plant diseases.
correct word here; the area is treated as an ecologically 'Physiology' is the study of the functions of living
sensitive zone. organisms.

95. b 'Lower' is contrary to the meaning implied in the 110. a Fear of snakes is called 'ophidiophobia'. 'Acrophobia'
sentence. Of the remaining three options, 'higher levels is the fear of heights. 'Cynophobia' is the abnormal
of protection' is closest to the context. fear of dogs. 'Trypophobia' is a claimed pathological
96. c Since the sentence starts in present tense, the correct fear of objects with irregular patterns of holes.
choice is 'observe' and not 'understood'. 'Thought' and

CEX-5324/P1BS/17 / Page 4 VA / Exercise - 4


Verbal Ability – 5 P-1(BS)

Subject Identification, Articles, Nouns, Phrase and Clause, Conjunction / Interjection

Answers and Explanations

16 b 17 a 18 a 19 c 20 b 21 c 22 b 23 a 24 d 25 a
26 a 27 a 28 b 29 b 30 a 41 c 42 b 43 b 44 c 45 b
46 c 47 b 48 a 49 d 50 a

Practice Questions
21 c 22 a 23 c 24 a 25 b 26 b 27 b 28 b 29 b 30 b
31 d 32 d 33 b 34 c 35 d 36 d 37 c 38 d 39 b 40 c
41 b 42 c 43 d 44 d 45 b 46 d 47 c 48 d 49 b 50 d

1. The doer of the action (hurry) is the ‘man’. So, the 12. We use 'a' before words that start with a consonant
subject of the sentence is ‘man’. or consonant sound and 'an' before words that start
with a vowel or vowel sound. So, the blank will take
2. The doer of the action (laugh) is ‘they’. So, ‘they’ is the ‘an’.
subject here.
13. A particular mall is being talked about in the sentence.
3. The doer of the action is ‘children’. So, the subject of So, the blank will take the article 'the'.
the sentence is ‘children’
14. No article is required here. Illnesses such as toothache,
4. The verb in the sentence is ‘shining’ and the thing that earache, flu do not require any article. However,
is shining is the ‘sun’. So, the subject is ‘sun’. illnesses such as headache and cold require the article
'a' before them.
5. The sentence talks about ‘grass’ which is greener,
thus making it the subject of the sentence. 15. Definite article 'the' is used to talk about musical
instruments.
6. The verb is ‘grow’ and what are growing are ‘flowers’.
So, the subject is ‘flowers’. 16. b No article is required before ‘meat’. ‘The’ is redundant.

7. The verb is ‘play’. The sentence has two subjects 17. a The correct phrase is 'after a long day' since we are
since two people are ‘playing’ – ‘I’ and ‘brother’. So, not talking about any particular long day. We are
the subjects are ‘I’ and ‘brother’. referring to a long day in general.

8. The verb is ‘existed’ and what has existed is ‘world’. 18. a Articles are omitted before productive nouns denoting
So, ‘world’ is the subject of the sentence. a unique position i.e., a position that is normally held at
one time by one person only.
9. The verb here is ‘brewed’. What is ‘brewed’ is ‘coffee’.
So, the subject here is ‘coffee’. 19. c 'The' is redundant in part (c). The correct phrase is 'to
get to work'.
10. In English, the subject of a command, order, or
suggestion — ‘you’, the person being directed — is 20. b 'At least' suggests that the sentence is not talking
usually left out of the sentence and is said to be the about particular checkpoints but suggests that
understood subject. So, ‘you’ is the subject here. spectators have to pass through three checkpoints
randomly. 'The' suggests that there are three particular
11. In the sentence, we are talking about a specific dog checkpoints that the spectators have to pass, which
and the specific neighbours. So, both the blanks will contradicts the idea suggested by 'at least'. So, the
take the article 'the'. correct phrase will be 'passing through at least three'.

VA / Exercise - 5 CEX-5325/P1BS/17 / Page 1


21. c 'Coward' is a noun. It is used for a person who is not 36. "I will be going" is the main clause while "to the market"
brave. The sentence requires a noun describing the and “in the afternoon” are phrases.
trait. So, 'coward' should be replaced by 'cowardice'.
37. "I think" and "she should go" are the clauses. “To see
22. b 'Gears' is incorrect. When used in relation to supplies, the doctor” is the adverbial clause.
foods or clothes needed for a special purpose, ‘gear’
is treated as non count. So, the correct word will be 38. "They were singing" is a clause while "in a loud voice"
‘gear’. is an adverbial phrase.

39. "He gave me a glass" is a clause while "full of water"


23. a The game is 'darts' while the pointed object that is
is an adjective phrase.
thrown at a dart board is called a 'dart'. Here, the
sentence refers to the game and therefore, 'dart' will 40. "In a red shirt" and "swimming championship" are the
be replaced by 'darts'. The verb 'is' is correct since adjective and noun phrases respectively.
'darts' is one of those plural nouns that take a singular
verb. 41. c An interjection that expresses pain is required in the
blank. So, 'ouch' best fits in the blank. 'Ah' is used to
24. d Generally uncountable nouns such as 'sugar' do not express a range of emotions including surprise,
have plural forms. However, in the sentence it means pleasure, sympathy, and realization.
'two cubes of sugar'. So, 'sugars' is correct. 'a little' in
part (c) is also correct. It means a small quantity of 42. b 'But' is used to introduce a statement that adds
milk. something to a previous statement and usually
contrasts with it in some way. The given sentence
brings about that contrast by saying that although the
25. a 'All' in part (a) suggests that plural form of 'diagnosis'
bus stopped, the man didn't get off. 'And' would have
should be used. So, it should be replaced by been correct had the sentence said that the man got
'diagnoses'. 'Analyses' is plural and therefore, correct. off: 'The bus stopped and the man got off.' Here it
doesn't bring about a contrast; it simply joins two
26. a As a noun, 'nuptial' usually takes a plural form. So, the statements: 'The bus stopped' and 'the man got off'.
correct word will be 'nuptials'.
43. b 'Darn', which shows disappointment, fills in the blank
27. a 'Mean' can either be used as a singular or as a plural. appropriately.
However, when it implies wealth, it is always used as
a plural. So, 'mean is' should be replaced by 'means 44. c 'Shoo' is a word said to frighten or drive away a
are'. person or animal. Since, the cat is drinking milk from
the woman's cereal bowl, the woman will try to drive
her away and not feel happy about it.
28. b 'Electronic' is incorrect. 'Electronics' looks plural but is
actually singular. So, 'electronic' should be replaced
45. b 'Although' means in spite of the fact that.
by 'electronics'.
46. c 'Yet' means but at the same time or but nevertheless.
29. b The plural of 'looker-on' is 'lookers-on'. Only 'yet' fills in the blank to make it grammatically
correct. 'As though' means as would be the case if.
30. a The correct phrase is 'many a man'. 'A' before 'men'
suggests that the word should be 'singular'. 47. b Although 'either…or' is the correct structure, 'either'
will not be used here. It suggests an unavoidable
31. "Although she is in need of money" is the dependent choice between alternatives and this is not the case
clause while "she always gives alms to the beggars" here. The blank will take 'whether', which is used to
is the independent clause. introduce a single alternative, the other being implied
or understood.
32. "A box of silverware" is a noun phrase.
48. a 'Dear me' expresses surprise. 'Alas' expresses grief
33. "They left the hotel" is an independent clause while or pity. 'Doh' expresses frustration or realization. 'Uh'
"as fast as they could" is an adverbial phrase. expresses hesitation. Here, a word that expresses
surprise is needed. So, 'dear me' is the correct
34. "I found the child" is the clause while “hiding under the answer.
bed” is the adjectival phrase.
49. d 'Unless' means except if. Other options will make the
35. "I saw her in France" is the main clause. “where I had sentence erroneous.
gone during my vacations” is an adverbial clause.

CEX-5325/P1BS/17 / Page 2 VA / Exercise - 5


50. a 'Huh' is used to express scorn, anger, or surprise. 16. The correct article is ‘A’. See above.
'Bah' is an expression of contempt or disagreement.
'Hey' is used to call for attention. 'Hmm' shows 17. The and X (no article required), what Sir Don Bradman
agreement. Only 'huh' fits in the meaning of the was to Australian cricket Tendulkar has been to Indian
sentence. cricket. Hence, ‘The Bradman’.

Practice Questions 18. X (no article required).

1. ‘John’ is the subject of the principal verb ‘stood’. 19. X (No article required).

2. ‘The gold medal’ is the nominative phrase and therefore 20. ‘An’. Although the word honourable begins with the
the subject. If we break it up medal is a noun and the consonant ‘h’ it is pronounced as ‘Onerable’ i.e. word
subject proper, ‘gold’ is an adjective and ‘The’ is an beginning with a vowel sound. Hence, ‘An’ is the right
article. article.

3. ‘Rev. Desmond Tutu’ is the subject of the principal 21. c The error is in part (c). It is not clear who was walking
verb ‘is held’. barefooted – Ram or the scorpion. Hence, the
correction is ‘While he was walking barefooted...”
4. ‘Raymond’ is the subject of the principal verb ‘attended’.
22. a The correction is ‘in the garden”. Pace up and down
5. ‘Jack and Jill’ are the subjects of the verb ‘went’. Jack means to walk nervously because you are worried.
and Jill are two singular nouns joined by ‘and’. The alternative acceptable structure is ‘paced the
garden”.
6. ‘The grizzled captain’ is the subject of the verb
‘surveyed’. 23. c Part (c) of the sentence is wrong. It should read ‘and
picked it up’. While talking of babies it is customary to
7. ‘Matron’ is the subject of the verb ‘glared’. use the neuter gender because it is likely that the sex
of the baby is not known.
8. ‘Sarla’ is the subject of the verb ‘marched’.
24. a Part (a) is wrong, “you only” is a literal translation from
9. ‘Darjeeling’ is the subject of the verb ‘is located’. Hindi. The correct expression is “you yourself told
me”.
10. ‘The principal cause’ is the subject of the verb ‘is’
25. b Part (b) is wrong because it is a literal translation from
11. The correct article is ‘The’ because there is only one Hindi. The correct expression is “right then to return”.
New York.
26. b Correct expression – “the receptionist said he wasn’t
12. Article ‘The’ should be used because there is only one free. “Error of tense”.
doyen and hence it is particular.
27. b The error is in part (b). It should read “and which sells
13. X (i.e. no article) and ‘a’. Articles are not used before for Rs. 500 per copy”. In a ‘non-defining’ clause that
proper nouns and ‘India’ is a proper noun. cannot be used in place of which.

14. The correct article is ‘A’ Daniel wa a judge famous in 28. b Part (b) is wrong. It should read “that ever lived on
historical lore for his astute judgements. Justice earth”. The superlative is followed by ‘that’ not ‘who’.
Anantraman is being compared with Daniel. In other
words he is another Daniel. This converts Daniel into 29. b Error of tense. Part (b) should read “who lends you an
a common noun and hence the article ‘A’. umbrella when the sun is shining”.

15. The correct article is ‘The’. ‘Few people’ would mean 30. b Error of tense. Part (b) should read “is said to have
practically no one. ‘A few people’ would mean a handful been”.
of people – no one in particular. ‘The few people’ means
only a few people came and these particular few 31. d ‘By dint of’ is the correct preposition.
people were served a lavish lunch.

VA / Exercise - 5 CEX-5325/P1BS/17 / Page 3


32. d Option (d) is the correct choice. In lieu of means in 42. c Option (c) gives the reason for the refusal. Hence,
place or instead of. correct. The other options can be fitted in but do not
yield a forceful sentence.
33. b Option (b) is the correct answer.
43. d Option (d) meaning “during the period when” is the
34. c Option (c) is the correct answer. correct answer.

35. d Option (d) is the correct answer. 44. d Options (b) and (c) are meaningless. Option (a) is
grammatically correct but semantically wrong. Hence,
36. d Option (d) is the correct answer. In consequence of option (b) is the correct answer.
means “as a result of”.
45. b Option (b) meaning “inspite of that” is the correct
37. c Option (c) which means “regarding” is the correct answer.
answer.
46. d Option (d) is the correct answer.
38. d Option (d) is the correct answer.
47. c Option (c) is the correct answer. It expresses the
39. b Option (b) is the correct answer. emotion of joy and celebration.

40. c Option (c) is the correct answer. Options (a), (b) and 48. d Option (d) expressing surprise is the correct answer.
(d) are wrong or irrelevant.
49. b Option (b) expressing extreme approval is the correct
41. b Option (d) is incorrect because of double negative. answer.
Options (a) and (c) are meaningless. Hence the correct
answer is option (b). 50. d Option (d) expressing surprise and concern is the
correct answer.

CEX-5325/P1BS/17 / Page 4 VA / Exercise - 5


Verbal Ability – 6 P-1(BS)

Pronouns, Adjectives, Adverbs, Prepositions


Answers and Explanations
1 a 2 b 3 d 4 a 5 b 6 d 7 c 8 b 9 a 10 b
11 c 12 a 13 b 14 a 15 a

Practice Questions
1 c 2 d 3 d 4 c 5 c 6 c 7 d 8 c 9 c 10 d

1. a 'Which' is used in sentences that contain extra also been correct. Then it would have meant that we
information that could be left out of the sentence without appreciate the person who contacted the office.
affecting the meaning or structure. If the meaning gets
changed, then 'that' is used. In this case, if we remove 9. a ‘Ships’ are referred to as female. So, here 'her' will be
the part after 'unemployment', the meaning of the used.
sentence still remains the same. Therefore, 'which' is
the correct answer. 'Who' and 'whom' are used to 10. b The correct pronoun is ‘me’. The object of the verb is
refer to people. ‘accompanied’.

2. b 'Who' and 'whom' are used to refer to people. ‘Who’ 11. c 'Tallest' is the superlative degree, which is used when
should be used in the subject position in a sentence, comparison is made between more than two people.
while ‘whom’ should be used in the object position. In While comparing between two people, comparative
the sentence, 'whom' will be used, as it is the object of degree is used. So, 'tallest' should be replaced by
‘admire’. The subject is 'student'. 'taller'.

3. d To check which pronoun will fit in the blank, we will 12. a 'Troubling' is a verb. The sentence requires an adjective
remove the noun - girl, and then decide which of the to describe the 'counselors' just as 'doubtful' describes
options follow. So, the sentence now becomes "The 'allies'. So, 'troubling' should be replaced by
horror movie didn't frighten ________ at all". Clearly, 'troublesome'.
only 'us' fits in the blank to make it grammatically correct.
13. b 'Many' modifies things that can be counted while 'much'
4. a In the sentence, ‘he hasn’t visited’ is the non-restrictive modifies things that can't be counted. Since 'hair' cannot
clause, i.e., if we removed this clause, the meaning of be counted, 'many' in part (b) should be replaced by
the sentence does not change. So, the blank will take 'much'.
‘which’.
14. a 'Little' is used with uncountable nouns in the positive
5. b 'Whose' is used to show which person or thing you degree. 'Less' is the comparative form of 'little'. It is
are talking about. Marylyn is the owner of the book. used in comparative structures, especially before
‘Whose’ is the possessive form of ‘who’. So, 'whose' uncountable nouns. So, 'little' should be replaced by
best fits in the blank. 'Whosoever' means whoever. 'less'.

6. d The appropriate adjective in the first blank is 'my' 15. a Both 'less' and 'lesser' are used with uncountable
because here we need a possessive adjective as a noun. However, 'lesser' is used for 'quality' while 'less'
subject before the object 'pencil'. 'Hers' is the correct is used for 'quantity'. So, 'lesser' should be replaced
possessive pronoun of 'her' in this case because here by 'less'.
it stands for 'her pencil'.
16. 'Richest' is the correct form to fill in the blank as the
7. c The correct sentence will take either ‘she’ or ‘I’. Read blank requires superlative degree.
it as ‘The moon is as beautiful as she (is)” or “The
moon is as beautiful as I (am)”. 17. The blank will take 'more expensive' to indicate
comparison between the prices of beer and petrol in
8. b 'Your' here means that we appreciate the fact that Goa.
you contacted the office. The use of 'you' would have

VA / Exercise - 6 CEX-5326/P1BS/17 / Page 1


18. 'Latter' is used to refer to the second of two persons 38. The first blank will take the preposition 'for'. It is used
or things that have been mentioned. When more than to say that something is available in exchange for
two have been mentioned, 'last' is used. So, the blank something, which in this case is nothing, as the bottle
will take 'latter'. is free of cost. The second blank will take the
preposition 'with'. It is used in the sense of along with.
19. The blank will take 'nearest' as a superlative form is
required here. It means more near in space. 39. The first blank will take 'off'. Here it is used as a function
word to indicate physical separation from a union.
20. Comparative degree is used in order to compare two
things. The comparative form of 'courage' is 'more 40. The blank will take the preposition 'for'. Here it means
courageous'. because of.

21. Accidently Practice Questions


22. Extremely slowly
1. c Option (a) is incorrect. It suggests that the Gangnam
dance is performed for the purpose of winning. Option
23. Yesterday
(b) is incorrect because it is not clear who does the
dance. Option (d) is meaningless. The correct answer
24. Briefly
is option (c).
25. Quite fast
2. d Options (a) and (c) accuse BCCI directly of being
unfair. Option (b) is meaningless. Option (d) is the
26. The correct form of the word is 'hard'.
correct answer because it does not make a sweeping
charge of unfairness but sees some unfairness in the
27. The correct form of the word is 'badly'.
selection process.
28. The correct form of the word is 'awfully'.
3. d Options (a), (b) and (c) do not make any grammatical
sense. Option (d) is the correct answer.
29. The correct form of the word is 'most beautifully'. We
often use 'more', 'most', 'less' and 'least' to show degree
4. c Option (a) negates the intended meaning options (b)
with adverbs.
and (d) are grammatically in correct. Option (c) is the
correct answer.
30. The correct form of the word is 'absolutely'.
5. c Options (a), (b) and (d) are grammatically incorrect.
31. The correct preposition to be used here is 'with' as it is
Option (c) is the correct answer.
used to say that people or things are together in or at
one place.
6. c Options (a), (b) and (d) are grammatically incorrect.
The correct answer is option (c).
32. 'At' is used to indicate the place where someone or
something is. So, it is the correct preposition to fill in
7. d Option (a) should read ‘you and I ----- our best’. Likewise
the blank.
options (b) and (c) are grammatically incorrect. The
correct answer is option (d).
33. The correct preposition to be used here is 'among'. It is
used when you are dividing or choosing something
8. c Option (a) should have the nominative pronoun I instead
and three or more people or things are involved
of the reflexive ‘myself’. Options (b) and (d) apart
from using the reflexive incorrectly have grammatical
34. 'Into' is used to refer to a position in or inside something.
errors. Option (c) is the correct answer.
35. The correct preposition to be used here is 'like'. As a
9. c Option (a) should read ‘He and I .... our’. Option (b)
preposition, it means similar to or close to something.
should read ‘our assignment’. Likewise option (d) is
Although 'around' is also grammatically correct, here
wrong. Option (c) is the correct answer.
we will avoid using it because it is an adverb and not
a preposition.
10. d In option (a), (b) and (c) the tenses are mixed up.
Option (d) is the correct answer. Past tense is used to
36. The correct preposition to be used here is 'about',
indicate an imaginary situation.
which is used to indicate the subject of something
said or written.
11. ‘Whom’ instead of ‘who’ – Objective not nominative.
37. The correct preposition to fill in the blank is 'without'. It
12. ‘Which’ instead of ‘who’.
means not doing the action mentioned.

CEX-5326/P1BS/17 / Page 2 VA / Exercise - 6


13. ‘That’ instead of ‘who’ – superlative. 35. Adjective – Qualifies noun ‘evidence’.

14. Possessive pronouns do no take apostrophes. Hence, 36. ‘In’


‘Its’ instead of ‘it’s’.
37. ‘In’
15. ‘Yourselves’ instead of ‘yourself’.
38. ‘Among’ – only two then ‘between’. More than two –
16. No error. The dog has a name and hence a gender. among.

17. ‘If I were you’ – subjunctive mood. 39. ‘Of’

18. ‘I have known him’ – Present perfect instead of simple 40. For, one buys something for. But at a given price a
present. purchase is a bargain.

19. ‘Ought to’ not ‘must’ which stands for certainty. ‘Ought 41. ‘Between’. For location based an several points.
to’ is likelihood.
42. ‘For’
20. ‘Hers’ instead of ‘Her’ which is an adjective not
pronoun. 43. ‘Under’ meaning on earth. Everyone knows that the
sun is very old.
21. ‘Is’ not ‘are’. The subject ‘house’ is singular.
44. ‘For’ – ‘Cut out for’ means ‘trained for’.
22. ‘Me’ instead of ‘I’. Object of verb give.
45. ‘From’
23. ‘I myself’ instead of ‘I’ only Emphatic pronoun.
46. ‘Lend money’ to strangers.
24. ‘Chapter five’ not chapter fifth – numeral adjective
after noun is cardinal before noun is ordinal. 47. John loves to play cricket, to sing and to swim. –
Parallelism.
25. Grammatically wrong. ‘I am the principal’.
48. ‘Between you and me’ Object of preposition ‘between’.
26. Adjective – Qualifies brother predicatively. Hence objecitve case.

27. Adverb – Modifies verb ‘living’. 49. ‘Who’ not ‘Whom’ subject of verb in ‘was responsible’.

28. Adverb – Modifies verb ‘came’. 50. ‘Was’ not ‘were’. ‘Every’ is followed by a singular verb.

29. Adverb – Modifies verb ‘Eats’. 51. ‘Is’ not ‘are’. ‘Either’ is followed by a singular verb.

30. Adjective – Qualifies pronoun ‘something’. 52. ‘Who have’ not ‘who has’. Subject is novelists.

31. Adverb – Modifies verb ‘knows’. 53. ‘Who they say’ and ‘Whom my brother knows’

32. Adverb – Modifies verb ‘went’. 54. The same bat that Tendulkar uses.

33. Adjective – Qualifies noun ‘work’. 55. ‘Much greater than that of the Eiffel Tower’.

34. Adverb – Modifies verb ‘given’.

VA / Exercise - 6 CEX-5326/P1BS/17 / Page 3


Verbal Ability – 7 P-1(BS)

Verbs and Tenses


Answers and Explanations
31 d 32 a 33 a 34 c 35 c 36 b 37 c 38 a 39 b 40 b
41 a 42 c 43 b 44 b 45 c

1. Rahul goes to school every day. 15. The correct sentence will be "I went to bed after I had
had my dinner." Here, the first 'had' refers to time (Past
2. Sapna is visiting her family right now. Perfect Tense) and the second 'had' is a substitute of
'eaten' (after I had eaten).
3. I studied/was studying Economics in 1994.
16. Simple Present Tense is used in exclamatory
4. He has spoken/has been speaking French since he sentences beginning with 'here' and 'there' to express
was a child. what is actually taking place in the present. So, the
correct sentence will be "Here comes the much
5. Raj had visited many places before he came here. awaited trailer of the movie!"

6. We saw terrible things back then. 17. The auxilliary verb ‘did’ is in the Past Tense here, so
the sentence would take ‘know’ instead of ‘knew’.
7. Sometimes I still have dreams like I did twenty years
ago. 18. The sentence talks about Mr. Yuda’s decision to speak
out after the murder of his uncle; therefore the
8. Japan had never had democracy until 1945. sentence would take the Simple Past Tense. So the
correct sentence will be “Mr Yuda decided to speak
9. The father will call the family together if he thinks there out after the murder of his uncle, an opposition activist,
is disharmony. two months ago.”

10. When I was young, I never cooked because my parents 19. The tense consistency has to be maintained. The
had two servants. sentence will use Simple Past Tense all throughout.
So, the correct sentence is “Ashish told me that he
found out that Sanjay had a learning disability, but I
11. Future Continuous Tense is used to talk about future
think Ashish already knew that since they grew up
actions which are already planned or which are
together.”
expected to happen in the normal course of things.
So, the correct sentence will be "Joan said that she 20. The Past Perfect Tense has to be used here to show
will be staying at my place for three days." that the announcement was made as a result of the
injury that had been inflicted before lunch-time. The
12. Simple Future Tense is used to talk about what we correct sentence will be “During halftime, the
think or believe will happen in the future. So, the correct announcer told the spectators that a player had been
sentence will be "I believe I will win the competition." injured in the first half and needed a doctor’s aid.”

13. If two actions happened in the past, it becomes 21. We use Future Perfect Continuous Tense to emphasise
necessary to show which action happened earlier. how long something has been going on in a particular
Past Perfect Tense is used in such situations. So, the time in future. The correct sentence will be “By
correct sentence will be "She had left school before I Saturday, we will have been living in this house for a
joined." year.”

22. We use Past Perfect Continuous Tense for an action


14. The correct sentence will be "My mother wants to
that began before a certain point in the past and
speak to you." Verbs of emotion take Simple Present
continued upto that time. The correct sentence will be
Tense.
“ We had been looking at the photograph for about ten
minutes before we realised who the artist was.”

VA / Exercise - 7 CEX-5327/P1BS/17 / Page 1


23. Tense consistency will be maintained by talking about 30. This sentence talks about the person ensuring that
all the actions in the past tense. So, the correct Sanjay does his work correctly. For personal plans or
sentence will be “The clients stopped in front of each predictions we use ‘going to’ with the Simple Present
design, studied each one, and then took notes.” Tense. So, the correct sentence will be “Tonight I am
going to check that Sanjay does his homework
24. The sentence talks about the actions that will be taken correctly.”
in the wake of the request from aid workers. Therefore
the request has to be in the past tense or present 31. d The sentence is correct in its given form.
perfect tense and the actions in the future tense. So,
the correct sentence will be “A request for more help 32. a Present Perfect Continuous Tense is used when we
came from aid workers this morning. We will therefore talk about an action that began at some point in the
be sending more food and will need to see if they also past and is still continuing. So, option (a) is the correct
require medical help.” answer.

25. In this sentence we have a verb in the past tense- 33. a Verbs of emotion are not used in continuous form. So,
‘knew’. Therefore, the auxiliary verb that we use the correct answer is option (a).
should also be in the past tense. So, the correct
sentence will be “Maharishi’s Burmese girlfriend knew 34. c Simple Past Tense will be used here. So, option (c) is
that the society of her country would never accept the correct answer. Option (a) is incorrect because it
their relationship.” says that the action of conducting the meeting will
take place in the future. Since she is a fresher, we
26. Both actions are happening at the same time. If we cannot say right now that she will conduct it with
use ‘had fainted’, we are reporting that she and her aplomb or with confidence.
sister fainted before something incredible happened.
So, the correct sentence will be “Then, something 35. c Option (c) represents the correct way of asking a
incredible happened; she and her sister fainted from question.
the shock.”
36. b 'Mamta' is singular. So, the verb will take Simple Present
27. The given sentence uses a conditional (which is form. So, 'drives' will replace 'drive'.
known as the past unreal) that talks about a situation
in the past which did not happen. In such cases, the 37. c The verb following 'didn't' is always in the Simple
structure of a sentence is as follows: Present Tense. So, 'didn't called back' will be replaced
by 'didn't call back'.
Structure
38. a Future Perfect Tense is used when we talk about
If- clause Main clause
actions that will be completed by a certain future time.
would/should/could/might + So, 'will complete' will be replaced by 'will have
Past perfect tense
have + Past Participle completed'.
Example:
If you had studied you would have got a first 39. b Present Perfect Tense is used to denote an action
hard class beginning at some time in the past and continuing up to
the present moment. So, the underlined phrase will be
So, the sentence will be “If you had called him he replaced by ‘have been’.
would have come.”
40. b Past Perfect Continuous Tense is used for an action
28. Simple Present Tense is used with an infinitive when it that began before a certain point in the past and
refers to an action in the past. So, the correct sentence continued upto that point. So, the underlined phrase
will be “The doctor asked the child to lie on the bed.” will be replaced by ‘had been losing’.

29. The verb ‘doubt’ is rarely used in any of the continuous 41. a The sentence is in past tense. So, option (c) is negated.
The correct answer is option (a). Past Continuous
tenses. So, the correct sentence will be “Since the
Tense is used to denote an action going on at some
manager doubted the subordinate’s leadership abilities
time in the past.
he decided to keep a close watch on the team’s overall
performance.” 42. c Simple Present Tense and not Simple Future Tense is
used in clauses of time and of condition.

CEX-5327/P1BS/17 / Page 2 VA / Exercise - 7


43. b ‘increases’ will replace increase as it refers to the 16. The verb should be ‘have been made’ ‘Major
‘number’ of pilgrims. concessions have been made’.

44. b ‘brings’ will replace ‘will bring’ to correspond to the 17. The verb should be ‘have been excised’. ‘All bawdy
verb ‘comes’. passages have been excised’.

45. c ‘put them’ will replace ‘put it’ as we are talking of a 18. The verb should be ‘sailed’. ‘The clouds sailed lazily’.
plural noun ‘books’.
19. The verb should be ‘has flowed’. ‘Much water has
Practice Questions flowed down the Nile.’

1. The verb should be ‘striven’. ‘Had you striven harder’’. 20. The verb should be ‘has flown’. ‘The parrot has flown
from its cage’.
2. The verb should be ‘would have helped’. ‘He surely
would have helped you’. 21. The verb should be ‘was smitten’. ‘Majnu was smitten
with love’.
3. The verb should be ‘ridden’. ‘I have ridden past’.
22. The verb should be ‘had hit’. ‘Virender Sehwag had hit
4. The verb should be ‘were’ ‘If I were your husband I twenty four boundaries’.
would surely drink the poison’. The sentence is in the
subjunctive mood. It describes an imaginary situation. 23. The verb should be ‘parlay’. ‘Will you parlay on our
behalf’.
5. The verb should be ‘was drunk’. ‘He fell out of the car
because he was drunk’. 24. The verb should be ‘were monitored’. ‘The elections
were monitored by the CEC;s office’.
6. The verb should be ‘had cut’. ‘John had cut the timber
into ‘planks’. 25. The verb should be ‘had been sprung’. ‘By the time we
arrived many surprises had been sprung’.
7. The verb should be ‘is bound’. ‘He is bound to return
your money’. 26. The verb should be ‘were spotted’. ‘As the reindeer
raced across the ice they were spotted’
8. The verb should be ‘is bounded’. ‘The football field is
bounded on all four sides’. 27. The verb should be ‘tossing’. ‘Tossing their heads in
sprightly dance’.
9. The verb should be ‘smelt’. ‘I smelt a rat’.
28. The verb should be ‘cleaved’. ‘Samson had cleaved
10. The verb should be ‘will have divined’. ‘You all will Delilah..’.
have divined by now’.
29. The verb should be ‘had majored’. ‘The young man had
11. The verb should be ‘was meant’ or ‘is meant’. ‘The majored in Physics’.
book which is a primer was meant (or is meant) for
your younger brother’. 30. The verb should be ‘had lambasted’. ‘Dr Samuel
Johnson had lambasted Alexander Pope’.
12. The verb should be ‘had leant’ or ‘had leaned’. ‘Because
he had leaned forward too much’. 31. ‘If I were you I would never do this’.

13. The verb should be ‘had been going’. ‘Matters had 32. ‘He had had a plateful of meat’.
been going smoothly between’.
33. John has been having recurrent bouts of laryngitis
14. The verb should be ‘had’. ‘Had I been present there’. since two thousand and thirteen’.
Imaginary situation. Subjunctive mood.
34. ‘Your suit will have been stitched before your arrival’.
15. The verb should be ‘have foundered’. ‘Many ships
have foundered’. 35. ‘As the international price of crude oil has dropped
sharply the price of petrol in India ought to fall’.

VA / Exercise - 7 CEX-5327/P1BS/17 / Page 3


36. ‘Having had a whole chicken for breakfast he did not 48. When you went to Mumbai, Ramesh was the one
feel hungry enough for lunch’. person who missed you.

37. ‘The landlord prayed that the tenant be turned out 49. As they had ridden roughshod over the people when
immediately’. they were victorious, so when defeated they were
killed mercilessly.
38. ‘We can discuss the problem untill the cows came
home’. 50. A flock of geese flew across the sky.

39. ‘He arrived just as I finished eating’. 51. He has finished eating and can accompany you now.

40. ‘He will have done his homework before you return’. 52. Rustum has slain 500 men in battle.

41. If he does not do all this, all his efforts will come to 53. John had stolen the gold medal from the museum.
nought.
54. We eat parathas everyday.
42. The problem has already been analysed threadbare
by the experts. 55. He seems to be afraid.

43. Do not open the door, as it is he, the salesman again. 56. This lunch box consists of three separate boxes.

44. The train rolled over the side of the bridge and fell into 57. He has (or he owns) lots of real estate in Delhi.
the river.
58. I perceive that your knowledge of grammar is
45. Not having received his mother’s affection in childhood insufficient.
he became violent on the smallest pretext.
59. I have known him since he was a child.
46. The guard had been gone for so long that we started
wondering what had become of him. 60. I see that you have lost weight.

47. The baby was so scared it clung to its mother.

CEX-5327/P1BS/17 / Page 4 VA / Exercise - 7


Verbal Ability – 8 P-1(BS)

Subject Verb Agreement


Answers and Explanations

1 b 2 b 3 b 4 d 5 b

Practice Questions
1 b 2 a 3 b 4 a 5 a 6 b 7 b 8 c 9 b 10 c

1. b The correct sentence should be "The group of students 13. Here the verb acts on the singular subject 'cabin', and
from that college has never been to the zoo." 'Group' is therefore should also be singular. So, the appropriate
a singular entity and should be followed by a singular word is 'needs'.
verb.
14. Words joined to a singular subject by 'along with', 'as
2. b Verbs that follow subjects of the form either A or B and
well as', etc. are parenthetical. The verb should therefore
neither A nor B must agree with B, the noun closer to the
be singular. So, the appropriate word is 'heads'.
verb. Since ‘I’ is singular, ‘am’ should replace ‘are’.
15. 'Major part' is singular. So, the appropriate word is 'is'.
3. b The correct phrase is "…slouches because of his back
pain." We are talking about the singular noun 'brother'
16. 'Each' is a singular subject and is followed by a singular
and not 'others'. So, the verb should be singular.
verb. So, the appropriate word is 'is'.
4. d The sentence is correct in its given form.
17. 'Three-fourths' is plural. So, the appropriate word is
'have'.
5. b The correct phrase is "One-fifth of the locality is
experiencing..." When a plural noun denotes some
18. 'Voters' is the plural subject. Therefore, it will be followed
specific quantity or amount considered as a whole, the
by a plural verb. So, the appropriate word is 'are'.
verb is singular.
19. Singular verb is used with sums of money or periods of
6. Here the verb acts on the plural 'employees', and
time. So, the appropriate word is 'is'.
therefore should be plural. So, the inappropriate word
is 'is'.
20. Pronouns such as 'each', 'every one', 'everybody', etc.
are singular and require singular verbs. So, the
7. Here the verb acts on the plural 'reports', and therefore
appropriate word is 'is'.
should be plural. So, the inappropriate word is 'is'.
21. 'Guys' is a plural subject that should be followed by a
8. Here the verb acts on the plural 'profits', and therefore
plural verb. So, the appropriate word is 'are'.
should be plural. So, the inappropriate word is 'is'.
22. 'Many a' must always be followed by a singular verb.
9. When 'none' is followed by a mass noun (a noun that
So, the appropriate word is 'has'.
cannot be counted or made plural) it takes a singular
verb. So, the inappropriate word is 'are'.
23. The sentence is talking about a single person who is a
writer as well as a singer. So, the appropriate word is
10. Here the verb acts on the plural subject 'cushions', and
'has'.
therefore should also be plural. So, the inappropriate
word is 'was'.
24. As a general rule, we use a plural verb with two or
more subjects when they are connected by ‘and’. So,
11. Here the verb 'come' acts on the plural 'friends', and
the appropriate word is 'are'.
therefore should also be plural. So, the appropriate word
is 'come'.
25. ‘The United States’ is a proper name for a single country
and should therefore be followed by a singular verb.
12. Here 'cracks' are plural. Therefore the verb should also
So, the appropriate word is 'has'.
be plural. So, the appropriate phrase is 'have gone'.

VA / Exercise - 8 CEX-5328/P1BS/17 / Page 1


26. The pronoun 'no one' is singular and should be followed 15. 'are' With uncountable nouns 'none' is always singular.
by a singular verb. So, the appropriate word is 'is'. With countable nouns one may treat none as singular
or plural the latter being more common.
27. Pronouns such as 'each', 'everyone', 'anyone', etc. are
singular and require singular verbs. So, the appropriate 16. 'is' is appropriate.
word is 'is'. 17. 'is' subject is 'she' not 'novelists'.
28. 'Neither', 'either', 'each', etc. are followed by a singular 18. ‘are’ police – a collective noun – always plural.
verb. So, the appropriate word is 'is'.
19. ‘are’. Jury refers to the Jury members.
29. As a general rule, we use a plural verb with two or
more subjects when they are connected by 'and'. So, 20. ‘is’, ‘many is’ is treated as singular in formal english.
the appropriate word is 'are'.
21. ‘is’, ‘mathematics’ is singular and so are subject names
30. 'Goals' is a plural subject. So, the appropriate word is ending in S such as Physics, statics, statistics,
'are'. dynamics etc.

22. ‘is’ news is always singular.


Practice Questions
23. ‘is’ rickets is singular. So are the names of other
1. b 'must do your best' when a pronoun is used for the diseases ending in ‘s’ such as thrombosis, piles,
second and third person it must be of the second arthritis etc.
person.
24. ‘is’ with neither – nor a singular verb is used.
2. a 'It is he'. The real subject of the sentence is he. It is a
temporary subject. Hence HE and not HIM. 25. ‘is’ atlas is singular.

26. ‘are’ two singular nouns joined by ‘and’ are treated as


3. b 'has to do his or her (or their) duty'. With each only a plural.
singular verb is used.
27. ‘is’ when two nouns refer to a single idea they are
4. a 'Yudhishthira, Bhima, Arjuna, Nakula and Sahadeva's' treated as singular e.g. ‘bread and butter’, ‘dal and roti’
When something is owned jointly by several people etc.
the apostrophe and s are attached to the last named
owner. 28. ‘is’ three hundred runs constitute a single objective
here. Hence, singular e.g. ‘500 miles is a long distance’,
5. a 'My brother, the artist's '.When two nouns are in Rs. fifty thousand is a large sum etc.
apposition the possessive is formed from the second
noun. 29. ‘are’ It is not a single target.

30. ‘are’ is appropriate


6. b 'I like the last but three(or third) on the right.' Fourth last
is not English. The correct English expression is last
31. ‘flock’ birds is plural.
but three.
32. ‘is’ herd is a collective noun, hence singular.
7. b 'is going to attend'. The subject is only 'My brother' .
Along with does not form part of the subject.
33. ‘is’ name of a book. Hence, singular.
8. c 'have eaten their dinner'. See explanation from Q67.
34. ‘are’ is appropriate.
9. b 'Live in large flocks'. Deer is plural.
35. ‘is’ is appropriate.
10. c 'have been rewarded'. Players is plural.
36. ‘is’ is appropriate
11. 'is' The noun earth is singular.
37. ‘are’ refers to advertisements.
12. 'is' Many is plural but many a is singular.
38. ‘are’ is appropriate.
13. 'are' is appropriate.
39. ‘were’ is appropriate.
14. 'are', 'Few men',' a few men', 'the few men' are all
40. ‘has’ is appropriate.
plural.

CEX-5328/P1BS/17 / Page 2 VA / Exercise - 8


Verbal Ability – 9 P-1(BS)

Grammar Revision Test – 1


Answers and Explanations
1 d 2 c 3 c 4 a 5 c 36 b 37 a 38 b 39 c 40 a
41 b 42 b 43 c 44 d 45 a 46 c 47 a 48 a 49 a 50 a
51 d 52 a 53 a 54 c 55 b 56 b 57 a 58 d 59 c 60 c
61 c 62 d 63 b 64 b 65 a
Practice Questions
1 b 2 c 3 d 4 c 5 d 6 a 7 c 8 c 9 c 10 a
41 c 42 c 43 b 44 d 45 c 46 b 47 b 48 c 49 b 50 c

1. d The sentence is correct in its given form. Adjectives 11. 'Beard' here acts as a verb. It represents an action of
usually follow the following order: confronting and opposing with boldness.
general opinion - specific opinion - size - shape - age
- colour - nationality - material 12. 'Very' is an adverb which modifies the adjective 'poor'.

2. c 'Lest' is a conjunction and is used when you are saying 13. 'Good' is an adjective which describes the noun
something in order to prevent something from 'news'.
happening. Here, the required adjective is 'least', which
means smallest in amount or degree. 14. 'However' acts as a conjunction.

3. c The comparative ‘er’ is not used when we compare 15. 'Eccentric' is an adjective that describes 'British life'.
two qualities in the same person or thing. So, ‘braver’
should be replaced by ‘more brave’. 16. 'Channels' is a noun.

4. a 'A little' has a positive meaning. 'Not quite reliable' 17. 'Around' is an adverb that is dependent on the verb
suggests that the sentence will not take 'a little' but 'sit'.
'the little', which means negligible.
18. 'Often' modifies the main verb 'feel'. Hence, it is an
5. c Words such as 'senior', ‘superior’, ‘perior’, etc. are adverb.
followed by ‘to’ instead of ‘than’. So, ‘than’ in part (c)
will be replace by ‘to’. 19. 'Profligate' is an adjective. It means carelessly and
foolishly wasting money, materials, etc.
6. 'Elude' is a verb. It means to manage to avoid or escape
from somebody/something. 20. 'Rescind' means to end (a law, contract, agreement,
etc.) officially. It is a verb that expresses the action of
7. 'Suspect' as an adjective means something that may ending a stipulation.
be false and that cannot be relied on. Its synonym is
questionable. 21. In English, the subject of a command, order, or
suggestion - 'you', the person being directed - is usually
8. 'Comely' is an adjective. It means attractive or pleasant left out of the sentence and is said to be the understood
to look at. subject. So, 'you' is the subject here.

22. 'Lives' is the verb. It acts on 'friend'. So, 'friend' is the


9. In the given sentence 'behind' acts as an adverb. ‘Fallen subject of the sentence.
behind’ means late in paying money or completing work.
23. The doer of the action (or the one getting affected by
10. Here 'bar' refers to a place. Thus, it is a noun. smog-filled air in this case) is 'we'. So, 'we' is the
subject.

VA / Exercise - 9 CEX-5329/P1BS/17 / Page 1


24. 'Taking dangerous risks' is what seems to frighten 41. b Definite article 'the' is used before superlative
most hardworking people. So, 'Taking dangerous risks' adjectives.
is the subject of the sentence.
42. b 'Cloth' is a fabric formed by weaving, felting, or knitting
25. The sentence talks about 'mind' that can be full of wool, cotton, etc. 'Clothes' are articles of dresses or
many dangerous things. So, 'mind' is the subject of the garments. The sentence means that his garments were
sentence. torn. So, 'cloths' will be replaced by 'clothes'.

26. The verbs in the sentence are ‘echoed’ and 'grabbed'. 43. c 'Two' in part (c) suggests that the plural of 'ox' should
So, ‘name’ and 'girl in the wheelchair' are the subjects. be used. So, 'ox' should be replaced by 'oxen'.

27. 'Jumped' is the verb. So, the subject is 'tall girl' 44. d The sentence is correct in its given form.

28. The verb is 'dedicated'. So, the subject becomes 'Taj 45. a Words joined to a singular subject by 'as well as',
Mahal'. 'with', etc. are parenthetical. The verb should therefore
be put in singular. So, 'want' should be replaced by
29. 'Developed' is the verb. So, the subject is 'villages'. 'wants'.

30. 'Cause' is the verb. So, 'rumors' is the subject. 46. c 'So far' indicates time from the start of something up to
the present moment. So, 'may had remained' in part (c)
31. 'at the banquet hall' is a prepositional phrase while ‘the should be replaced by 'may have remained'.
dance was held’ is a clause.
47. a 'Committee' can be used both as singular and as plural.
32. 'While she sang' is a dependent clause and 'the If we refer to the members of the committee, we use
audience could not take its eye off her' is an a plural verb with it. If we refer to the committee as a
independent clause. 'off her' is a prepositional phrase whole, we use a singular verb with it. In the sentence,
that is nestled within the independent clause. 'committee' is not referred to as a whole; it refers to
the members of the committee who are having
33. 'I had sweet curd' is an independent clause while sandwiches. So, 'is' in part (a) will be replaced by
'before I left for the interview' is a dependent clause. 'are'.

34. 'I go to the temple' is a clause while 'every morning' is 48. a Definite article 'the' is used before the names of certain
a phrase. books such as 'the Vedas', 'the Ramayana', etc.

35. 'After working late into the night' is a dependent clause 49. a When one noun is qualified by two possessive nouns
while 'Jack fell asleep on his desk' is an independent both must have the possessive sign, unless joint
clause. possession is indicated. So, 'king's' in part (a) should
not have the possessive sign. So, the correct phrase
36. b Names of instruments which have two parts forming is 'The King and Queen's stay'.
a kind of pair are always used in plural. So, 'pair of
tong' will be replaced by 'pair of tongs'. 50. a Simple past tense is used to indicate actions that
completed in the past. In the sentence, 'ever since'
37. a 'Now couldn't remember' suggests that the 'household suggests that the action of playing guitar is still
tasks' have been done in the past. So, 'will be doing' continuing. So, present perfect tense should be used
will be replaced by 'had been doing'. instead of simple past tense. So, 'I'd played' will be
replaced by 'I've played'.
38. b Definite article 'the' is used before the names of musical
instruments. 51. d The sentence is correct in its given form. If you want
to refer to the specific quantity of options, then the
39. c 'Force' means strength and 'forces' means troops. definite article 'the' is used.
Here, the sentence is talking about troops, so, 'force'
will be replaced by 'forces'. 52. a When a plural noun denotes some specific quantity or
amount considered as a whole, the verb is generally
40. a 'Dozens' is incorrect. It should be replaced by 'dozen'. singular. So, '30 minutes are' should be replaced by
'30 minutes is'.

CEX-5329/P1BS/17 / Page 2 VA / Exercise - 9


53. a Verbs of thinking such as 'think', 'suppose', 'believe', 67. The correct pronoun to replace the phrase is 'their'.
'agree', etc. are not used in continuous form. So,
'thinking' should be replaced by 'think'. Names of 68. The phrase will be replaced by 'it'.
mountain ranges take the definite article 'the' before
them. However, names of summits do not require the 69. The correct pronoun to replace the phrase is 'them'.
definite article 'the' before them.
70. The correct pronoun to replace the phrase is 'he'.
54. c Groups of islands such as 'the West Indies' take the
definite article 'the' before them. Practice Questions
55. b 'Number' is singular. The verb should modify 'number'. 1. b The correct word is ‘customary’ i.e. in accordance
So, 'are' in part (b) should be replaced by 'is'. with established custom.
‘Accustomed’ means what someone is used to.
56. b 'Yet', which is used to introduce a statement that adds
something to a previous statement and usually 2. c The correct words are ‘were called’ because there
contrasts with it in some way, will fill in the blank. were two bombs

57. a The blank will take 'outside', which means apart from. 3. d There is no error in this sentence.

58. d 'Phew' is a sound that people make to show that they 4. c The correct phrase is ‘ran across’ meaning met
are hot, tired, or happy that something bad has finished accidentally.
or did not happen.
5. d There is no error in the statement
59. c 'Jeepers' is an interjection that is used to express
surprise. 'Bah' is used to show dislike or disapproval 6. a ‘Many a student’ is singular and therefore the verb
of something. 'Shoot' is used for expressing anger or should be ‘has’
disappointment.
7. c The correct expression is ‘The picture of health’
60. c 'Considering' fits in the blank as it means taking into
consideration. 8. c The correct expression is ‘Bring up to speed’ meaning
bring people abreast of the latest developments.
61. c 'Or' is used to suggest a negative alternative.
9. c The correct expression is ‘upto scratch’ which means
62. d 'During' is used to say when something happens; 'for' upto the required standard.
answers the question 'how long?'. So, the blank will
take 'for'. 10. a The correct expression is ‘cut and thrust’ meaning
sharp exchanges.
63. b 'Despite' is used to show that something happened or
is true although something else might have happened 11. Preposition
to prevent it. So, the blank will take 'despite'. 'In spite' is
followed by 'of' and therefore is incorrect. Similarly, 12. Noun
'regardless' is followed by 'of'. Without the 'of', it acts
as an adverb. 'Owing to' means because of. It will 13. Preposition
render the sentence meaningless.
14. Noun
64. b 'Ugh' is used to show that you are annoyed, disgusted,
or upset about something. 'Whew' is a whistling sound 15. Conjunction
or a sound like a half-formed whistle uttered as an
exclamation. 16. Preposition

65. a 'By' is used to show the degree or amount of something. 17. Adverb
'Past' as a preposition means later than something.
18. Adverb
66. Nephew has to be a boy. So, the phrase will be
replaced by 'him'.

VA / Exercise - 9 CEX-5329/P1BS/17 / Page 3


19. Adjective 36. ‘Helped’ is the verb. Hence, ‘Byes’ is the subject.

20. Noun 37. ‘IS Becoming’ is the verb. Hence, ‘Recidivism’ is the
subject.
21. Adjective
38. ‘Was’ is the verb. Hence, ‘Ambition’ is the subject.
22. Adjective
39. ‘Rallies’ is the verb. Hence, ‘voice’ is the subject.
23. Interjection
40. ‘Must Tumble’ is the verb. Hence, ‘Sceptre and Crown’
24. Noun is the subject.

25. Verb 41. c Pronoun ‘Hers’ is the right answer.

26. Conjunction 42. c Interjection ‘Hey’ is the right answer. The other three
options do not go with the exclamation mark.
27. Noun
43. b ‘Nether’ meaning lower is the right answer, Nether
28. Adverb world means Hell.

29. Adjective 44. d ‘Besides’ meaning ‘in addition to’ is the right answer.

30. Pronoun 45. c ‘Large’ is the right answer

31. The verb is SAT and therefore ‘general’ is the subject. 46. b ‘Detritus’ meaning natural waste from a process’ is
the right answer.
32. The verb is ‘IS’ and hence the subject is ‘Purpose’.
47. b ‘Put out’ means disheartened.
33. The verb is ‘WAS PRODUCED’ and hence the subject
is ‘Knife’. 48. c ‘Turns out’ means ‘produces’

34. ‘Have’ is the verb, Hence ‘Prayers’ is the subject. 49. b ‘Throw down the gauntlet’ means to challenge some
one.
35. ‘Had’ is the verb. Hence, ‘Craftsmen’ is the subject.
50. c ‘Ran up’ means accumulated.

CEX-5329/P1BS/17 / Page 4 VA / Exercise - 9


Verbal Ability – 10 P-1(BS)

Modifiers and Parallelism


Answers and Explanations

11 c 12 a 13 d 14 c 15 c 16 c 17 d 18 a 19 c 20 d
36 d 37 a 38 c 39 c 40 c

Practice Questions
19 d 20 a 28 b 29 a 30 b 31 a 32 b 33 b 34 d 35 c
36 b 37 b 38 d 39 a 40 c

1. The correct sentence is "While I was walking back second part of the sentence it appears that Ginny is
from work, it started raining." In the first part of the fading. To make the meaning clear, we place the subject
sentence, an action is being performed. However, the next to the modifier "fading so slowly" by repeating
subject performing the action is not stated. This can the entailing word from the first part of the sentence.
result in the wrong assumption that the performer of So, the correct sentence is "Ginny looked towards
the action is "it". To correct this grammatical error, we the horizon - a horizon fading so slowly that it made
place the performer with the action being performed. her wonder."
In this case we modify the first part of the sentence to
include the actor (I/we). 6. This sentence is trying to give the information that the
subject has slept for only three hours on a particular
2. This sentence is talking about a book that was written
night. However, placing the modifier next to the verb
by "her" and which the subject "I" have in my bag. In
"slept" alters the meaning of the sentence and indicates
the given statement, this can be misunderstood to
that the subject didn't sleep at all. To correct this
mean that the book being talked about has been written
grammatical error, we place the modifier next to the
inside the bag. To avoid this confusion, we place the
modifier near the word it modifies. So, the correct term it actually modifies, which in this case is "three
sentence is "In my bag, I have the book that she wrote." hours". So, the correct sentence is "We slept for barely
three hours that night."
3. In the sentence, the modifying phrase "in order to
prevent sunburns" is placed in the middle of the infinitive 7. There are two actions being performed in the sentence.
verb "to block". This is called a split infinitive. Even However, neither the actor nor the second action has
though this arrangement doesn't alter the meaning of been mentioned clearly. In order to make the sentence
the sentence, it makes it confusing. In order to avoid clear, we include the performer of the action in both
this grammatical inconsistency, we generally avoid parts of the sentence. So, the correct sentence is
placing modifiers between "to" and the verb of the "After I read the specifications, I realized that there
infinitive. So, the correct sentence is "In order to were many faults in your design."
prevent sunburns, he lifted the umbrella over his head
to block the sun." 8. In order to correct the sentence, we will place a
subject after the modifier while keeping the meaning
4. In this sentence, it is unclear whether the modifier of the sentence intact. So, the correct sentence will
"quickly" pertains to "retrospect" or "improves". Such be “Having been friends with Pip since childhood, I
an anomaly is called a squinting modifier. In order to wasn’t bothered much by our fight.”
state this sentence clearly, we place the modifier just
before or just after the word we want it to modify. In 9. The sentence talks about a chandelier that was over
this case there are two possible correct answers, a hundred years old and had started to creak. The
each with a different word being modified. So, the
modifier in the sentence ("which was over a hundred
correct sentence can either be "Taking a moment to
years old") has been incorrectly placed at the end of
quickly retrospect improves your memory." or "Taking
the sentence. To avoid making this mistake, we place
a moment to retrospect improves your memory quickly."
the modifier next to the word it modifies (i.e. chandelier).
5. This sentence is talking about the witnessing of a So, the correct sentence will be “The chandelier, which
horizon that is fading away slowly. However, in the was over a hundred years old, had started to creak.”

VA / Exercise - 10 CEX-5330/P1BS/17 / Page 1


10. In this sentence, a person has watched a movie based can't be tired. Instead, it is more likely that Shaurya
on another person's life and has also read his journal, himself is tired. Option (d) correctly places the subject
based on which he finds the movie to be unnecessarily next to the modifier and hence, is the answer.
bloated. However, the structure of the sentence makes
it seem like the movie has gone through the person's 21. In the sentence, we are not comparing IQ with Sanchit;
journal. The correct sentence, which clearly states we are comparing the IQ of Rahul with the IQ of Sanchit.
the subsect of the modifier is “After going through his So, the correct sentence is "Rahul's IQ is greater than
journal, I find the movie on his life unnecessarily Sanchit's." Or “Rahul's IQ is greater than that of
bloated.” Sanchit.”
11. c The first part of the sentence talks about an action
22. The correct sentence is "A low-fat diet, most experts
being performed. The structure of the sentence makes
agree, is the best way to reduce artery blockage and
it seem as if the modifier pertains to "the book" rather
achieve weight loss." The writer describes the second
than the person performing the action. To correct this,
benefit of a low-fat diet as "the achievement of weight
we choose the option that includes the actor (she).
loss"-a noun phrase. Clearly, following the first
12. a Option (b) has a dangling modifier without a clearly infinitival phrase with another in the form of "to reduce
stated subject. Option (c) can be misunderstood to artery blockage and to achieve weight loss" improves
mean that the subject has had a divorce with his/her clarity.
therapist. Option (d) is grammatically incorrect. Option
(a) has the clearest meaning among the options given. 23. If we break the sentence, then "…its natural to be
sweaty palms" is clearly incorrect. The correct
13. d Options (a), (b), and (c) have modifiers that are
sentence will be "When one takes the SSC exam, it's
incorrectly placed. Options (a) and (b) suggest that
perfectly natural to be a little nervous, irritable and
her slippers were rushing down the escalators. Option
have sweaty palms.
(c) suggests that the escalators were rushing down.
Option (d) correctly places the modifier and suggests
24. The correct sentence is "Eating huge meals, snacking
that it were her slippers that got caught in the moving
between meals, and exercising too little can lead to
escalators.
obesity."'Eating', and 'snacking' are gerunds. So,
14. c Options (a) and (b) suggest that the file is 'exercise' should be changed to 'exercising'.
"accidentally". Option (d) suggests that the wife is
"accidently". Option (c) correctly places the modifier 25. The correct sentence is "Mustaine likes people who
"accidently" before to the verb "threw". have integrity and character." The verb 'have' should
modify both the nouns - integrity and character.
15. c Option (c) is the correct answer as it correctly
describes that I was passing by the mall when I met 26. The correct sentence is "I like editing books more than
Saurabh. Option (a) is wrong because we are not just reading them." Here, an infinitive is paired with a
sure who met Saurabh. Option (b) is wrong because gerund. However, a gerund should be paired with a
we are not sure Saurabh met whom. Option (d) is gerund only.
incorrect because it is not clear who among Saurabh
and I was passing by the mall. 27. The correct sentence is "Career Launcher needs
16. c Statement (a) suggests that the waitress is sauced teachers who are ambitious, self-motivated, and
with lumpy gravy, which is not logical. In order to dedicated." Here, a verb form has been given with
correct this, we will place the modifier after “grey one adjective and one noun. Parallelism can be
meatloaf”. achieved by converting all three words into adjectives.

17. d Option (d) is correct as it correctly places the modifier 28. For the parallelism to be there, all three activities should
“in torn clothes” next to “the man”. be mentioned as verbs. So, the correct sentence is
"As an artist, he drew, painted, and sculpted."
18. a It is a case of split infinitive. No modifier should be
placed between the 'to' and the verb. To correct a split 29. The correct sentence is "Aggression and melancholy
infinitive error, move the modifier to the front of the are behaviors that many steroid-users exhibit." In this
infinitive. So, option (a) is the correct answer. sentence, both the subjects of the sentence should
be nouns.
19. c To solve a misplaced modifier error, move the modifier
to place it next to the word it modifies. So, 'Because of 30. The sentence has two gerunds and an infinitive form.
his many years of experience' should be placed just To make it a parallel structure, we will change the
before 'Surya was' to make the sentence meaningful. infinitive form to a gerund. So, the correct sentence is
"There's nothing I like better than finding a fresh water
20. d Here, Shaurya's delight is being modified by the phrase lake, setting up camp and spending the weekend
'tired of taking so many medicines'. 'Shaurya's delight' fishing."

CEX-5330/P1BS/17 / Page 2 VA / Exercise - 10


31. The sentence should read as ‘She loves tasks that 7. The correct sentence is ‘The irate student wanted to
are challenging, motivating and handomsely paying.’ speak to the centre manager, to withdraw the
admission, or he wanted to get unlimited teaching
32. The correct sentence is ‘I am looking forward to
sessions.’
meeting you and hearing about your trip.’
8. The correct sentence is ‘Her children are well behaved,
33. The correct sentence is ‘Jane likes to ski, run, skate
her home is immaculate and she’s a good cook.’
and swim.’ (or ‘Jane likes to ski, to run, to skate and to
swim.’) 9. This new product offers ease of operation, economy
34. The correct sentence is ‘The earthquake wrecked not and ready availability.
only the railway lines but also the trains.’
10. The manual gives instructions for operating the
35. The correct sentence is ‘The lecturer told the students machine and adjusting it.
that they should study a lot, get enough sleep and do
some relaxation exercises before the exam.’ 11. We are wholesellers of clothing for men, women,
teenagers and children.
36. d There is no error in the statement. The first part of the
sentence before hyphen (-) is the main clause and 12. You can take either the red one or the blue one.
rest are the 'elements' (subordinate clauses) that the
main clause includes. All of them are parallel- 'balance', 13. She is not only quite rich but also quite stingy.
'age' and 'wearing'. 'Wearing' is correct because it is a
noun (gerund) here and not a verb. 14. The book reflects a great deal of research, which is
all too rare.
37. a Following the principles of parallelism, 'supplying'
should be replaced with 'to supply' as 'to keep' is an
15. This is a service that is of considerable value to
infinitive while 'supplying' is a gerund.
researchers and that many supervisors of research
38. c According to the rules of parallelism, a string of nouns departments have used.
should have all singulars or all plurals. Since we have
'whispers', 'tingles' and 'shocks'; 'shout' should also 16. When you concentrate on a job, you find that time
be in plural. So, ‘shout’ will be replaced by ‘shouts’. passes rapidly or When one concentrates on a job,
one finds that time passes rapidly.( there should be
39. c Barking dogs is not a sound. So, ‘barking’ should be
parallelism, consistency in pronouns)
followed by ‘of’.
40. c 'Making' is a verb. So, 'to sell' should be replaced by 17. When reading the paper, I enjoy looking at the models
'selling'. in Delhi Times, checking up with the movies, and
skipping the rest.
Practice Questions
1. The corrected sentence is ‘I will go to the library 18. I was thrilled- I bought a ticket to the Ricky martin
tomorrow, return the book that is five days overdue, concert, shopped for a new pair of jeans, and read
and apologize for holding it up.’ the latest book on Clinton’s escapades in office.

2. The corrected sentence is ‘Espinoza’s style was 19. d Option (a), (b) and (c) contain errors. The three
remarkable for its dexterity, grace, and effusiveness.’ requirements - ‘consultant’, ‘office assistant’ and ‘office
boy’ – all have to be placed in the same form. Option
3. The corrected sentence is ‘Her conditions for signing (d) is correct.
the contract are, first, that the commissioner must
approve the terms; second, that the other players 20. a The given sentence is correct. The -ing form of the
must sign as well.’ verb is used because the subject is still in the process
of learning / teaching these things. Also option (d)
4. The correct sentence should read as ‘Our
suggests that the subject involved herself in all these
transportation crisis can be solved by a bigger plane
activities one by one, which is not the case. Options
or a wider road.’ In comparison, parallel structure should
(c) and (d) can be eliminated as in (c) a forced action
be followed. is implied which is incorrect and in (d) there is tense
5. Correlative conjunction should be placed immediately inconsistency. Hence, (a) is correct.
before the parallel terms. The correct sentence will be
‘Tom and Kiara have attended both the training sessions 21. The error in this sentence is that of a modifier. The
and the classes.’ dinner was not watching the trilogy. So, a subject
needs to be introduced in the sentence. The correct
6. ‘To’ is redundant in this sentence. The correct sentence sentence should be stated as ‘Watching the trilogy
is “Bungee jumping is more adventurous than water back to back on DVD through the evening, he/she
skiing.’ burnt the dinner.’

VA / Exercise - 10 CEX-5330/P1BS/17 / Page 3


22. It appears from the given sentence that ‘a few loose 31. a The given sentence is correct because it contains the
papers’ were returning from the meeting. Here the subject and places it correctly. It was the group that
subject is missing from the first part of the sentence. was walking through the jungle and then decided to
The earlier part needs to be modified. The subject change the course. Option (b) is incomplete since it
should be introduced in the first part. So the sentence does not tell who decided to change the course.
should be- ‘As I was returning from the meeting, a Options (c) and (d) do not contain a subject. So, option
few…attention.’ (a) is correct answer.

23. The modifier phrase “which had occurred…” is to be 32. b Options (a) and (c) have modifier errors. Option (d)
placed immediately after the sprain because it gives distorts the meaning of the sentence. The correct
additional information about the sprain. The correct answers is option (b).
sentence should be: ‘The doctor mentioned that the
sprain which had occurred one week earlier, is 33. b Options (a), (c) and (d) all have modifier errors. The
responsible for the injury to his voice box.’ first part of the sentence needs a subject. Only option
(b) presents a grammatically correct sentence.
24. The modifier ‘with green stripes’ should be placed
next to the dress. The correct sentence should read 34. d Only option (d) presents a grammatically correct
as ‘The woman bought a dress with green stripes at sentence.
the store.’
35. c The given sentence presents a case of dangling
25. ‘....a funnel shaped cloud’ is incorrect as this statement modifier. After the comma, we need the subject - 'last
misleads the reader to think that the funnel was looking competitor' and not 'her ski-tip'. The ski-tip did not
towards the south. Introduction of the subject is somersault into the snow, the last competitor did. The
important, as it is not clear who was looking towards correct sentence should be, “Trying to keep her
the south. It should read as, ‘Looking towards the balance on the icy surface, the last competitor caught
south, I saw a funnel shaped cloud stir up the dust.’ the pole with the tip of her ski, and somersaulted into
the soft snow.”
26. ‘Nearly watched’ misleads the reader. The correct
sentence should read as ‘Mary and John watched 36. b The correct sentence should have 'sweets on the
nearly two hundred movies last year.’ paper plates' next to 'poor'. So, option (b) is correct.
Option (a) suggests that the poor were on the paper
27. Since Greg Chappell is one of the mentors, he should plates. Option (c) suggests that sweets were poor.
be placed after ‘his many mentors’. This is a case of Option (d) appears as if poor is an adjective for sweets.
misplaced modifier. ‘His many mentors among whom
Greg Chappell also figures, provided....weakness.’ 37. b Option (b) correctly places the modifier 'piled up next
to the washer' next to 'laundry'.
28. b Option (b) places the modifier correctly. It is the dentist
38. d With an ear-to-ear grin, a prepositional phrase, should
who removed the braces, so ‘the dentist’ should be describe Mr. Patil. To fix the problem, the sentence
placed next to ‘the braces’. Option (c) is incorrect should be written as: "With an ear-to-ear grin, Mr. Patil
because ‘in spite of’ has been used which tries to returned the perfect calculus test, which delighted
define a condition. Option (d) does not contain ‘the Sanchi."
dentist’ who is giving the advice to Kamal, so it is
incomplete. 39. a The phrase 'sitting on the window sill' should come
right before the subject 'little Stuart'. So, option (a) is
29. a Option (b) is incorrect because the additional phrase correct. Option (d) is incorrect because it suggests
requires commas. Option (c) and (d) do not reveal the that Snowbell is the subject and is sitting on the window
correct meaning of the original sentence. They distort sill, which is not correct since snowbell cannot be
sitting on the window sill and slinking behind the living
the meaning of the sentence by placing the words
room sofa at the same time.
incorrectly. So, the original sentence is correct.
40. c The correct sentence should have 'Shradha' as a
30. b There is a misplaced modifier in the given sentence. standalone subject. In option (a), the subject is
The modifier has been placed correctly in option (b). 'Shradha's cell'. The cell couldn't have been talking on
Options (a) and (d) are incorrect because they seem the phone. Option (b) is incorrect as it suggests that
to suggest that the living room was made by Gia the thief and not Shradhawas talking on the phone.
whereas the meaning of the original sentence reveals Option (d) is incorrect because of the same reason.
that the painting was made by her. Option (c) is Option (c) correctly places the subject before 'talking
incomplete because it does not explain who made the on the phone'.
painting.

CEX-5330/P1BS/17 / Page 4 VA / Exercise - 10


Verbal Ability – 11 P-1(BS)

Punctuation and Error Spotting


Answers and Explanations

21 a 22 a 23 b 24 c 25 b 26 d 27 c 28 d 29 b 30 b
31 b 32 b 33 c 34 a 35 a 36 a 37 b 38 d 39 b 40 b
41 b 42 b 43 b 44 b 45 c 46 a 47 b 48 c 49 c 50 b

Practice Questions
21 a 22 a 23 c 24 c 25 d 26 b 27 c 28 c 29 c 30 c
31 c 32 b 33 b 34 d 35 b 36 c 37 c 38 a 39 a 40 c
41 d 42 a 43 a 44 b 45 d

1. “I need your help now,” she said. “I need it more than 17. The people who live by the water must be prepared
ever.” for occasional flooding.
2. Some praised the performance as excellent and others 18. The boat was seventy-five feet long eighteen feet
thought it was only fair. wide, and it had a mast about eighty feet high.
3. Did she say, “I have enough money?”
19. To anyone interested in flying, planes hold endless
4. “Have you enough money?”, she asked. fascination.

5. What is meant by ‘dog eat dog?’ 20. Jeff and Steve, left alone for the weekend, invited all
their friends to a party.
6. Our play was obviously a ‘bust’!

7. “The play was a ‘bust!”, our coach exclaimed. 21. a The correct phrase should be 'a great time'.

8. “Read E.B. White’s ‘Waiden’: It is, I think, his best essay”. 22. a 'Old children' suggests that the children are old in age.
So, 'old' should be replaced by 'older'.
9. He asked whether I would go.
23. b The correct usage is 'he groomed himself'.
10. Will you send the signed copy as soon as possible?
24. c In part (c), the correct phrase should be 'managed to'
11. While Bill was riding, his bike got a flat tyre. because the first part of the sentence is in the past
tense.
12. The mailman did not leave the package, for Jeff was
not at home. 25. b One does not congratulate someone 'over' something;
one congratulates someone 'on' something. So the
13. After doing her homework, Millie, as you might expect, correct usage is 'him on'. 'Over' means above in place
talked on the phone for an hour. or position.

14. His work criticized many commonly held beliefs, 26. d The sentence is correct in its given form.
however, and it was strictly censored.
27. c Part (c) contains an error. The correct phrase should
15. The car, which ran into mine, was a Buick. be 'I sometimes spent'. Simple past tense is used for
something that was true for some time in the past. So
16. Dan went to the airport to pickup Dave, and Ellie went 'had spent' is incorrect.
to the train station to meet Debbie.
28. d The sentence is correct in its given form.

VA / Exercise - 11 CEX-5331/P1BS/17 / Page 1


29. b Part (b) contains an error. 'main country' should be 44. b In case of statements that express something unreal
replaced by 'mainland'. or contrary to fact, the correct structure is 'if he
had…he would have.' So, 'if he would have' in part (b)
30. b 'Try out' is incorrect. The correct phrase is 'try on'. should be replaced by 'if he had'.
'Try on' means to put on a piece of clothing to see if it
fits and how it looks. 'Try out' means to test or use 45. c Since 'rock' is a verb, the modifying word should be
somebody/something in order to see how good or an adverb. So, 'slow' should be changed to 'slowly'.
effective they are.
46. a 'Already' should be replaced by 'all ready', which
31. b There are two characters, so they cannot be 'an means the group was ready. 'Already' means prior to
orphan'. It should be replaced by 'orphans'. a given time or previously.

32. b In this sentence, the trends discussed seem to do two 47. b The verb should be 'requires'. The verb must agree
things: show and indicate. The verbs should be parallel with the subject, which in this sentence is singular -
in form. So, 'indicating' should be replaced by 'indicate'. 'each one'.

33. c 'Patients' are people receiving medical treatment. So 48. c Change 'I' to 'me'. The pronoun is the object of the
replace 'patients' with 'patience' which means verb.
tolerance.
49. c 'being' in part (c) is incorrect. It should be replaced by
34. a The sentence should read as 'Saif knew that the other 'to be'.
applicants weren't as good as he…'; using 'him' here
is incorrect. The sentence can also be written as 50. b 'Affects' has been incorrectly used here. 'Affect' refers
'Saif knew that the other applicants weren't as good to the conscious subjective aspect of an emotion
as he was…'. considered apart from bodily changes. It should be
replaced by 'effects' which refers to a change that
35. a 'Traveled' is incorrect here. The rest of the sentence somebody/something causes in somebody/something
describes the plot in the present tense. So 'travelled' else.
should be replaced by 'travels'. Moreover, in vivid
narratives, simple present tense is used as substitute Practice Questions
for simple past.
1. "Only one option was available to us: surrender," said
36. a The correct usage would be 'has attracted people' the ex-major, "and we did."
because the subject 'annual trade fair' is singular.
2. Over the last six months, the security establishment
37. b The correct usage would be 'to see' and not 'seeing'. has been signaling a new policy of robust response,
the kind the country has not witnessed in the recent
38. d The sentence is correct in its given form. past.

39. b There is parallelism error in the sentence. 'arriving of 3. Walking up and down Church Street, I realised that I
computers' should be replaced by 'arrival of computers'. simply could not find Gangaram's.

40. b When two actions occur in the past, the earlier action 4. You don't want any more rolls, do you?
is indicated by the past perfect tense. So, the correct
phrase would be 'Jim had left the city'. 5. My cell number has three 7s and Tim's phone number
has four 2s.
41. b 'rather' in part (b) is redundant.
6. They called it the 'land of milk and honey'.
42. b The pronoun 'one' is used in part (a). To maintain
7. Throughout her life, my mother-in-law reigned over
consistency, 'you' in part (b) should be replaced by
the kitchen, wielding the pans, ladles and tawas with
'one'.
a skill and finesse that the most seasoned chef would
have envied.
43. b You cannot compare other sculptors to Rodin's
creations. The correct sentence should be 'unlike other 8. It wasn't easy: to begin with, I had to find the right
sculptors' creations' house.

CEX-5331/P1BS/17 / Page 2 VA / Exercise - 11


9. On a motorcycle, my grandmother has traveled to Delhi, 24. c The sentence incorrectly uses ‘each other’. ‘One
India; San Francisco, California; and Paris, France. another’ is used when the group consists of more
than two people, animals or things. ‘Each other’ is
10. Today, my son, who is around 30, says, "old is gold." used when the group consists of just two people,
animals or things.
11. It, therefore, needs to be asked if it is possible or
practicable to reconcile these divergent laws. 25. d This sentence is grammatically correct.

12. NPR recently featured an article entitled "Egg War: 26. b There is a pronoun-agreement error here. Since the
Why India's Vegetarian Elites are Accused of Keeping subject is you change yourself to you.
Kids Hungry."
27. c The pronoun me is incorrect, as the sentence indicates
13. So, what signal exactly are Mr. Doval and Mr. Parrikar that Amit liked the new model more than the author
sending to Islamabad? liked the model; not that Amit liked the new model more
than he liked the author. It should read either I or I did.
14. Congrats! You finally got your Master's degree.
28. c Weakly is describing the appearance of the paintings,
not the painter’s sense of vision. So, it should be an
15. Amaravathi, the capital for Andhra Pradesh after
adjective (which describes a noun), not an adverb. It
bifurcation, is to be a fabulous riverfront city modelled
should read look weak.
on Singapore.
29. c When comparing two people, you must always use
16. Is it necessary to build [the] capital here at the cost of
the comparative form the more rather than the
farmers' livelihood?
superlative form the most which is only used to
17. Cong. hardens stance; demands resignations. compare three or more things or persons.

18. A photograph published in The Sun, of Queen Elizabeth 30. c In this option, it is a singular pronoun, which doesn’t
giving the Nazi salute as a six-year-old, has expectedly agree with the plural subject species. This sentence
raised a storm in Britain. should read they swing.

19. Down south in Florida, Governor Baxter has a single 31. c ‘the more suitable’ is correct. A superlative (most, best
response to any news report she dislikes - defamation etc.) is not used for comparing two things.
case.
32. b ‘who are always’ is the correct usage.
20. True, in an apartment building, the roof space - and
thus the potential generation - available to each home 33. b ‘a lot of travelling’ is the correct idiomatic usage.
is limited.
34. d No error.
21. a (a) should read ‘When she completes…”. Using
‘complete’ in the past tense would be incorrect here 35. b ‘so’ used in the part (b) is not required.
as the latter half of the sentence uses the future perfect
continuous tense to denote an action that will continue 36. c This sentence has used the incorrect conjunction.
into future with reference to the earlier action which Since the sentence contrasts two very different things,
would act as the starting point for it. replace and with but for greater logic and clarity.

22. a The sentence has the error of a misplaced modifier. In 37. c There is a verb-tense problem in this answer, change
the sentence it seems as if the books are the subject spent to spend since target (the first verb in the
of ‘waiting’. The correct way to state the sentence is
sentence) is in the present tense.
“The books lay speckled around in the room, waiting
for some semblance of order”.
38. a The verb should be claim or have claimed. There is
no reason to use the past perfect tense as there is no
23. c The sentence has an error of pronoun antecedent,
action occurring after it to warrant this tense.
the pronoun used in the sentence is ‘one’, the pronoun
used in (c) should be ‘one’ instead of ‘he’. According
39. a The pronoun (which) in the sentence refers to the
to rules of grammar the antecedent should match the
students. Thus, the sentence should read the students
precedent.
who. The word which is appropriate for everything
else except humans.

VA / Exercise - 11 CEX-5331/P1BS/17 / Page 3


40. c Parallelism is the problem in this sentence. To keep the 43. a The use of ‘them’ is incorrect. The correct usage would
list parallel – addiction, obesity, diabetes - omit to be ‘themselves’.
before diabetes.
44. b Whenever the modal ‘did’ is used, the verb that follows
41. d No error. ‘A number’ as a determiner is always plural takes its original form. Hence, ‘had’ should be replaced
and ‘the number’ as a determiner is always singular. by ‘have’.
Therefore, the sentence maintain correct subject-verb
agreement. 45. d When describing two events, which have happened
in the past, simple past is used for the part that has
42. a This is a case of subject-verb disagreement. The happened later. There is no error.
subject, ‘the points’, is plural and it should have a
plural verb. Hence, ‘allow’ should be used in place of
‘allows’.

CEX-5331/P1BS/17 / Page 4 VA / Exercise - 11


Verbal Ability – 12 P-1(BS)

Phrase Replacement and Sentence Improvement


Answers and Explanations

1 a 2 c 3 b 4 a 5 b 6 b 7 c 8 a 9 d 10 c
11 b 12 c 13 d 14 c 15 c 16 b 17 c 18 a 19 a 20 b
21 c 22 c 23 b 24 b 25 d

Practice Questions
1 a 2 c 3 d 4 a 5 d 6 d 7 b 8 d 9 a 10 c
11 a 12 b 13 a 14 d 15 b 16 c 17 b 18 a 19 d 20 d
21 a 22 d 23 a 24 a 25 b

1. a According to subject-verb agreement, a singular (a), (b) and (d) are logically incorrect. 'Into' as a
subject takes a singular verb, while a plural subject preposition means to or towards the inside of
takes a plural verb. ‘Everybody’ is a singular subject something. Hence, option (c) is correct.
and should be followed by a verb in the singular form.
Hence, the correct usage is: 8. a ‘Besides’ means in addition to and ‘beside’ means ‘at
“Everybody……was enjoying his or her……place.” the side of’. Hence, the correct usage is: “There
Hence, option (a) is the correct answer. are…….besides…..department”. Also, the plural form
‘are’ should be used because ‘assistants’ is plural.
2. c The preposition ‘to’ should follow ‘accustomed’. So, Hence, option (a) is the correct answer.
the correct usage is: “Americans…….accustomed
to…..meals”. Hence, option (c) is the correct answer.
9. d ‘An average person’ is singular and will take the
singular verb ‘has’. So, options (a) and (b) are negated.
3. b Option (a) is grammatically incorrect. The correct way
‘Hair’ is used when we refer to it as a whole. ‘Hairs’ is
of writing the sentence is ‘...he was disinterested...’
used when we can count them. In the given case,
Option (d) is incorrect because the correct usage is
‘disinterested in’ and not ‘disinterested about’. Hence, ‘hair’ is used as a count noun. So, the sentence is
option (b) is correct. correct in its given form.

4. a ‘Every student’ is a singular subject, so, the correct 10. c Option (a) is incorrect because ‘you’ is never used
pronoun to be used in the given case will also be with the verb ‘goes’. The correct way of writing the
singular (his or her) and not ‘their’. Hence, option (a) is sentence is ‘...if you go...’ Option (b) is grammatically
correct. incorrect. The correct way of writing the sentence is
“what else can you expect if...” Hence, option (c) is
5. b In case the gender is not specific, we use the the correct choice.
possessive pronoun ‘its’. Therefore, the correct
answer is given in option (b). Option (a) uses ‘it's’ 11. b We are talking about more than one reason, so, the
which means ‘it is’. And options (c) and (d) are incorrect plural form of the verb should be used. Also, you do
because gender is not specified in the given sentence. not work ‘at’ a part-time job. You ‘have’ a part-time job.
Hence, option (b) is the correct answer. Hence, option (b) is the correct answer.

6. b ‘Break in’ is an intransitive verb and it has no direct 12. c Option (d), 'about closing trials' is grammatically
object and ‘break into’ is transitive verb and we should incorrect. 'Whether' as a conjunction is the correct
specifically what was broken into. The correct usage to indicate choices or possibilities. However,
sentence should be: thieves broke into his house option (a) is incorrect because 'judges' is plural and
yesterday. Therefore, option (b) is the correct answer. can not take the singular pronouns 'he or she'. Similarly,
option (b) is incorrect. Thus, option (c) is the correct
7. c The right preposition to be used here is ‘upon’. ‘Sprang’
answer.
means a leap or sudden movement; to jump. Options

VA / Exercise - 12 CEX-5332/P1BS/17 / Page 1


13. d The given sentence is absolutely correct and need actions that have happened earlier than the other
not be changed. Hence, option (d) is the correct action. For the latter action, we use the simple past
answer. tense. Option (c) has the correct tense structure and
thus is the answer.
14. c As per the rule, when using ‘neither...nor’ in a sentence,
if both elements are singular, then the verb is singular 23. b We cannot use the singular form of the verb ‘to be’ for
too. For example: neither Sita nor Gita is going to write the subject ‘I’ here because it is a wish. Here, we
the letter. In the given context, the correct sentence need to use ‘were’. As per English standards, we use
should be: neither she nor her plight was any longer 'were' when writing of something hypothetical,
amusing. Hence, option (c) is the correct answer. imaginative, unlikely, or contrary to fact. Hence, option
(b) is the correct answer. Option (c) is incorrect
15. c ‘On opening a bag’ is a modifier which is referring to because ‘the show of NCPA’ is incorrect. The show
the subject ‘Ram’. So, it has to be placed immediately does not belong to NCP.
after it. Hence, option (c) is the correct answer.
24. b We cannot use ‘have’ because the sentence talks
16. b The correct usage should be ‘worse’ as it is a about the Bhoi tribal group as a singular entity here.
comparative form of adjective ‘bad’. ‘Worst’ is a ‘Although’ is misplaced in options other than option
superlative form and should not be used here. Hence, (b). Hence, option (b) is the correct answer.
option (b) is the correct answer.
25. d In the given sentence since we are talking about many
17. c ‘'No sooner… than…' is the correct idiom, which is problems we cannot use singular form of the verb.
used to say that something happened immediately after ‘Have’ should be used instead of ‘has’. The given
something else. Option (c) has the correct tense statement is correct and thus, option (d) is the correct
structure (simple past tense, as it is used to indicate answer.
an action completed in the past) and thus is the correct
answer. Another way of writing the given sentence 26. Bill proudly displayed the antique ship, beautifully
is: "no sooner had the teacher entered the class than sanded, in his showcase.
the students stopped shouting." 'No sooner' introduces
the event that occurred first, hence, would take the 27. With only two per cent......
past perfect tense followed by simple past tense for
the second event. 28. The most important food-energy source for three
fourths of the world’s population is grain.
18. a ‘On a building/construction site’ is a correct usage.
Thus, the correct sentence should be, “The new 29. The possibility of earthquakes is ……
building is on the site of its predecessor on Milton
street.” Hence, option (a) is the correct answer. 30. The phenomena of public education is another example
of democracy at work.
19. a 'Fewer' is used when we are referring to people or
things in the plural nouns and for countable nouns. 31. Everyone except the pitcher and me was ……
'Less' is used when we are referring to something
that can't be counted or doesn't have a plural (e.g. 32. Neither the council members nor the mayor takes ……
money, air, time, music, rain). Thus, option (a) is the
correct answer. 33. …… for his/her sins.

20. b Only option (b) has the correct sentence structure 34. …… was more brilliant, but each worked in his sphere.
and thus is the right answer. There are parallelism
errors in the other options. 35. …… except Smith and her.

21. c Only option (c) is right as the sentence structure and 36. The job is worth doing.
tenses are correct. There are parallelism errors in the
other options. 37. He was deaf both to……

22. c The given sentence is grammatically wrong. We 38. I passed by ………


cannot use ‘would have’ twice in a sentence. As a
rule, the past perfect tense is used for describing 39. Afterwards we went home for dinner.

CEX-5332/P1BS/17 / Page 2 VA / Exercise - 12


40. I have a pain in my leg. was made by Gia. Option (b) is incomplete because it
does not explain who made the painting. The modifier
41. The messenger will arrive presently. has been placed correctly in option (a), rendering it
the correct answer.
42. The river has flowed over its banks.
5. d The given sentence is correct because it places the
43. I have now given up football. subject correctly. It was the group that was walking
through the jungle and then decided to change the
44. My brother has my English book. course. Option (a) suggests that it was the jungle that
changed the course. Options (b) and (c) do not contain
45. ……… delighted; he had.... a subject. Hence, option (d) is the correct answer.

46. …….. as great as that of any other…………… 6. d Present perfect continuous tense has been used
correctly in the given sentence which suggests that
47. ……… friendly, well mannered and keen witted. the old man started selling newspapers 10 years back
and is still selling them. Option (a) is incorrect because
48. Running for an important office, managing a large it should be ‘had to sell’. ‘Had to sold’ is grammatically
organisation and practicing...... incorrect. Option (b) contains an incorrect verb ‘have’
and (c) refers to the future tense. Hence, option (d) is
49. Edward resists not only learning to correlate new the correct answer.
facts but also remembering....
7. b In the given sentence simple present tense ‘when you
50. ……….. the girl whom he loved had left him for another. receive’ should be used. The given sentence is in
simple future tense, option (a) is in future continuous
Practice Questions tense and option (c) uses the past tense and distorts
the sentence. Hence, option (b) is correct.
1. a Option (a) places the modifier correctly. It is the dentist
who removed the braces of Kamal and advised him. 8. d The past continuous is used to indicate an action which
The given sentence is incorrect because if we remove was going on at some point in the past. Option (a)
the parenthetical information, the sentence will not uses the past perfect which is incorrect. Option (b)
make any sense. Option (b) is incorrect because ‘in changes the meaning of the given sentence. Option
spite of’ has been used which tries to define a (c) uses the past perfect continuous tense. Hence,
condition. Option (c) is incomplete because it is not option (d), which refers to an action (listening) going
clear who is giving the advice to Kamal. on in the past, is the correct answer.

2. c The given sentence is incorrect because it sounds as 9. a Since the sentence is in past continuous tense, ‘will
if it was the thesis that had gone through the reference be talking’ should be replaced by ‘was talking’. Hence,
material. Option (a) is incorrect because it was not the option (a) is the correct answer.
student who needed revision. Option (b) is incomplete
because it does not explain ‘what is it’ that the student 10. c Here, the sentence expresses a plan that is to be
decided to revise. Option (c) is the best choice since followed in the future. So, ‘I am not going to do‘ should
it clearly defines that it is the student who went through be used here. Hence, option (c) is the correct answer.
the reference material and decided that the thesis
needed revision. 11. a (b), (c) and (d) fail with the use of ‘model’, ‘draining’
and ‘capitalisation’ respectively.
3. d Option (a) is incorrect because the usage of ‘being’ is
inappropriate. Options (b) and (c) do not reveal the 12. b The sentence should be in the present tense.
correct meaning of the original sentence. They distort
the meaning of the sentence by placing the words 13. a (b), (c) and (d) fail by using the words ‘awake’, ‘shot-
incorrectly. The correct answer is option (d) which out’ and ‘accurate’ respectively.
clearly states that it is the mural that depicted a
masterpiece and the artist’s true vision and style. 14. d ‘Cannot’ indicates incapability, whereas ‘could not’
indicates ‘unwillingness’ as well. The sentence is also
4. a The given sentence and option (c) are incorrect in the present tense, so ‘what chance does’ instead
because they seem to suggest that the living room of ‘what chance did’.

VA / Exercise - 12 CEX-5332/P1BS/17 / Page 3


15. b Since the ‘high and low’ stand equal before the law, 32. Before designing a park, the architect must consider
‘justice is served’ in front of them. the public.

16. c With ‘since’, use ‘have been talking’ and ‘makes’ go 33. ……smarter than those in other offices.
with this tense.
34. Nancy had to choose between two dinner entrees.
17. b (b) is the best choice that indicates that the president
wished to ‘keep his views to himself’. 35. Could I have less soup please?

18. a What it means is that the usually peaceful House was 36. …….. indebted to ………
filled with arguments today.
37. You shouldn’t have said ……….
19. d Easy, use the term ‘on the behalf of’.
38. ……….. he could neither have a hot-plate in his room
20. d ‘The shopper’ takes the singular ‘tends’ verb. nor take showers after 11.00 o’clock.

21. a ‘The most recent’ here means ‘the latest’. 39. Joan’s broken yellow pencil came from this box.

22. d It is a modifier question, the comma should be followed 40. …….. not only all over the bus but also all over the
by the subject ‘Ahab’. sidewalk.

23. a This is the most succinct expression. 41. The audience felt both pride and satisfaction………

24. a (a) brings out clearly that the colleague must find a 42. ………… and inspirational………
new job.
43. ………… arrogant, pampered and hot-tempered
25. b The verb ‘joined forces’ should come after ‘agent’.
44. No error (No sooner…… than)
26. ……… spokesperson, but they came to realise that
Jane…….. 45. The reason the machine failed is that the motor………..

27. ……………. honoured since they bolstered the 46. I greatly enjoyed my sister’s wedding.
………….
47. Sandra could hardly remember a time when………
28. That he agrees with you does not ……………
48. …….. best time when playing polo.
29. The prisoner was extradited from ………..
49. No error (Not only – but also)
30. …………. he is likely to continue smoking.
50. ………. strap came out were like……...
31. ……. years, Jack was eager to get a job.

CEX-5332/P1BS/17 / Page 4 VA / Exercise - 12


Verbal Ability – 13 P-1(BS)

Error Spotting
Answers and Explanations
1 a 2 d 3 b 4 b 5 d 6 a 7 b 8 c 9 b 10 a
11 c 12 c 13 a 14 a 15 c 16 c 17 b 18 a 19 d 20 c
21 b 22 b 23 c 24 c 25 b 26 d 27 a 28 b 29 d 30 b
31 b 32 b 33 b 34 c 35 c 36 d 37 c 38 c 39 d 40 b

Practice Questions
1 b 2 b 3 d 4 d 5 b 6 c 7 c 8 a 9 b 10 d
11 d 12 c 13 c 14 a 15 b 16 c 17 a 18 d 19 d 20 d
21 c 22 b 23 c 24 a 25 b 26 d 27 d 28 b 29 a 30 c
31 c 32 a 33 c 34 b 35 b 36 d 37 d 38 a 39 c 40 c

1. a ‘Have grown’ is the correct form which should be 8. c 'Not only… but also…' is the correct idiomatic expres-
used as ‘conditions’ (subject here) are plural. As per sion. Thus, 'but also foster' is the correct form. Hence,
subject-verb agreement, plural subject takes plural option (c) is the answer.
verb. Hence, option (a) is the answer.
9. b In the given sentence, ‘the second subject’ is singular
2. d ‘Would they’ is incorrect and the correct form should and therefore should take the singular verb ‘is’ instead
be ‘would one’ because ‘would one’ refers to ‘one of ‘are’. The correct sentence should be: the second
wouldn’t want’ in part ‘c’ and can not be changed to subject for management is complimentary to the first
‘they, which is a personal pronoun for the third per- year’s study. Hence, option (b) is the correct answer.
son. Hence, option (d) will be correct.
10. a Instead of ‘its’, ‘it’s’ (it is) should be used. Hence, op-
3. b ‘Indignantly’ because an adverb form is required to tion (a) is the correct answer.
modify the verb ‘responded’
11. c The correct usage would be ‘which are’ because it
4. b 'Among' is the correct preposition which is used for refers to ‘two or more words’. Hence, option (c) is the
the choices that involve more than two items. 'Be- correct answer.
tween' is used for choices involving two items. Hence,
option (b) is the answer. 12. c Ingestion / consumption / intake of excessively rich
foods is yet another reason (error in parallelism).
5. d 'Necklesses' is incorrect and the correct form is 'neck- Hence, option (c) is the correct answer.
laces', which is the plural form of 'necklace'. Hence,
option (d) is the answer. 13. a 'Large amount of water bags' is incorrect and the
correct form should be 'large number of water bags'
6. a 'Had gone' is the correct form which should be used as 'number' is modifying water bag, which is a count-
instead of 'would have gone'. As a rule, the past per- able noun. Hence, option (a) is the answer.
fect tense is used for describing actions that have
happened earlier than the other action in the past. 14. a 'Look into something' means to examine something
Hence, option (a) is the answer. which is the correct phrasal verb to be used in the
given context. Thus, option (a), 'see in his eyes' is
7. b 'Were of the view' is incorrect and 'was of' is the incorrect in the given context.
correct form as 'committee' should be treated as a
singular subject (in the given context) and should be 15. c The correct usage would be ‘he had won’ because
followed by a singular verb, 'was'. Hence, option (b) when two actions are indicated in the past tense, the
is the answer. earlier action should be denoted by the past perfect
tense. Hence, option (c) is the correct answer.

VA / Exercise - 13 CEX-5333/P1BS/17 / Page 1


16. c 'Village' is a singular subject and should therefore 27. a The article ‘the’ should precede ‘gravest’. ‘The’ is
take the singular verb, 'was'. Hence, option (c) is the always used before a superlative that describes a
answer. noun. For example: Katrina was the most destructive
hurricane. Hence, part (a) is the correct answer.
17. b 'Teacher' is a singular subject and should therefore
take a singular verb, 'urges'. Hence, option (b) is the 28. b When we combine two clauses with 'together with',
answer. we use a singular verb. Therefore, 'are' in part (b)
should be replaced by 'is'. Hence, option (b) is the
18. a Instead of ‘will claim’, it should be ‘claims’. correct answer.

19. d ‘Of’ should precede ‘interest’. 29. d The sentence is correct in its given form.

20. c 'For attack' is incorrect and the correct form is 'to 30. b Possessive pronouns like my, your, his, her, etc. are
attack'. 'To' as a preposition is used as a function used before verbs ending in 'ing'. Therefore, 'me' is
word to indicate movement or an action or condition part (b) should be replaced by 'my'. Hence, option (b)
suggestive of movement towards a place, person, or is the correct answer.
thing. Hence, option (c) is the answer.
31. b It should be ‘agree’ and not ‘agrees’ because the subject
21. b The correct sentence should be "...it's going to rain is plural.
tomorrow." 'Its' is the possessive form of 'it', as in
"your dog is very cute, what is its name?" 'It's' is an 32. b It should be ‘nor’.
abbreviation for 'it is' or 'it has', as in "it's a nice day,
isn't it?" Hence, option (b) is the correct answer. 33. b It should be ‘were’ to agree with ‘countries’.

22. b What we need in this sentence is the adverb 'carefully' 34. c It should be ‘their’ to suit ‘provinces’.
that describes the verb 'drives'. One always drives
carefully, not careful. 'Careful' is an adjective that 35. c It should be ‘but had written…’.
requires a noun or a pronoun that it describes. Hence,
option (b) is the correct answer. 36. d It should be ‘global’.

23. c There should be consistency in the use of pronoun to 37. c The distinctions should be ‘between’ the two.
keep the sentence parallel. The sentence begins with
the pronoun 'one', which should be used elsewhere 38. c It should be ‘thinks and writes’.
in the sentence too. The correct sentence should be
"...work on one's steering..." The first pronoun can 39. d ‘Newest’ can be only one ‘person’.
also be changed to 'you', but that would also change
'learns' to 'learn' making two parts incorrect in the 40. b It should be ‘the other’s a’.
sentence which is not possible. Hence, option (c) is
the correct choice. Practice Questions
24. c 'Arrive at something' is a phrasal verb, which means 1. b It should be ‘in view of’.
to decide on or find something, especially after
discussion and thought. The correct sentence should 2. b Omit ‘so’.
be "…instructor arrived at a decision…"Thus, option
(c) is the correct answer. 3. d The sentence is error free.

25. b There is a subject verb agreement error. 'Large 4. d Use ‘themselves’.


corporations' is plural, which should be followed by a
plural verb 'seem'. Hence, option (b) is the correct 5. b Do not repeat ‘because’ if you have said ‘reason’.
answer.
6. c Use ‘to the custody of’.
26. d The sentence is correct in its given form. Hence, option
(d) is the correct answer. 7. c Use ‘unreasonably’.

8. a Use ‘appears very elementary to’.

CEX-5333/P1BS/17 / Page 2 VA / Exercise - 13


9. b It should be ‘lay’. 21. c It should be ‘the repeated occurrence…calls’.

10. d It should be ‘better’. 22. b 23. c 24. a

11. d It should be ‘had received’. 25. b 26. d 27. d

12. c It should be ‘casually’. 28. b 29. a 30. c

13. c ‘The colour’ should take singular ‘is’. 31. c 32. a 33. c

14. a It should be ‘experience. . . indicates’. 34. b 35. b 36. d

15. b It should be ‘further’. 37. d Surrounded by well wishwishers, Rohit experienced


a feeling of confidence and well being.
16. c It should be ‘between you and me’.
38. a It is preferable that you take this test now rather
17. a It should be ‘had seen’. than after a month.

18. d Use ‘its’ for collective singular pronoun. 39. c Having studied so hard, Shalini was all set to take a
long break from work.
19. d It should be ‘me’.
40. c If you can find me the right candidate I will give you a
20. d Use the term ‘total…is’. handsome reward.

VA / Exercise - 13 CEX-5333/P1BS/17 / Page 3


Verbal Ability – 14 P-1(BS)

Phrase Replacement & Sentence Improvement


Answers and Explanations

1 d 2 a 3 a 4 d 5 c 6 c 7 b 8 c 9 d 10 a
11 c 12 d 13 a 14 b 15 a 16 d 17 b 18 a 19 c 20 c

Practice Questions
1 b 2 a 3 c 4 a 5 c 6 d 7 b 8 c 9 a 10 a

1. d ‘Imposed by diktat’ means to be forced on the people 15. a All the other choices change the intended meaning of
by law. the sentence, (b) with ‘if’, (c) with ‘able to be trained’
and (d) is grammatically unsound.
2. a The author is more concerned about safety than dirt.
16. d There is a parallelism here ‘to shake. . . to lead’.
3. a You ‘hear’ a call.
17. b ‘Leaving it unable to act’ would still have been correct,
4. d ‘May not know’ indicates the present tense as well as in the absence of which we mark (b).
the modesty on the auhtor’s part.
18. a The other options are inappropriate.
5. c (a) and (d) are grammatically inaccurate and (d) does
not take the long-term time span into account. (b) is 19. c The comma should be there ‘at best, remain only’.
more emphatic.
20. c The other options are either wordy or inappropriate.
6. c You should use ‘avert’ to mean ‘prevent’; ‘could’ is a
diplomatic way of indicating a high rate of probability. 21. As the door opened, the crowd surged in.
Pandemonium erupted.
7. b ‘From’ and ‘to’ indicates the spectrum of the activity.
22. They had clearly lost their way.
8. c ‘In emulation of’ means ‘to copy someone’.
23. ...neither ….. nor....
9. d (d) brings out the meaning best, passenger car sales
were not doing well enough, but the growth became 24. His becoming upset was quite...
positive in spite of negative growth figures by a few
manufacturers. 25. Have you ever visited San Juan?

10. a ‘Efficacious’ means to ‘produce the desired result’. 26. ……….. is red and exceptionally bulky.
‘Switch over’ is to change to or choose someone or
something else. 27. Since Harnet ………

11. c ‘Not only. . . but also’ is a healthy idiomatic expression. 28. My family has lived…………

12. d ‘ordered that. . . be killed. . . vaccinated’ is the right 29. Everyone except him and me………..
expression of parallelism.
30. ………. than any other girl……..
13. a The CBI would take the singular pronoun ‘it. . . its’.
31. ....than it was in the past.
14. b The sentence means ‘India exiled. . . over a period of
20 years. . .’ 32. Remove ‘first’.

VA / Exercise - 14 CEX-5334/P1BS/17 / Page 1


33. …….. than it occurred fifty years ago. 10. a Here we are not talking about writing speed, but a
certain style of writing, so you say ‘at random’ just as
34. …….. likely to be at a severe disadvantage….. you say ‘at will’.

35. …….abstinence while one is an adolescent. 11. If they had paid attention…………..

36. …… atmosphere when hydrocarbons and nitrogen 12. She is not running and does not intend to
oxides react …….. run……………

13. He disapproves of your insisting that the rope of pearls


37. ………… one had better be prepared...
was purposely misplaced.

38. He swims like a fish. 14. The car runs more quietly when I add a heavier
transmission fluid.
39. ….. worked as slowly as……….
15. ………. record was better than or at least as good as
40. ……….. ability to ………. work as hallmarks...... hers..
16. ....ditch, but she……
41. If I were he, ……………
17. It was an awesome sight.
42. ……… to the person who drew………………
18. …… arrived, however, it was...
43. ......energy resources, a circumstance that resulted in
19. ……… to go unless……….
unemployment among the workers.
20. ………… spoke softly and politely.
44. The author in this article implies that everybody who
…………. 21. ……… than to earn.

45. …………… realized that I had forgotten to add oil. 22. No error.

23. ……… bed early and to get up…….


Practice Questions
24. ……… has worked harder and has gone to
1. b Use the same simple past tense for actions happening fewer………..
together — ‘approached. . . started’.
25. …….. as walking……….
2. a (b) makes no sense, (c) does not say who maligned
the book and (d) is grammatically wrong. 26. …….but also for his inability to meet the school’s
standard…..
3. c (a) and (b) imply that ‘his friends’ arrived at the airport.
(d) does not show gratitude to the ‘friends’. (c) is most 27. …….. played and to the late ……….
accurate.
28. ……… and submitted it to ………
4. a (b) may have been considered if ‘once’ had been
replaced by ‘ones’. (c) with ‘they’ is too specific. (d) 29. No error.
cannot be accepted on grammatical terms. (a) talks
about the majority who do care for their clients. 30. ……………….. I am obviously at peace.

5. c The comma should be followed by the subject ‘she’. 31. After his becoming president, his hometown became
prosperous.
6. d You build ‘bridges, buildings’, for ‘links’, preferably use
‘forge’. 32. When you drive a tractor, your patience must……..

7. b Use ‘deposits swelled’ enticingly which can even go 33. ………….. principal, we felt our anger subside.
with ‘appetite’ in the sentence.
34. As I drove down the ………
8. c
9. a ‘Scold at will’ means ‘to scold as one pleases’. 35. ………… is going to make......

CEX-5334/P1BS/17 / Page 2 VA / Exercise - 14


Verbal Ability – 15 P-1(BS)

Grammar Revision Sheet – 2


Answers and Explanations

1 c 2 b 3 c 4 d 5 c 6 d 7 c 8 b 9 c 10 c
11 a 12 c 13 d 14 c 15 a 16 c 17 c 18 c 19 c 20 a
21 c 22 d 23 d 24 d 25 b 26 d 27 d 28 a 29 b 30 c
41 b 42 d 43 b 44 a 45 c 46 c 47 a 48 b 49 a 50 c
51 a 52 c 53 d 54 a 55 c 56 d 57 b 58 b 59 d 60 a
61 b 62 c 63 a 64 b 65 a 66 d 67 c 68 b 69 b 70 a

1. c To make the structure parallel 'who will benefit' should 11. a Part (a) has an error. The correct way to phrase it is
be matched with 'to what extent' because 'and the "The day was both long and tedious"
extent' does not have the component of questioning,
though the context requires that. 12. c The error is in part (c). The correct phrase should be
"not for words but for action". Correlative
2. b In part (b), rather than 'consolidating', it should be
expressions like 'and' and 'but' should be followed by
'consolidate' because the verbs should match when
the same grammatical construction.
in a string.

3. c Both should be followed by 'manufacturers and 13. d The sentence is correct in its given form.
dealers' to keep the structure parallel.
14. c The sentence has two nouns - 'punctuality' and
4. d The sentence is correct in its given form. 'meticulousness' and a clause - 'his continued hard
work'. The make the structure parallel, the clause
5. c 'Running' is a gerund here whereas an infinitive is
should be replaced by 'diligence'. So, part (c) will
required to match 'to have' and 'to buy'.
become 'and diligence'.
6. d Only option (d) is consistent because it adheres to the
structure of the sentence. 15. a The article 'the' should be used consistently with each
noun. So the correct phrase is 'The Chief Minister of
7. c The sentence is given in active voice. So, the same Delhi and the President of America'.
should be followed. Option (a), (b) and (d) are incorrect
because they are not in the active voice. So, option 16. c 'Should' in part (c) should be removed to make the
(c) is correct. sentence parallel in structure.

8. b The given sentence uses the -ing form of the words. 17. c 'Boy scouts presented' and 'band marched' are in
So, the same structure should be followed. Thus, simple past tense while 'mayor was riding' is in past
option (b) is correct. continuous tense. To make the structure parallel, 'mayor
was riding' should be replaced by 'mayor rode'.
9. c Option (a) and (b) both contain errors of parallelism. In
option (d) the parallel structure is incorrect and the 18. c Both 'class' and 'work' are followed by a preposition.
use of ‘full’ is redundant. So, option (c) is correct So, 'home' should also be followed by a preposition.
because all the adjectives are used correctly. So, the correct phrase is 'or at home'.

10. c A double negative is a non-standard usage of two 19. c The correct structure is 'not only to…but also to'. So,
negatives. Options (a) and (b) contain this grammatical the correct phrase is 'but also to use metalanguage'
mistake. Option (d) alters the meaning of the given
sentence and contains a parallelism error. In option (c) 20. a 'that clause' construction should be used consistently.
the parallel structure is correct and the use of ‘forgiving’ So, the correct phrase is 'Her conditions are, first, that
and ‘just’ gives the correct meaning to the sentence. she should'.

VA / Exercise - 15 CEX-5335/P1BS/17 / Page 1


21. c That weekend our plan was to go up the mountain, 39. He hid his cell phones in the following places: Mrs.
as far as the first camp. Patil's room, Mr. Prasad's office and Miss Julie's
cupboard.
22. d Lately, people have stopped using the buses as the
service has become unreliable. 40. That tall man, Swati's granddad, is this month's win-
ner.
23. d As soon as they reached the camp-site they pitched
their tents, lit a fire and washed up. 41. b Generally, we use a colander to drain (discarding
liquids like pasta water) and a sieve to strain (saving
24. d The teacher said that any student who missed school liquids like broth for stock). Also, a sieve is small in
should get a letter from his/her parents. size and it can be difficult drain pasta in it. So, 'sieve'
should be replaced by 'colander'.
25. b The farmers faced a crisis and could be out of
business, if the output / dropped any lower. 42. d No improvement is required in the given sentence.

26. d If only people realised the importance of trees they 43. b 'Scarcely' means almost not while 'seldom' means
would not cut them so freely. almost never. So, 'scarcely' is inappropriate in the
context. Thus, options (a) and (d) are negated.
27. d It is a pity that you cannot see that your parents are 'Seldom ever' is grammatically incorrect. So, option (c)
really worried about you. is also negated. The sentence means that the tree
rarely grows over 4 inches. So, option (b) should
28. a Should you feel scared, you can give me a call and I replace the underlined part.
will come over.
44. a The correct phrase is 'work wonders', which means
29. b My sixth sense tells me that I should not expect him to have a very beneficial effect on someone or
to keep his word. something.

30. c The children were playing with a ball and running 45. c 'Each of the two' is incorrect. The underlined
around when the accident occurred. portioned should be replaced by 'both the brothers'.

31. The City-like large, angular headlights that fuse into 46. c 'Price' of something is never 'charming', 'bewitching'
the multi-element grille, in particular, look very attrac- or 'captivating'; it can, however, be 'attractive'.
tive.
47. a You empathize/sympathize 'with' and not 'for'
32. "That's something I've been thinking about," Ranjani someone. So, options (b) and (c) are negated.
says, "I feel it's the lack of positive news that's made 'Empathize' is to share the same feelings as another
it stand out." person while 'sympathize' is to feel sorry for
someone who is in a bad situation. Here, the sentence
33. Samar, the owner of Skip's Car Wash Service, said he will take 'empathize' since a 20-year-old stylist can
would be here on Wednesday; however, there's sure feel sorry for a mother of three but cannot share
always a chance he won't be able to come, especially the same feelings.
if he is not feeling well.
48. b Option (a) is incorrect since it will change the meaning
34. My elder brother, who lives in Bangalore, is a doctor; of the sentence. The sentence means that she did not
my younger sister, who lives in Delhi, is a teacher. know much about her sister. 'surprising less' is
grammatically incorrect, rendering option (c)
35. Tired of answering the reporters' questions, actor incorrect. 'Low' is used for things that are not high or
Salman Khan gritted his teeth and angrily retorted, tall. So, option (d) is also negated. 'Little' here means
"That's none of your business, you morons!" not very much, thus making option (b) correct.

36. Smita Patil, Ph.D., is the director of undergraduate 49. a 'Gushing' means flowing very quickly and in large
studies in our college. amounts. So, option (d) is incorrect. 'Speeding' means
driving faster than the legal speed limit. Since the driver
37. He had only one desire in life: to master the art of is in the car, he will be 'speeding' and not 'running'.
origami.
50. c Option (a) is incorrect because it appears that they
38. The ice hockey player made a controversial move: a unexpected met a ride, which cannot be true. Option
cross-check on another player. (b) is incorrect since it does not bring about the

CEX-5335/P1BS/17 / Page 2 VA / Exercise - 15


complete meaning of the sentence. The sentence says have been grammatically correct. 'Every bit complex'
that they 'asked' the friend to give them a ride, which means equally complex. It does not make any sense in
cannot be understood from option (b). Option (d) is the sentence. 'Therefore' means for that reason. It
incorrect because 'while walking through the village' does not make any sense in the sentence.
has wrongly been placed after 'ride'. Option (c)
correctly brings out the meaning of the sentence, and 62. c You do not call off a 'search team'; you call off a
is hence, correct. 'search'. So, option (c) is correct. 'Look out' means to
search for something from among your possessions.
51. a Adjectives usually take the following order:
Generalopinion - Specific opinion - Size - Shape - Age 63. a 'Off late' is incorrect. The correct phrase is 'of late'
- Colour - Nationality - Material which means recently.

52. c Usually, 'after' is used before 'while' (after a while) 64. b 'All-in-all' means on the whole. It does not fit in the
and 'later' is used after 'while'(a while later).'Little' is meaning of the sentence. The correct phrase is 'all-in',
not used with time; it is used with 'while'. With time we which means including the cost of all parts of
use 'some' (some time later). something. 'All out' means in a very determined way
using every possible effort.
53. d No improvement is required in the given sentence.
'Exclusion' is a situation in which a child is banned 65. a Sentences that contain the phrase 'one of those' take
from attending school because of bad behaviour. a singular verb.So, option (a) is the correct answer.
Option (c) is incomplete and hence, incorrect.
54. a Past subjunctive is used after as if/as though to indicate
unreality of improbability. So, the underlined phrase 66. d The sentence is correct in its given form.
will be replaced by 'as if I were her husband'.
67. c The correct structure is 'would rather…than'. It means
55. c 'Many a' is singular. So it should be followed by 'pirate'. would prefer to.So, option (c) is the answer.
So, the correct sentence is 'Many a pirate has lost his 'Instead…than' is not a correct structure. If we were
life at sea.' to use 'instead' in the sentence, then the sentence
would be "She will not give a speech; instead, she'd
56. d The sentence is correct in its given form. die."

57. b The preposition 'by' should follow 'pass'. 'Pass by' 68. b Past subjunctive is used after the verb 'wish' to indicate
means to go past. a situation which is unreal or contrary to a fact. So,
'belong' should be replaced by 'belonged'. 'She' is
58. b 'Meet with something' means to be received or treated singular. So, 'wishes' will be used instead of 'wish'.
by somebody in a particular way. Other options will
render the sentence meaningless. 69. b 'At the time' suggests that the idea looked good
sometime in the past. So, the underlined word should
59. d 'Give over' is used to show that you do not believe be in the past tense. So, option (b), 'seemed', is the
what has been said to you. So, the sentence is correct correct answer.'Seemed to be like' is grammatically
in its given form. 'Give in' is to finally agree to what incorrect.
someone wants, after refusing for a period of time.
'Give up' means to stop trying to guess. If a machine or 70. a The correct phrase is 'as/so far as to'. The phrase
part of your body 'gives out', it stops working. does not required the 'ing' form of verb. So, option (a)
is the correct answer.
60. a The correct phrase is 'far more advanced'. 'Far' here
means very much or to a great degree. 'By far' is used Practice Questions
with comparative or superlative adjectives or adverbs.
It meansby a great amount. For example: The last of 1. Speaking to the Indian community, the President asked
these reasons is by far the most important.'By far' as them to join the campaigns launched by Prime Minister
used in option (b), will change the meaning of the Modi such as 'Digital India', 'Startup India, Stand up
sentence. 'Far more advancing' and 'far most India', 'Clean India Mission', 'Smart Cities' etc.
advanced' are grammatically incorrect.
2. Last week during patrol, a team from the Delhi Police
61. b 'However' is used with an adjective or adverb to mean Vigilance Branch spotted two police personnel "striking
'to whatever degree'. So, option (b) will replace the a deal" with a man they had apprehended for carrying
underlined phrase. 'No matter complex' is grammatically Rs 95 lakh in cash.
incorrect. Had it been 'no matter how complex', it would

VA / Exercise - 15 CEX-5335/P1BS/17 / Page 3


3. Balaji Motion Pictures CEO Aman Gill said: "It's too early 17. Four lawmakers - two MPs and two members of the
to say that right now. But 2,000-odd screens, may be House of Lords - debated the question "Would Brexit
a bit more, right now that's the attempt, so we will benefit India" in a committee room in parliament.
have clarity by tomorrow mostly."
18. According to Blackman, the EU had moved since 1975
4. Hasan, who was also not present in court, will undergo from being an economic organisation to a political one.
a medical examination to see if he is fit enough to
withstand his punishment. 19. Renewed clashes overnight at a Pakistan-Afghan
border crossing killed an Afghan border guard and
5. Under "Jet Advance" facility, the passenger will have wounded five, an Afghan official said Wednesday as
the flexibility to change the booking to an earlier flight Islamabad dispatched more troops and weapons to
for a nominal fee, Jet Airways said in a statement. the volatile boundary amid an escalation in tension
between the two neighbours.
6. National carrier Air India had 3111 delayed flights, while
20. There was some damage on the Pakistani side, they
low-cost domestic carrier Spicejet had 2205 delayed
said, without elaborating.
flights.
21. Last month, the Torkham crossing was closed over a
7. The US Congress' decision to not subsidise the sale similar incident and reopened following a meeting
of eight F-16 fighters was the latest nail in the coffin. between Pakistan's army chief Gen. Raheel Sharif
and Afghan Ambassador Omar Zakhilwal.
8. One, it saw Pakistan as the indispensable partner of
its grand design for a negotiated settlement between 22. China, apparently acting at the behest of its "all weather
the Taliban and the Kabul regime. ally" has strongly opposed India's candidature.

9. Washington has been wearying, as Christine Fair of 23. In the roller-coaster world of global capitalism and
George Washington University has written, of Wall Street adventures, mergers are more often than
"Pakistan's tactics of extracting rents from Washington not questionable successes.
while proving itself unable or unwilling to bring the
Afghan Taliban to the negotiating table". 24. LinkedIn is a strategic fit from that point of view
because its 433 million users, 35 million of whom are
10. With India's equity in the US rising, Pakistan phobia in India, are businesses and professionals who can
rampant in the Beltway and Pakistan believing it has a potentially become Microsoft's premium customers for
geopolitical white knight in the form of China, the curtain a variety of applications and services including online
is falling on the latest act of the poisonous Washington- storage.
Islamabad drama.
25. Among the biggest of them, Internet calling firm Skype,
11. The 100000 ton Stennis, which carries F18 fighter
employee networking site Yammer and Nokia's mobile
jets, joined nine other naval ships including a Japanese
phone unit are not significant successes.
helicopter carrier and Indian frigates in seas off the
Japanese Okinawan island chain.
26. Under normal circumstances, when they have to earn
12. Tensions between Beijing and Tokyo recently jumped their meals, only the fittest survive, keeping the
after a Chinese warship for the first time sailed within population in check.
24 miles (38 km) of contested islands in the East China
Sea. 27. In effect, over the years, tens of thousands of Indians
have died on the roads because governments have
13. It's not so much the higher commissions as the charges neglected urban public transport.
on product returns and frequent policy flip-flops that
have got the vendors in a tizzy, according to the 28. The Chileans will meet Group C winners Mexico in the
merchant associations. quarter-finals on Saturday, with Argentina facing
Venezuela the same day, thanks to their 3-0 win over
14. Santram's father Atmul Murjani had found the old man, Bolivia to top Group D earlier on Tuesday.
still in his military dress, in 1971 at Sholapur bus stand.
29. Going by the words of skipper Dhoni, more changes
15. The left-hander replaced Karun Nair at the opening are likely in the Indian batting order as only the top
slot, after the latter couldn't bring much success in the three batsmen have gotten substantial time in the
first two ODIs against Zimbabwe. middle.

16. In the 48 Twenty20 matches he has played, he scored 30. China would have probably, and maybe gladly, let its
896, with 66 being the highest at an average of 20.36. Great Fire Wall down for the message to be delivered
to India.

CEX-5335/P1BS/17 / Page 4 VA / Exercise - 15


Verbal Ability – 16 P-1(BS)

Eclectic Reading and Level – 1 Questions


Answers and Explanations

1 d 2 b 3 d 4 c 5 c 6 a 7 c 8 c 9 b 10 d
11 d 12 c 13 d 14 d 15 a 16 c 17 c 18 b 19 a 20 c
21 b 22 b 23 d 24 b 25 a 26 d 27 c 28 c 29 a 30 b

Practice Questions
1 b 2 a 3 d 4 a 5 d 6 b 7 c 8 a 9 c 10 b
11 b 12 d 13 b 14 a 15 d 16 c 17 b 18 a 19 c 20 b
21 d 22 b 23 b 24 d 25 a 26 c 27 b 28 c 29 b 30 c
31 c 32 d 33 b 34 d 35 b 36 b 37 c 38 b 39 b 40 d
41 a 42 d 43 b 44 a 45 d 46 b 47 c 48 d 49 b 50 c
51 d 52 c 53 b 54 a 55 d 56 d 57 a 58 b 59 d 60 c

1. d Refer to the first sentence, “Primitive…source of 7. c The third paragraph states that during the industrial
warmth… a means of cooking food…” Hence, option revolution, more and more coal was needed for the
(d) is the correct answer. steam engine and factories. Hence, option (c) is the
answer. Local industries used coal even before the
2. b Refer to the sentence, “His discovery ...... used for advent of Industrial Revolution. However, it cannot be
illumination…preserving a flame”. This clarifies that inferred from the passage that this demand increased
the primary purpose of the firebrand was to preserve at any given point of time.
a flame. Hence, option (b) is the correct answer.
8. c The penultimate paragraph lists several problems
3. d Refer to the sentence, “His discovery ...... …incidental (flooding, fire damp, poison gas, underground pit
to the primary....” This implies that ‘primary’ refers to collapses) faced by the mine workers while working
firebrand ‘mainly’ used for preserving. Hence, option in the mine. It also says that the weight of the ground
(d) is the most appropriate answer. above a worked coal seam was extremely large. It
does not mention anything about splitting of the coal
4. c Refer to the sentence, “Lamps …developed by seam. Hence, option (c) is the answer.
accident”. This clearly states that lamps were
developed incidentally, without planning. Option (c) 9. b The last paragraph states that despite the dangers,
‘chance’ is the most appropriate answer. the production of coal increased. We cannot infer that
the increase is because of the increase in the working
5. c ‘Persist’ means to continue to do something despite population. The author mentions “regardless of these
difficulties or opposition, in a way that can seem dangers” only to emphasise the fact that it did not act
unreasonable. The appropriate antonym from the given as a deterrent. Moreover, we know that the coal shafts
options is ‘discontinued’. Hence, option (c) is the most were now bigger, which might be the only reason that
appropriate answer. led to the increase of coal production. Hence, option
(b) is the correct answer. Options (a) and (c) can be
6. a Refer to the first paragraph of the passage, where it clearly inferred from the third paragraph. Option (d)
is stated that Britain “had to make do” with charcoal can be inferred from the last paragraph which states
because it was cheap and easy to acquire. Statements that industries grew around coal mining areas which
B and C are not mentioned in relation to charcoal. created problems since no thought was given to the
Hence, the answer is option (a). facilities that the people settling around the industries
would need.

VA / Exercise - 16 CEX-5336/P1BS/17 / Page 1


10. d ‘Colossal’ means of an exceptional or astonishing 18. b The word ‘Lingua Franca’ means a shared language
degree. ‘Pharaonic’ means something that is enormous of communication used between people whose main
in size or magnitude. Hence, option (d) is the correct languages are different. Hence, option (b) is correct.
answer. ‘Petite’ means having a small trim figure.
‘Cosmogony’ refers to the theory of the origin of the 19. a Refer to the lines “The major challenge today is not
universe. finding jobs but finding enough skilled labor to maintain
the competitiveness of the economy.” Hence option
11. d Options (a), (b) and (c) are incorrect as they are not (a) is correct.
mentioned in the passage. Option (d) is the correct
answer as according to the given passage, 20. c Refer to the first paragraph of the passage which
Vivekananda preached that “Truth alone prevails”. states that “Numerous churches, temples, mosques
and pagodas exist next to each other proving the
12. c The passage shows that Vivekananda’s influence led remarkable level of religious and cultural tolerance”.
to a decline in the missionaries’ funds. Refer to the Both statements 1 and 3 can be inferred from this line.
sentence, “The missionaries found that the The same sentence negates 2. Statement 4 is false
subscriptions they had so long been receiving from because although the passage states that most of the
the people were steadily declining.” Hence, option (c) population comprise of Hindu Indians, it makes no
is the correct answer. reference to the number of temples. Hence, option (c)
is correct.
13. d Refer to the line, “Vivekananda also said that India did
not need religion but material support.” From this we 21. b The two capitals talked about in the passage are Rome
can infer that according to Swami Vivekananda, India and Paris. Thus, option (b) is correct.
was self-sufficient in religion, but required material
support. 22. b The word ‘tarnished’ as used in the passage means
destroyed or spoiled. Therefore, option (b) is correct.
14. d The passage states that the missionaries started
denigrating Swamiji when they found that the 23. d The second paragraph of the passage states that the
subscriptions they had so long been receiving from 50,000 French fans gathered at Champs Elysees “to
the people were steadily declining. From this we can tell their team it had been heroic despite the defeat”. In
other words, the fans wanted to convey to their team
infer that Vivekananda’s popularity with the American
that they had given their rivals a tough fight. Therefore,
people caused a drop in the Church’s collections.
option (d) is correct. Option (c) is incorrect owing to
Hence, option (d) is the correct answer.
the word ‘congratulate’. Options (a) and (b) are
incorrect because the passage contains no evidence
15. a According to the passage, Vivekananda preached
to support them.
that, “Truth alone prevails”. Clearly, he did not defend
himself when missionaries and other people conspired 24. b Statement 2 is incorrect because the second
against him because he firmly believed that the truth paragraph states that the faces of the French fans
about his innocence will get revealed anyhow. were mournful “not least because superstar Zidane’s
Therefore, option (a) is the correct answer. final match had been tarnished”. Statement 4 is incorrect
because the last paragraph states that the Italian
16. c The third paragraph states that at the end of the year victory was a “double triumph” because they had two
2003, “the population of Mauritius was over 1.2 million” reasons to celebrate – one, their triumph in this match
and the second paragraph states that the “overall and that they “avenged their Euro 2000 defeat at the
balance of payments is positive”. Both the references hands of French”. This also proves that Italy had once
prove statements 1 and 2 to be true. Statement 3 is not been defeated by France, rendering statement 3
stated or suggested anywhere in the passage. Hence correct. Statement 1 is also correct because the
option (c) is correct. second paragraph clearly states that Zidane was
expelled in the final match.
17. c Refer to the following line in the second paragraph,
“Diversification away from sugar (but not at its 25. a The passage gives equal weightage to the discussion
expense) has been achieved. Textile and garment, of both - the ecstasy felt by the Italians and the mournful
and tourism are the two other leading sectors today.” atmosphere in France - after Italy’s victory over France
This proves that of all the options, ‘sugar’ is most likely in World Cup 2006. Therefore, option (a) is correct.
to have been the leading industry in Mauritius at Options (b) and (c) are inept because each of them
refer to only one country – France and Italy respectively.
sometime in the past. Thus, option (c) is the correct
Option (d) is incorrect because it’s too general.
answer.

CEX-5336/P1BS/17 / Page 2 VA / Exercise - 16


26. d The passage begins with the invention of rock-and- process would have helped India become a global
roll, who all it influenced and was influenced by, and superpower. Option (a) is thus the answer.
finally, its march into the arena of pop music. Therefore,
option (d) is correct. 3. d From the first few lines of the passage it can be clearly
understood that India has done well on the front of
27. c The first paragraph of the passage enlists all except democracy, equity and economy. Hence, option (d) is
Elvis Presley. Therefore option (c) is correct. correct. Option (c) is incorrect because even though
the last lines of the passage make a reference to the
28. c The first paragraph of the passage states that true Indian political leadership, its diplomats and civil
rock-and-roll is a “combination of various elements servants, it cannot be inferred from this that India is
from country and Western, gospel, rhythm and blues, being perceived as an emerged power as a result of
and jazz”. Therefore, option (c) is correct. the efforts of its politicians, diplomats and bureaucrats.
Option (a) is incorrect because there is no link between
29. a Option (a) is false because the passage states that political leaders from the lowest castes and India being
rock and roll was invented in America and not that it considered an emerged power. Option (b) can also be
was invented by Americans. Therefore, option (a) is ruled out because ‘countrymen’ is a very general term
the correct answer. Option (b) can be inferred from when the specific sections of the Indian elite have
the last paragraph of the passage that talks about been mentioned in the last paragraph.
“rock-and-roll’s ascendancy into the world of pop
music”. Option (c) is true because the first sentence 4. a Refer to the last sentence of the third paragraph for
of the last paragraph states that “Teenagers and the the answer.
money they were willing to spend on records provided
an impetus for rock-and-roll”. This means that rock- 5. d Refer to the third paragraph for the answer.
and-roll was popular among the teenagers. Option (d)
is true because the second paragraph mentions that 6. b The passage talks of political upheavals. This means
“Alan Freed began to play rhythm and blues to a largely a situation where things go topsy turvy in the political
non African-American audience”. world. Hence the correct answer is DISRUPTION.

30. b The passage states that the money spent by the 7. c The passage talks of persistent poverty i.e. poverty
teenagers provided the impetus/support/boost to the that is difficult to get rid of. Hence the correct answer
rock-and-roll music. Therefore, option (b) is correct. is UNRELENTING.

Practice Questions 8. a

1. b The last sentence of the first paragraph states that 9. c The passage talks about reinforcing the sentiment i.e.
despite certain problems that India is facing, the country strengthening the sentiment. The opposite of this is
has still managed to obtain a high place in the global WEAKEN which is the correct answer.
economy. If these problems were to be reduced or
eradicated then it can be inferred that it would help 10. b As used in the passage demise means death. Hence
India’s case further. Statements (1) and (2) are a the correct opposite is BIRTH.
reversal of the problems that have been mentioned in
the last sentence of the first paragraph. They are thus 11. b The author says that the government borrowings are
correct. Statement (3) cannot be inferred from the likely to be large for the rest of this year. This does not
passage. And hence the answer is option (b). mean that the government is going to borrow heavily
this year because what is mentioned in the passage
2. a Refer to the first line of the second paragraph. It states is just a speculation. Refer to the sentence “If the
that the Goldman Sachs report predicted that by 2050 objective is … in the SLR reduction”, this sentence
the Indian economy would be the third largest in the clearly tells that SLR reduction is not sufficient to
world. This helped India obtain a stronger image among increase funds for lending.
the global economies. Refer to the words, ‘The process
of admitting India to the global high table can be traced 12. d As per the passage it is the reduction in SLR and not
back to the BRICS report by Goldman Sachs’. This just the SLR that can improve the quality of monetary
indicates that the Goldman Sachs report brought India transmission. Furthermore, it is reduction in SLR and
into the limelight. Option (2) can be ruled out because not SLR per se that boosts liquidity. Hence, option (d)
there isn’t enough information to infer that the HAP is the correct answer.

VA / Exercise - 16 CEX-5336/P1BS/17 / Page 3


13. b The passage tells that the banks have been investing investors.” Option (a) is incorrect as it is too broad in
in SLR securities much more than what was mandated. scope. It does not describe how the market is evolving
This does not mean that they have been investing in and why it’s attracting new entrants. Option (c) is
SLR securities much more than required. Refer to the incorrect as the passage mentions some non- profit
sentence “Bankers have always sought the sanctuary organizations are undergoing this change. Option (d)
of government securities rather than dispense is also incorrect as the author states that MFIs are
commercial loans.” This makes it clear that the bankers beginning to work towards these goals.
always seek the sanctuary of government securities
rather than commercial loans. Hence, option (b) is the 23. b The goal of microfinance has always been and remains
correct answer. alleviating poverty. However, statement II articulates it
better as statement I can mean giving away money as
14. a Refer to the second last sentence of the first charity. Statement III cannot be the goal of microfinance
paragraph. as a whole as it has emerged only recently and even
then not all MFIs aim to pursue such and objective.
15. d Refer to the second paragraph where the answer is
given. 24. d The passage traces the development of microfinance
institutions and how microfinance has come to be
16. c The word has been used in the sense of RESERVE perceived and practiced by various organizations. The
i.e. a protected area. Animals are safe in a sanctuary. passage does not deal with the impact of microfinance
The money of banks is safe in Govt. securities. on poverty, or with the rapid growth and importance
of microfinance.
17. b It is compulsory for banks to invest a percentage of
their money in Govt. securities. 25. a Refer to the last sentence of the first paragraph for
Hence the correct answer is OBLIGED. the answer.

18. a ‘Mandated’ means assigned. 26. c Alleviation of poverty means easing of poverty. Hence
option (c) is the answer.
19. c In the passage the word boost has been used in the
sense of increasing. Hence the correct answer is 27. b In the passage the word transforming has been used
DAMPEN which means to discourage or put a damper in the sense of changing or converting. Hence the
on. correct answer is option (b).

20. b In the passage the word has been used in the sense 28. c ‘Sustainability’ means support.
of putting out flames. Hence the correct answer is
STOKING. 29. b In the passage the word evolving has been used in
the sense of growing or developing. Hence option (b)
21. d In the last paragraph of the passage it is mentioned meaning rotting is the correct answer.
that big consumer retail firms are beginning to emerge
as consumer lenders. From this we can infer that 30. c Venturing means daring or proceeding courageously
lending money is not their traditional line of business. in unknown areas. Hence option (c) is the correct
The author says that out of these “a few are answer.
venturing” into microfinance. Therefore out of the
given options a big retail chain is least likely to be an 31. c The passage starts with a criticism of the NACs
MFI. NGOs and public sector banks have always been proposal and then moves on, in the next few
involved with microfinance. Option (c) can be inferred paragraphs, to discuss the possible options in
from the sentence “they are being joined in the improvement. Option (c) is the answer. Option (a) is
microfinance marketplace by consumer finance incorrect because though the author extols the virtue
companies, like GE Finance and Citi Finance”. Option of a universal PDS, it is one of the options to better the
(d) is the correct answer. situation. Option (b) is incorrect because the passage
is also concerned about those families that have not
22. b The answer can be found in the last two lines of the been taken into account by the poverty line. Option (d)
first paragraph.” Yunus also demonstrated that loans is incorrect because the passage by itself cannot
made to the working poor, if properly structured, had overturn the NAC’s proposals nor does it seek to do
very high repayment rates. His work caught the so.
attention of both social engineers and profit-seeking

CEX-5336/P1BS/17 / Page 4 VA / Exercise - 16


32. d Option (a) has been provided as a criticism in the first 42. d Statement 1 can be ruled out because there is no
paragraph. It is clear that the NAC’s proposals have information in the passage to reach this inference.
been limiting. Option (b) can also be inferred from the Statement 2 is correct and mentioned in the last line of
fourth line of the first paragraph. Option (c) can be the third paragraph – the line indicates that the
inferred from the first sentence of the third paragraph. monsoon has also added to the increase in hunger.
Option (d) is incorrect because there is no mention in This situation is linked to India’s worsening malnutrition
the passage that food security policies also benefit rates and hence finds its place in the same paragraph.
the rich. And of this situation, the author also does not Statement 3 is correct as it is mentioned in the first
offer any criticism. paragraph. Statement 4 is correct and can be inferred
from the fifth paragraph. Option (d) is the answer.
33. b Refer to the last paragraph. It indicates that the quota
would be less than that actually set. Statement 1 can 43. b The argument that the Prime Minister puts forth is that
thus be inferred. Statement 2 can be inferred from the distributing free food will destroy any incentives the
points made in the paragraph. Statement 3 is farmers will have to produce. This indicates an
speculation because the passage doesn’t help interference with the market economy and hence
establish that the only reason the rich will not get grain option (b) is the answer. Option (a) can be ruled out
from the PDS is due to the low quality of grain offered. because there is no hint that farmers will bear the
Hence option (b) is the answer. cost of distributing free grain. Option (c) cannot be
inferred from the information in the passage. Option
34. d Option (d) is the correct answer. In the sentence “The (d) is incorrect because it doesn’t help to understand
process of targeting ... that have been included.” given why farmers would be demotivated to produce if the
in the third paragraph the author says that using BPL government decides to distribute grain free of cost.
limits the availability of food grains to only some families
and others are left out. This is a hindrance to the goal 44. a Refer to the fourth paragraph for the answer.
of universalizing. Also, from last paragraph it can be
inferred that the system does not let us determine the 45. d As per the second paragraph, India’s record is worse
accurate amount of food grains required, as some than Sub-Saharan Africa. Also, as per the fifth
families who are counted under the current system paragraph, 37% of India’s population lives below the
may not take the food grains. poverty line.

35. b Refer to the first sentence of the last paragraph for 46. b To pledge means to swear.
the answer.
47. c Global means worldwide.
36. b Damaged is the right answer.
48. d ‘Imminent’ means looming.
37. c Feasible means practicable or that which can be done.
49. b Ensuring means following. Hence the correct answer
38. b ‘Discriminatory’ means inequitable. is preceding.

39. b Retrograde means goring backward. Hence 50. c Extolled is the correct answer.
Progressive is the correct answer.
51. d The answer is (d) and this has been given in the first
40. d Malnourished means not receiving enough paragraph. Combative is a synonym of competitive
nourishment. Hence well fed is the correct answer. but with a negative connotation and it does not fit into
the context of the paragraph. Democratic is an
41. a The passage mentions that India has dismally high antonym for authoritarian.
malnutrition rates. The last line of the fourth paragraph
also mentions the inference that one can draw from 52. c The author discusses the two sides of the bank -
these high rates – ‘The reality is that a country external pressures and the internal environment - in
desperate to take its place at the world’s top table is the context of reforms. However, it is clearly inferred
unwilling to commit to feeding its own population.’ from paragraph 3, 4, 5 and 6 that she feels that bank
Statement 1 is correct. Statement 2 can be ruled out reform has not taken place because of its strong internal
because there is no certainty that there will be a culture. The last paragraph clinches the answer as
positive change because of the food security bill. Option option (c). Option (a) is incorrect as this was part of
(a) is the correct answer. the failed efforts for reforms in the 1990s. Option (b)

VA / Exercise - 16 CEX-5336/P1BS/17 / Page 5


is incorrect as indicated from the first line of paragraph 54. a The author’s tone is incisive or analytical.
3. The external reforms are not seen as important
when compared with the failed efforts of the bank in 55. d Both the statements are incorrect. Refer to the third
changing its internal environment. Option (d) is an end paragraph for the answers.
objective for the bank but it does not reflect the changes
that need to be made thereby not answering the 56. d
question. Option (c) is the correct answer.
57. a
53. b The question asks for the statement which most
reflects Weaver's attitude. In the first and last 58. b
paragraphs of the passage, Weaver agrees that the
World Bank is a hypocritical organization she also 59. d
moves away from this to look at how it can move
away from this. Option (a) does not answer this 60. c
question entirely and instead criticizes the bank. Option
(b) is the best answer in the context of the entire
passage. Option (c) has never been stated nor implied
by Weaver, it is what other critics have said about the
Bank. Option (d) has to do with the failed reforms of
the 1990s and not the time period the author is writing
in.

CEX-5336/P1BS/17 / Page 6 VA / Exercise - 16


Verbal Ability – 17 P-1(BS)

Types of Questions and Level – 2 Questions


Answers and Explanations
1 d 2 d 3 b 4 a 5 d 6 b 7 d 8 a 9 d 10 c
11 a 12 c 13 a 14 d 15 d 16 c 17 b 18 d 19 b 20 d
21 c 22 b 23 c 24 a 25 d 26 b 27 d 28 c 29 c 30 b
Practice Questions
1 c 2 b 3 c 4 d 5 a 6 b 7 c 8 c 9 b 10 c
11 b 12 b 13 b 14 c 15 a 16 b 17 c 18 a 19 b 20 c
21 a 22 c 23 b 24 a 25 a 26 b 27 c 28 b 29 c 30 a
31 b 32 d 33 b 34 a 35 d 36 b 37 c 38 b 39 c 40 b
41 c 42 b 43 d 44 d 45 d 46 b 47 c 48 b 49 c 50 b
51 c 52 c 53 a 54 c 55 b 56 d 57 b 58 d 59 d 60 b

1. d Option (d) can be inferred from the penultimate 4. a The author agrees with both statements A and B. Refer
paragraph which says that “The way in which parents to the first line of the passage wherein the author
view their bodies impacts their children’s attitudes.” says, “Beauty is a valuable commodity...” Hence, it
Hence, it can be clearly understood that children can can be understood that in today’s society, beauty is
be influenced by their parents. Option (a) cannot be given a lot of significance. In the third paragraph, the
inferred from the passage. The first paragraph says author says that 94 percent of the women are
that beauty pageant contestants have the potential to judgmental about their looks. This means that there
make headlines and these girls are beautiful and most are only a few women who are satisfied with their
often thin too. It might not be true the other way round. looks. Hence, option (a) is the answer. Refer to the
Option (b) is beyond the scope of the passage since line ‘The media is often portrayed as the bogeyman in
we do not know whether anti-obesity programme the body-image debate’. Bogeyman is a mythical
helped in reducing obesity or not. Option (c) cannot be creature adopted by parents to scare little children.
inferred from the passage since it is just an opinion of ‘Lilliputian’ is used to describe little people. Therefore,
Jane Caro. We do not know whether the author agrees we can say that statement C is not in line with what
with the statement or not. the author agrees to.

2. d The opening paragraph tells us that all Miss India’s and 5. d ‘Odious’ means arousing or deserving hatred. Hence,
Miss World’s make headlines, but Chloe Marshall its synonym is disgusting, rendering option (d) the
gathered more attention because she was a “full correct answer.
figured” 2008 Miss England runner up. She created a
“stop-the-press” moment since large women are not 6. b “To be on tenterhooks” means in a state of uneasiness,
‘generally’ considered ‘fairest of them all’. However, it strain, or suspense. Therefore, option (b) is correct.
cannot be inferred that she was not considered
“fairest of them all”. This rules out option (b) and makes 7. d Options (a) and (b) are incorrect because the passage
option (d) the correct answer. Option (a) is incorrect states that a rich man can buy many things but not
as ‘not unlike’ means that she [Chloe] was like the happiness while a poor man, if contented, can be
others. happy without money. Therefore, option (d) is correct.
Option (c) is incorrect because the cause of
3. b The entire passage talks about women wanting to be unhappiness is not always money.
slim and associating slim with being beautiful. Refer to
the last line, “...the notion that fat is bad, slim is good...”
So option (b) is the correct answer.

VA / Exercise - 17 CEX-5337/P1BS/17 / Page 1


8. a The passage states that a rich man who does not 17. b Refer to the first sentence of the passage where the
“enjoy good health”, or whose “only son has taken to author uses the word 'therefore' to signify why images
evil ways” or who has a “lot of business worries” can play such a pivotal role in our lives today. So, option
become unhappy. Therefore, option (a) is correct. (b) is the correct answer.
9. d The passage states that there is a difference between 18. d In the second paragraph, the author says that a
having money and having contentment. While the latter photographic image can be staged and gives the
may not give happiness in the absence of the former, example of Bayard to support his statement. So, option
the former can make one happy without the latter. (d) can be said to be a limitation of a photographic
Therefore, option (d) is correct. Option (c) is incorrect image. Option (a) is incorrect since it is a limitation of
because the passage talks about ‘happiness’. photography and not a photographic image. Options
(b) and (c) are not limitations.
10. c The passage states that although a man may be poor,
he can be happy if he is contented. Therefore, option 19. b Refer to the last sentence where the author mentions
(c) is correct. Options (a), (b) and (d) are incorrect that Bayard created an alternative reality or alter ego
owing to the words ‘only’, ‘never’ and ‘always’ through his photograph. So, option (b) is correct. Option
respectively. (a) is wrong because the author mentions in the
second paragraph that in a painting, an artist may give
11. a At the beginning of the passage, the author states shapes to his fancies or draw a subject that is present.
that '… while the nuisance that the non-smokers cause Thus, it cannot be inferred that a painting is always a
the smokers is spiritual'. Therefore, option (a) is correct. reflection of the artist's imagination. Option (c) can be
All other options are irrelevant because the passage negated from the first sentence of the second
gives no information about them. paragraph. Option (d) is incorrect. The option means
that people intentionally express emotions when being
12. c The author mentions that a non smoker is akin to a man photographed. However, this is not true. Refer to the
without weakness and likely to be completely rational. line "People have learnt to express emotions…" What
Moreover, in his opinion, non smokers are "apt to be the sentence means is that the person taking the
correct and unemotional" while a smoker is more likely photograph expresses his emotions, etc. through the
to be genial, open hearted and unaffected. Thus the pictures he takes. In other words, the pictures reflect
author feels that a smoker's morality is higher than the emotions, ideas and concepts of the person taking
that of a non smoker. the picture.

13. a Option (b) cannot be the answer since treachery is 20. d 'Chimera' refers to something that exists only in the
not mentioned anywhere in the passage. Option (c) is imagination and is not possible in reality. So, option (d)
incorrect because disdain of such people is not talked is the answer.
about in the passage. Similarly, prejudice is not 21. c Refer to the last sentence of the first paragraph.
mentioned in the passage, so it is eliminated. Option "…issues concerning economic deprivation…
(a) is the correct answer because in the first foundation of any reinvention of the Left." So, option
paragraph the author says “his (non smoker’s) habits (c) is the correct answer.
are too regular, his existence too mechanical and his
head always maintains its supremacy over his heart.” 22. b Refer to the first sentence of the second paragraph. It
clearly states that the three have moved towards the
14. d Read the last four lines of the first paragraph. "Much international agenda for social reconstruction within
as I like reasonable persons, I hate completely rational which socialism must be put forward as a programme
beings. For that reason, I am always scared and ill at to salvage a world from inequality, abuse of power
ease when I enter a house in which there are no and the hegemony of the US. So, option (b) is the
ashtrays." Therefore, option (d) is correct. correct answer.

15. d None of the options can be drawn from the quoted 23. c Refer to the first sentence of the third paragraph. It
lines. Hence, the correct answer is option (d). Option clearly says that the measures have been taken to
(b) is incorrect owing to the word ‘all’. check the erratic plays or unpredictable moves of
market forces. So, option (c) is the correct answer.
16. c The author begins the passage by citing the importance 24. a Refer to the last sentence of the third paragraph.
of photographs in today's life. In the second paragraph, "…Chavez has been fighting for regional integration
the author mentions that a photograph may not always and a society that bases itself on the ideology of the
represent reality. Hence, photographs are a fallacy of new South American Left". So, option (a) is the correct
reality. So, option (c) is correct. answer.

CEX-5337/P1BS/17 / Page 2 VA / Exercise - 17


25. d 'Hegemony' means influence or control over another 2. b The passage mentions that “The political parties to
country, a group of people, etc. which the AITUC and the CITU are affiliated had en-
tered into a thoroughly opportunistic alliance with
26. b The passage considers the situations in which small Jayalalithaa’s AIADMK in the state assembly elections
minority enterprises are likely to find themselves in last year and wanted to keep the alliance going”. And
when they attempt to expand their businesses in a since it is mentioned that the AIADMK government was
market dominated by big companies run by Whites. “instrumental in the defeat of the strike”, it can be
This makes option (b) correct. inferred that had the AITUC taken a stand contrary to
that of the government, their alliance could have been
27. d The passage essentially talks of the situations in which jeoparalized. Therefore option (b) is the correct an-
small businesses might face financial difficulties when swer.
they try to expand themselves. This makes option (d)
correct. 3. c The author mentions Maruti Suzuki and NLC as exam-
ples of companies where the plight of workers is
28. c The passage states, ‘But civil rights groups and minority such as has been described by the author in the pas-
business owners have complained to Congress about sage. The companies have not be mentioned specifi-
minorities being set up as “fronts” with White backing, cally to highlight the situation of only their workers.
rather than being accepted as full partners in legitimate Option (a) is negative. The author merely says that
joint ventures.’ This implies that such small businesses given such workers’ conditions in these two compa-
are only nominal partners of the White firms, and do nies “one can only imagine what their predicament is
not actually take part in the business. This makes option in labour-intensive manufacturing”. He is not making
(c) correct. any comparison. Rather it is more of an assertion.
Thus, option (c) is the correct answer.
29. c The passage states, ‘Even in the best of
circumstances, fierce competition from larger, more 4. d Refer to the second and last paragraphs.
established companies makes it difficult for small
concerns to broaden their customer bases: when such 5. a Refer to the first paragraph for the answer. As per the
firms have nearly guaranteed orders from a single second paragraph, statement 2 is incorrect.
corporate benefactor, they may truly have to struggle
against complacency arising from their current 6. b Quandary is the correct answer.
success.’ This clearly implies that small concerns
should try to expand their customer base. 7. c In the context of the passage Heavy Hand means
Insensitive.
30. b The passage states, ‘Indeed, some federal and local
agencies have gone so far as to set specific 8. c ‘Affiliated’ means attached.
percentage goals for apportioning parts of public
works contracts to minority enterprises.’ This makes it 9. b The opposite of parity is disparity.
clear that specific goals have been set by federal and
local agencies even without the law requiring them to 10. c The opposite of Faltering is Firm.
be so specific.
11. b Options (a), (c) and (d) are all mentioned in the pas-
Practice Questions sage. Regarding the role of NGO’s, the passage says
that “reaching significant scale will require moving
1. c In order to arrive at the answer, the first two para- beyond the NGOs model”. Option (b) is contrary to this
graphs can be referred to. Option (a) can be inferred point, and is therefore the correct answer.
from the first sentence of the passage – ‘authoritari-
anism in the workplace, unjust victimisation of work- 12. b Refer to the third paragraph in which the author dis-
ers who are at the forefront of resistance’. The pas- cusses the scope of financial services, stating also
sage mentions “the lack of a fighting spirit on the part that the scope needs to be further widened. Option
of the central trade unions affiliated to the mainstream (a) can be inferred from the penultimate line of the
political parties”. Thus, option (b) can be inferred. paragraph. Option (b) is incorrect and cannot be in-
Option (c) is incorrect because the passage speaks ferred from the information in the passage. There is
about unjust victimization of workers who are at the no suggestion that financial services for the poor are
‘forefront of resistance’. There is no information on also linked to infrastructure development. Option (c)
consistent victimization of employees. Option (d) can can be inferred from the third line of the paragraph.
be inferred from the second line of the first paragraph Option (d) can be inferred from the last line of the
and the last lines of the second paragraph. paragraph. Option (b) is thus the answer.

VA / Exercise - 17 CEX-5337/P1BS/17 / Page 3


13. b Statement 1 can be ruled out because there is no “Since they deliver…inefficient government system.”
suggestion of introducing FDI. Statement 2 is clearly Option (c) is incorrect as it cannot be inferred from the
mentioned in the passage. Thus, option (b) is the cor- passage that the regulatory mechanism will be brought
rect answer. in through privatization. Option (c) is the answer.

14. c Statements 1 and 2 can both be inferred from the 23. b The author of the passage is in favour of for-profits
passage. Consider the following “ impact of micro- and this can be determined from his opening argu-
finance is not only on poverty reduction but also on ments. However, he is also ready to look at both sides
several other MDGs’’. Statement 1 can thus be in- of the argument - for-profit and not-for-profit (govern-
ferred. Consider ‘other MDGs such as achieving uni- ment run schools). And in the course of the passage
versal education, promoting gender equality and wom- the author lists the pros and cons for both sides.
en’s empowerment’. All these MDGs point to areas of Refer to the last line of the first paragraph in which the
social development and hence statement 2 can also author hints at the system that he foresees - one in
be inferred. Therefore, option (c) is the correct an- which both for-profit and government players exist
swer. but with a transparent regulatory mechanism. Option
(b) is the answer. Option (a) is incorrect as not once
15. a Refer to the first sentence of the second paragraph does the passage attempt to state that the govern-
for the answer. The first sentence of the third ment should not be allowed a hand in the education
paragraph nullifies the second statement. system. Option (c) is incorrect as it goes against the
information in the passage - the author calls option (c)
16. b The word overarching has been used in the passage an extreme position hinting that this is not the best
to suggest most important. Hence option (b) is the alternative. Refer to the second line of the penultimate
right answer. paragraph. Option (d) is not mentioned in the pas-
sage.
17. c Foster has been used in the passage in the sense of
promoting. Hence option (c). 24. a Refer to the second paragraph for the answer. The
last sentence of the passage nullifies the second
18. a ‘Status quo’ refers to the existing state of affairs. statement.

19. b Option (b) is the correct answer. 25. a Refer to the fourth paragraph for the answer. The
second statement is out of scope.
20. c Option (c) is the correct answer.
26. b Flagrantly is a synonym for blatantly.
21. a The first paragraph presents the author’s argument
on why for-profits should be included in education. 27. c Self Evident is a synonym for axiomatic.
The next few paragraphs bring in the controversies
on both sides of the debate for including for-profits. 28. b The correct antonym is opponent.
Option (a) is the answer. Option (b) can be ruled out
as the author clearly states in the last line of the sec- 29. c The correct answer is impractical.
ond paragraph that the schools that should be non-
profit are clearly not so. Option (b) also does not fall 30. a The opposite of ‘significant’ is trite.
within the primary purpose of the passage. Option (c)
is incorrect as primary school education does not en- 31. b Refer to the first paragraph. Option (1) can be ruled
ter the debate. The stress is on education and for- out because the eighth line provides information that
profits. Option (d) is also incorrect because other sec- the developing countries do not have the requisite
tors have been mentioned only in the last line of the safety measures for women workers. This however
passage. The comparison or reference to other sec- cannot be generalised for the rest of the world. Op-
tors does not find itself in any part of the passage tion (2) is correct because the success of workplace
otherwise and one can assume it is not an important provisions is said to depend on a favourable environ-
issue for the author. ment/scenario (prospects) aided by a conducive po-
litical climate and a proactive judiciary. Option (b) is the
22. c Option (a) can be inferred from fifth line of the first answer.
paragraph - “For-profits, in my view, bring in passion,
expertise and innovation.” Option (b) can be inferred
from the last line of the first paragraph. Option (d) can
be inferred from the last line of the third paragraph -

CEX-5337/P1BS/17 / Page 4 VA / Exercise - 17


32. d Refer to the first paragraph. The author says that he deficit and rising inflation and subsequently to dis-
is not surprised that stipulations on minimum wages cuss matters of economic growth and globalisation.
are missing in the Convention. He then comments, in The subsequent paragraphs in the passage are all
the last line of the paragraph, that society does not written in an effort to present a platform that helps
recognise domestic work as an important economic establish the rationale for the government to tackle the
activity. This is all the information that the passage has rising fiscal deficit. Option (c) is the answer. Option
that can be linked to the lack of provision on minimum (a) is incorrect as the people of the country and their
wage. Option (a) cannot be inferred from the pas- importance is not discussed. The common man has
sage as there is no reference to it. Option (b) is also not been mentioned in the passage. Hence option (d)
incorrect because there is indication of this in the can also be ruled out. Option (b) is also incorrect
passage - there is no link between the establishment because the negative impact of not controlling the
of minimum wages and the implementation of the new fiscal deficit on outside investment is not discussed in
provisions. Option (c) may have been a logical argu- the passage.
ment if there had been more information provided or if
it were known as to why the convention can set 42. b Refer to the sentence - “Raising diesel prices
international provisions for workers’ rights (their work by…deficit under control.” given in the first paragraph.
hours, weekly rest and collective bargaining) but not It is clear in this statement that the increase in the
for minimum wage. If such an argument were pro- diesel subsidy bill has a direct link with the increasing
vided (or even a hint given) then option (c) would fiscal deficit. Raising diesel prices will lead to a reduc-
have helped explain why this issue has been left out. tion in the diesel subsidy bill and will hence help con-
Option (d) is thus the answer. trol fiscal deficit. Option (b) is the answer. Option (a)
is incorrect because it fails to bring in the link between
33. b Refer to the last paragraph. The author states the diesel prices and fiscal deficit. Option (c) is incorrect
benefits of regulating domestic work and streamlining because the passage does not suggest that raising
standards. Option (a) can be ruled out as there is no diesel prices is the only way to control the fiscal defi-
mention of this in the last paragraph. Option (b) is the cit - the passage only focuses on this aspect but it
answer and can be inferred from the sixth line ‘realis- cannot be inferred whether this is the only way to
ing more fully the right to compulsory basic education control fiscal deficit or not. Option (d) is incorrect be-
for children.’ and the eight line ‘enormously augment cause it is vague and does not establish the link be-
the goal of expanding basic education’. Option (c) is tween diesel prices and fiscal deficit.
incorrect as the author mentions that regulating do-
mestic work will help complement the governments’ 43. d Refer to the last paragraph. Assumption 1 is not valid
efforts to combat crimes against women and children. and cannot be inferred from the passage as the pas-
Option (d) is incorrect as it is too general and would sage provides no information which suggests that
only be considered if the right to education is realised globalisation ensures that economies have less au-
for children. tonomy and thus less say in matters of their economic
growth. Assumption 2 is incorrect as the sentence
34. a Refer to the first paragraph for the answer. “To illustrate, Indian farmers...markets by QE3.” given
in the last paragraph only states that Indian farmers
35. d Refer to the first paragraph for the answer. Both the get affected by the WTO negotiations. However, there
statements are incorrect as per the passage. isn’t enough information to assume that the WTO has
adversely affected the agricultural growth of the coun-
36. b Specification is the correct answer. try. Option (d) is the correct answer.

37. c Odd is a synonym for peculiar. 44. d Refer to the third paragraph in the passage that states
that with a high inflation rate one cannot expect the
38. b In the context of the passage the correct answer is RBI to reduce the interest rates. It can be assumed
legalize. that interest rates will either remain as they are or rise
further. The rest of the paragraph indicates that inter-
39. c Option (c) is the correct answer. est rates are rising - policy rates are high and the
home loan EMIs are rising. Option (a) can be ruled out
40. b because there is no indication in the passage about
funding of government requirements through market
41. c The passage starts by discussing the rationale for borrowings. Option (b) goes against the discussion in
raising diesel prices in order to control the rising fiscal the third paragraph. The author suggests an impact
deficit. It then goes on to discuss how rising food on the fiscal deficit not only in this paragraph but in the
prices, linked with the increase in diesel prices and entire passage. Option (c) is incorrect because the
rising inflation, can be neutralised. The passage then paragraph suggests a link between interest rates and
moves on to other issues linked with a high fiscal fiscal deficit. Option (d) is thus the answer.

VA / Exercise - 17 CEX-5337/P1BS/17 / Page 5


45. d Both the statements are incorrect. 53. a The second last paragraph discusses about community
system that has been examined in historical context in
46. b Viewpoint is a synonym. terms of material means of production in the last
paragraph. The next paragraph is likely to further
47. c Inflation means a general increase in prices etc. explain the evoluation of the distinctive community
system and development of social oragnizations in
48. b The opposite of Hike is Cut. Goa. Option (d) can be ruled out. Option (c) has been
mentioned in the first paragraph but there is not
49. c The opposite of discretionary is compulsory. indication throughout the passage that the author will
dwell on the benchmarks. The author never discusses
50. b The opposite of ‘coercive’ is beneficial. option (b) in the passage - he only mentions that the
Goan economy is among one of the best performing in
the country. The answer is option (a).
51. c The second paragraph speaks about the larger context
through which Goa derives its strength - value as 54. c Refer to the last paragraph for the answer.
derived from humanism. This is best explained by option
(c). Option (d) comes close but this has been given as 55. b Refer to the first and last paragraphs for the answer.
a wider context due to which the Goan economy is The second statement is true.
among the best performing in the country. Option (b) is
on of Goa's strengths. Option (a) is incorrect as it has 56. d 57. b 58. d
not been given in context of Goa's strengths.
59. d 60. b
52. c The author does bring in the narrative style in the
passage except that this is used in order to analyse
the Goan economy. Since the central idea is to
understand the Goan economy (in an attempt to
understand, the author even takes the reader into the
past) the author uses information and even history to
fulfill his purpose. Option (b) is ruled out because it is
a tone and not a style - also it may seem like the author
is praising the economy in the first two paragraphs
but again this is done keeping the primary purpose in
mind. The author does bring in information but he does
not only stick to giving out information. The author also
has opinions that he shares with the readers. The
correct answer is option (c).

CEX-5337/P1BS/17 / Page 6 VA / Exercise - 17


Verbal Ability – 18 P-1(BS)

Reading Comprehension – Revision


Answers and Explanations
1 b 2 b 3 c 4 c 5 c 6 c 7 c 8 b 9 c 10 d
11 d 12 a 13 d 14 a 15 c 16 b 17 d 18 d 19 c 20 a
21 d 22 d 23 d 24 b 25 b 26 a 27 a 28 d 29 d 30 d

Practice Questions
1 c 2 a 3 b 4 d 5 b 6 d 7 d 8 c 9 d 10 d
11 d 12 c 13 b 14 d 15 d 16 b 17 c 18 b 19 c 20 d
21 b 22 d 23 b 24 c 25 d 26 b 27 c 28 d 29 b 30 c
31 c 32 a 33 a 34 a 35 b 36 b 37 c 38 b 39 b 40 c
41 a 42 c 43 d 44 d 45 d 46 c 47 d 48 c 49 b 50 c
51 a 52 d 53 b 54 d 55 b 56 d 57 a 58 c 59 d 60 b

1. b The whole paragraph focuses on the eradication of been eradicated completely such that its vaccinations
smallpox such that it has ceased to be a threat to have stopped worldwide (b).
mankind today. Therefore, option (b) is correct. Option
(a) is incorrect because the passage only deals with 5. c Refer to the following statement in second paragraph,
WHO’s role in eradicating smallpox not WHO in general. “…each smallpox victim was sought out…and treated.”
Option (c) is limited because the passage focuses on This renders option (c) correct. Option (a) is incorrect
the eradication of the disease but not about the causes, because by “smallpox is no longer a threat”, we cannot
symptoms or treatment of the disease. Option (d) is infer that new cases have not been reported ‘this
incorrect because the passage does not talk about year’; even if no new cases were reported last year,
any disease except smallpox. the threat would still not exist. Option (b) is incorrect
because the passage states, “similar projects for
2. b Refer to the following sentence in the first paragraph, malaria and yellow fever had failed.” Option (d) is
“The goal was to eradicate the disease in one decade.” incorrect because the fact that there is a need to
Therefore, option (b) is correct. Options (a) is incorrect isolate smallpox patients from the uninfected ones to
owing to the word ‘decrease’. Option (c) is limited break the chain of further spread of the disease,
because providing vaccinations is only a way to realize proves that smallpox is communicable.
the real goal of eradicating smallpox. Option (d) is not
referred to in the passage. 6. c Option (c) is correct because the passage states that
these technical jargon have the “precision of a
3. c The second paragraph of the passage states, “The mathematical formula”. Option (a) is incorrect because
strategy was not only to provide mass vaccinations according to the passage, technical words may
but also to isolate patients with active small pox…” eventually become a part of common speech but to
Therefore, option (c) is correct. Options (a), (b) and begin with they are only used by “devotees of the
(d) are limited. particular art or science”. Option (b) can be negated
by the fact that “the technical vocabulary is very old”.
4. c The passage states that “similar projects for malaria Option (d) is incorrect because the passage nowhere
and yellow fever had failed”, therefore, option (a) is suggests it.
incorrect with respect to smallpox and so it is the
correct answer. All other options are true with regard 7. c Since the passage deals with technical vocabulary in
to smallpox because the passage dose state that language and the role it plays, it is likely to have been
smallpox was a threat to humanity (d), therefore, WHO written by a linguist. Therefore, option (c) is correct.
organized a campaign against it (c) and now, it has

VA / Exercise - 18 CEX-5338/P1BS/17 / Page 1


8. b The passage primarily deals with the incorporation of a national best-seller that helped bring about
technical terminology and its role. Therefore, option congressional investigations into the corrupting
(b) is correct. influence of comic books...” On basis of this, we can
validate option (d). Being insufficient to highlight the
9. c Option (c) is correct because the second paragraph actual message, option (a) is negated. Moreover, it
states, “An average man now uses these in his own talks about lack of literary merit which is not supported
vocabulary.” Options (a) and (b) are incorrect owing by the passage. Option (b) is incorrect in saying that
to the words ‘interchange’ and ‘always’ respectively. comic books introduced crime, violence, and drug
abuse to nation’s youth. It cannot be ascertained that
Option (d) is not given in the passage.
these were introduced by comic books. Option (c) is
too generic in nature and is negated.
10. d Option (d) is correct because it is not a role owing to
the phrase ‘very widely used’. All other options are
14. a Option (a) is true about Silver Age. It can be validated
roles played by technical jargon because the passage from the last lines of the last paragraph, “The comics
states that technical jargon “are necessary in technical business did not recover until the Marvel revolution of
discussion of any kind”, “save time, for it is much more the early 1960s ushered in the Silver Age.” Option (b)
economical to name a process than to describe it” and is incorrect as the passage does not hint/comment
secures “greater exactness in nomenclature”. about any differences or similarities between the comic
books of the two Ages. Option (c) and (d) are also
11. d Option (a) is negated as it cannot be evidenced from incorrect as it is not mentioned in the passage that
the passage. In the passage the author has not Comics Code Authority regulations could or could not
criticized anyone. Option (b) talks about comparing affect the comic books of Silver Age.
and contrasting the Golden Age with the Silver Age of
comic books, however, the passage nowhere 15. c In the second paragraph, we read, “These darker
stresses or discusses any such comparison or comic books aroused the anger of child psychologist
contrast. Option (c) is out of context as it talks about Fredric Wertham, who believed that comic books were
criticizing comic books for their corrupting influence, leading the nation’s youth into...” Thus we infer that
which is mentioned in the passage but not targeted/ Wertham’s efforts were an angry response to a trend
focused on. The passage begins by letting its readers in the subject matter of the comic book industry. Being
irrelevant and untrue, options (a) and (d) are easily
know that the Golden Age of comic books began with
negated. Option (b) is incorrect since “too far” is
the introduction of Superman in 1938, but there is less
subjective and not addressed in the passage.
agreement about when exactly the Golden Age ended.
This is followed by a discussion on how the end of 16. b The second paragraph begins by passing the judgment
the Golden Age of comic books was brought in. It is that Shagun TV can itself seem obsessed. It further
this discussion, on which the passage is primarily gives examples justifying this claim. Thus, the correct
aimed. Hence, the answer is (d). answer is option (b).

12. a Refer to the lines, “The creators of superhero comic 17. d The author talks about the new TV channel and its
books, not wanting to be left behind, responded by functions. But at certain points, like the second
matching their protagonists against darker criminals in paragraph, the author passes judgment and calls the
more violent encounters.” EC comics met the market channel ‘obsessed’. Thus, the correct answer is option
demand by providing the readers with more horror (d).
and adult content in their comic books. Responding to
the competitive pressure from horror comic books, 18. d The author, in the fifth paragraph, gives data related to
the creators of superhero comic books increased the the Indian wedding market in order to give an idea of
amount of violence in their stories. Option (a) mentions the market Shagun TV is going to tap. Hence, the
the same. Option (b) and (d) are factually incorrect, correct answer is option (d).
option (c) is inappropriate being a very flimsy
19. c In the fifth paragraph, the author mentions that the
conjecture.
wedding market is expected to grow 25 to 30 percent
annually. He gives a range within which he expects
13. d The clue to answering this question lies in carefully
the market to grow. One cannot say with certainty
reading the following lines of the second paragraph
that the market will grow at an increasing rate. The
of the passage, “....child psychologist Fredric Wertham,
passage is silent with respect to whether the growth
who believed that comic books were leading the
rate will increase every year. Hence, the author’s
nation’s youth into crime, violence, and drug abuse.
opinion on it cannot be ascertained. Option (c) is thus
Wertham’s book, The Seduction of the Innocent, was
the correct answer.

CEX-5338/P1BS/17 / Page 2 VA / Exercise - 18


20. a The last paragraph states that the channel will break Practice Questions
privacy taboos by bringing on couples to share private
details of their relationships. Hence, option (a) is the 1. c In the sentence “There is a technological...of big retail
answer. chains.” the author highlights that India has played an
important role in giving the retail giants a competitive
21. d There is no mention about the cost of living in space. edge. Option (c) is the answer. Options (a) and (b)
So option (a) is eliminated (b) is directly stated from are incorrect because they place importance on the
the passage. (d) can be inferred as the author is interests of the local shopkeepers whereas the au-
talking about the negative effects of outer space on thor does not highlight this in the passage. Option (d)
the human body such as bone mass loss due to disuse is incorrect because it goes against the information in
the passage and the stand that the author takes in the
of the bones. So (d) can be inferred from the passage.
first paragraph.
22. d As all the three are stated in the passage so option (d) 2. a Statement 1 can be inferred from the last line of the
which says all of these are correct. second paragraph - “What sets the real…keep their
competitive edge.” Statement 2 can be inferred from
23. d The author is talking of gravitational pull through out the sentence given in the third paragraph of the pas-
the passage. Therefore the crux of the passage is to sage - “Data analytics and customer relations…choose
highlight the importance of gravitational pull. Hence discount strategies.” Statement 3 cannot be inferred
option (d) is the answer. as there is no reference to the maintenance of large
inventories leading to the reduction in cost for con-
24. b Answer is option (b) as it is stated in the passage sumers. Hence, option (a) is the answer.
(1st para).
3. b In the last four paragraphs, the author presents ex-
amples of how Indian firms and professionals have,
25. b The author is telling us something about the gravitational
through technology, helped in the growth of the for-
pull and how it may affect the human body. Therefore eign retail giants. Refer to the last line of the first
the tone of the author is informational. Hence answer paragraph - “There is a technological dimension…edge
is option (b). of big retail chains.” It helps infer that option (b) is the
answer. Option (a) is incorrect because it does not
26. a The passage is primarily concerned with telling us point out the role of Indian technology firms and pro-
about article 32. It does talk about the Supreme Court fessionals. Option (c) is incorrect because it is a gen-
and Fundamental Rights, but the primary emphasis is eral statement while the author only presents infor-
on article 32. Hence, option (a) is the correct choice. mation on Indian IT firms and does not link this with the
growth of retail giants. Option (d) is incorrect because
27. a Refer to the line, "An application made under Article it leaves out the role of the Indian IT firms.
32...technical grounds." This line does not suggest 4. d Both the statements are incorrect. Refer to the fifth
that an application made under Article 32 of the paragraph.
Constitution before the Supreme Court, cannot be
refused under any circumstances as the given line 5. b Refer to the second paragraph for the answer.
only mentions technical grounds. All the other options Statement two is correct as per the passage.
can be deduced from the passage. 6. d 7. d 8. c
9. d 10. d
28. d A and B are mentioned in the passage. It is mentioned
that article 32 refers to the “Constitutional Remedy” 11. d Refer to the third paragraph. The passage discusses
for the enforcement of the Fundamental Rights and is the issue of tax evasion and states that investments
in itself a Fundamental Right. Hence, option (d) is are routed into India through countries like Mauritius
correct. so that investors can avoid paying taxes. This is done
by setting up a letter-box company in Mauritius so that
29. d The author has stated various facts and figures in a investors can avoid paying tax in India. It also indi-
factual/objective tone. He neither expresses his cates that Mauritius is a tax friendly country (havens)
opinions nor analyzes the issue. Thus, option (d) is but not necessarily tax free. Thus, option (b) can be
the correct choice. ruled out and option (d) is the answer. Option (c) can
be ruled out because nothing has been mentioned
30. d Violation means a failure to uphold the requirements about “obsolete tax laws” in the passage. Option (a)
of law, duty, or obligation. Thus, violation is the correct can be ruled out because treaty shopping is a loop
word. hole that investors have discovered, however, this
doesn’t mean that the practice is illegal.

VA / Exercise - 18 CEX-5338/P1BS/17 / Page 3


12. c Refer to the second paragraph that states that the 22. d Refer to the last paragraph which discusses the flip
Supreme Court ruled against the taxman (and in fa- side – in this case the negative as the first paragraph
vour of Vodafone) and accepted that Indian law does introduces a positive view point. Option (a) can be
not have provisions to stop the widely used tax eva- inferred from the seventh line of the paragraph, “Be-
sion methods used by incoming foreign investors. State- sides, the government...reining it in.” Option (b) can be
ment 1 is correct. Statement 2 is also correct and can inferred from the lines, “For instance, recent...boosting
be inferred from the sixth paragraph. Refer to the line, stock prices.” Option (c) can be inferred from the
“…the company to claim tax from is Hutchison, which lines, “The cheap money...received greater attention.”
made profit from the sale of its stake in the telecom Option (d) is incorrect as the author subtly criticises
company.” Option (c) is the answer. following a cheap money policy. However, there is no
mention of money obtained through cheap means or
13. b Option (a) can be ruled out by a reference to the through low means.
second paragraph in which it is stated that Indian law
was incapable of plugging a widely used tax dodge 23. b Refer to the first paragraph. Statement 1 is incorrect
by inbound foreign investment. However, this does because the line, “...that the economy will grow at a
not point to the general archaic nature of Indian law. reasonably fast clip of around 6.5 per cent...” implies
Option (b) is the answer and can be inferred from the that the rate of growth of the economy will occur at
last line of the fourth paragraph - “…taxes in retro- around 6.5 per cent and not much more than 6.5 per
spect are best avoided.” The passage also talks about cent. Statement 2 is correct and can be inferred from
“the fog over retrospective taxes” which tells us that the line “...the actions of the European Central Bank
there is a lack of clarity about the issue. Option (c) is and the Federal Reserve will help in bringing eco-
incorrect as the line, “Specifically, they must…tax rev- nomic growth in Europe and the U.S. back on track.”
enue” implies that taxes in retrospect should not be Statement 3 is incorrect as the ninth line states that oil
used to just raise tax revenue. This does not imply that prices will not go much higher than the current price
taxes in retrospect is not the only way to raise tax of $100 per barrel. However, there isn’t enough infor-
revenue. Option (d) can be ruled out because it goes mation in the passage to infer that the prices will fi-
beyond the scope of the passage and the focus of nally settle at a rate higher than the current $100 per
the author’s argument. barrel. Option (d) is the answer.

14. d Both the statements are incorrect. Refer to the first 24. c Both the statements are correct. Refer to the first and
and third paragraphs. second paragraphs.

15. d Both the statements are incorrect. Refer to the last 25. d Both the statements are incorrect. Refer to the first
sentence of the third paragraph and the fourth and second paragraphs for the answer.
paragraph.
26. b Confirm is the right answer.
16. b Nub is a synonym for crux.
27. c Surfeit means Excess.
17. c Unambiguous is a synonym for unequivocal.
28. d ‘Traction’ means grip.
18. b The correct answer is prospective.
29. b Acceleration is the opposite of deceleration.
19. c Defunct is the right answer.
30. c In this context steady is the opposite of faltering.
20. d
31. c The sentences “The stock market is in the...money
21. b Refer to the first paragraph. Option (a) cannot be back, with interest.” in the first paragraph clearly tell
inferred as there is no reference in the passage to the us that with the low value of their shares these com-
cabinet reshuffle. Option (b) can be inferred from the panies are now faced with paying back investors the
third line of the first paragraph – the level of deficit is
money they lent them. Option (a) cannot be inferred
still way above what is considered prudent and man-
as the passage does not talk about the number of
ageable. Option (c) is incorrect because there is no
correct or incorrect level of current account deficit years required to reach the preferred status. Option
reduction discussed in the passage. Option (d) moves (b) is a judgement on a past decision taken by the
the discussion to a different track – it suggests that companies - ‘however, the basis for the decision is
the balance of payments situation will not reach a not known and hence one cannot pass judgement on
critical stage if there is a reduction in imports and the decision itself.’ Option (d) is incorrect because the
exports. However, this option does not address the impact of the low share values on market expecta-
question itself. Option (b) is the answer. tions cannot be inferred from the passage.

CEX-5338/P1BS/17 / Page 4 VA / Exercise - 18


32. a Refer to the second paragraph that discusses the 42. c Refer to the sentence “While the protagonists
measures that the government can take in order to squabble...vegetables, milk and fruit” given in the sec-
revive investor sentiment. Option (a) cannot be in- ond paragraph. The author indicates that those con-
ferred from the paragraph as the measures that have nected with the right-to-food campaign spend their
been mentioned are concerned with bailing out com- energies on fighting over other issues instead of fo-
panies that need to pay up to their creditors and en- cusing on more critical issues like child malnutrition.
suring wise management of the economy. There is no Statement (3) is incorrect because it is not a criticism
mention of linking producers directly with consumers. but only a further explanation of the problem of child
malnutrition. Statement (2) is not a criticism of the cam-
Option (b) can be inferred from the sentence “It is
paign or of those connected with it. It is a factor con-
thinking of...pay off their creditors”. Option (c) can be
tributing to the point of criticism mentioned in state-
inferred from the sentences “Reviving investor
ment 1. Hence, option (a) is the correct answer.
sentiment...and macroeconomic stability.” Option (d)
can be inferred from the sentence “The markets have 43. d Refer to the last paragraph. Statement (1) cannot be
revived...prudent economic management.” inferred because the paragraph does not mention dis-
tribution as a weak link in the food security issue. It is
33. a Refer to the last paragraph. Statement (1) can be in- the lack of perspective that food security is not just
ferred from the sentence “Companies are starved ... about caloric intake but requires a more concerted
its social welfare agenda”. It implies that because the effort. Statement (2) is incorrect because it places the
government has availed of most of the household sav- blame of unemployment on food security. Whereas
ings, there are not enough funds within the country the paragraph states that the National Food Security
for the corporate sector. As a result, this sector has to
Bill, even with amendments, encourages an economic
borrow funds from abroad. Statement (2) cannot be
model that gives rise to unemployment and food crisis
inferred as the last sentence of the passage indicates
among the rural poor. This does not indicate that all
that the policy decisions of the government have been
work on food security leads to problems of unemploy-
detrimental to the interests of the corporate sector
ment. The answer is option (d).
and have adversely affected India’s previous growth
status as the second fastest growing major economy.
Hence, option (a) is the correct answer. 44. d Refer to the first sentence of the passage for the
answer.
34. a Refer to the third paragraph for the answer.
45. d The entire topic talks about food security. Hence, option
(d) is the answer. The other options are narrow in
35. b
scope.

36. b In a pickle means in a mess. 46. c In the given context paradigm means pattern.

37. c To tank is to crash. 47. d Variegated & varied both mean of many types.

38. b ‘Revive’ means Resuscitate. 48. c ‘Squabble’ means ‘to bicker’.

39. b Option (b) is the right answer. 49. b Reassuring is the opposite of disconcerting.

40. c The opposite of expansion is contraction. 50. c The opposite of protagonist is antagonist.

41. a Refer to the first paragraph. It criticises the debate on 51. a The answer should be option (a) The author provides
food security because this debate is more concerned information in the passage that supports the need for
with capitalist (commercial) aspects. The next sen- employment related skills formation and vocational
tence goes on to discuss that the debate does not training of the Indian workforce. Option (b) has been
focus on the fact that food also has social aspects mentioned but it is not the central idea as the author
moves the discussion towards a solution - reforms by
(something we eat to varying needs and tastes...and
the government. Option (c) is incorrect. The author
value in specific terms for preserving life, of self and
discusses weaknesses in the system (based more
others). Option (a) is the answer. Option (b) can be
on implementation) and not weaknesses in vocational
ruled out because the price of commodities is a very
training. Option (d) cannot be inferred - the author
limited capitalist/commercial aspect. Option (c) can be only says that "Training for workers in the unorganized
ruled out because the passage does not link livelihood sector has so far mainly been imparted by NGOs" but
to the debate on food security. Option (d) is incorrect this cannot be inferred to mean that NGOs must not
because food production has not been mentioned in impart this training.
the passage.

VA / Exercise - 18 CEX-5338/P1BS/17 / Page 5


52. d The first paragraph of the passage has a reference to about domination of the organized sector over the
the two crucial aspects - given in option (d). Option unorganized sector. This does not correctly explain
(a) has been incorrectly written. Option (b) is incorrect the reason that the author has given in the passage.
as the author mentions the access to quality education Option (d) has not been mentioned by the author as a
and vocational training as the crucial aspects. Option reason and hence it cannot be inferred.
(c) has been given by the author as a substantiation
for his argument that the government should be 54. d Both the statements are incorrect. Refer to the second
involved in making India's workforce more competitive paragraph for the answer.
in the international labour market.
55. b The author of the passage is an economist because
53. b In the last line of the second paragraph, the author the passage deals with Economics.
discusses the reason for training being confined only
to the organized sector. It is clear that the answer is 56. d 57. a 58. c
Option (b). Option (a) cannot be inferred and we do
not know that the two events were a cause and effect 59. d 60. b
phenomenon. Option (c) is incorrect because it speaks

CEX-5338/P1BS/17 / Page 6 VA / Exercise - 18


Verbal Ability – 19 P-1(BS)

Reading Comprehension – Revision


Answers and Explanations

1 d 2 c 3 b 4 c 5 b 6 d 7 a 8 d 9 c 10 d
11 a 12 b 13 c 14 b 15 a 16 c 17 a 18 c 19 c 20 b
21 a 22 d 23 b 24 b 25 d 26 a 27 c 28 c 29 d 30 c
31 a 32 c 33 b 34 a 35 b 36 c 37 b 38 d 39 a 40 d
41 d 42 c 43 a 44 d 45 c 46 c 47 d 48 b 49 c 50 b
51 d 52 b 53 b 54 c 55 a 56 d 57 b 58 d 59 b 60 a
Practice Questions
1 d 2 c 3 d 4 c 5 b 6 a 7 b 8 c 9 d 10 c
11 b 12 a 13 c 14 a 15 d 16 c 17 b 18 a 19 c 20 d
21 b 22 a 23 a 24 c 25 a 26 d 27 d 28 a 29 b 30 a
31 c 32 b 33 d 34 a 35 a 36 d 37 a 38 a 39 d 40 c
41 b 42 c 43 a 44 c 45 a 46 b 47 a 48 d 49 c 50 b
51 c 52 d 53 b 54 c 55 a 56 d 57 a 58 c 59 c 60 d
61 c 62 b 63 c 64 d 65 d 66 d 67 c 68 b 69 a 70 d

1. d The passage states that “growth of the press depends “essential for effective newspaper management”. The
on both” – “subscribers who can afford to buy first paragraph mentions that when it comes to
newspapers” as well as “businessmen who can newspaper management, aspects like “subscribers
afford to advertise in it”. Hence, option (d) is the correct who can afford to buy newspapers” and
answer. All other answer choices are limited. “businessmen who can afford to advertise in it” cannot
be neglected. It is because both aid the inflow of money
2. c ‘Newspaper Management’ is the most apt choice as to the newspapers. This proves that (a) and (b) are
the given passage talks about the factors that affect required for good newspaper management. Option
and facilitate newspaper management. Hence, option (c) is the answer because the passage only states
(c) is the correct answer. Other options are inept that the department heads “must be aware of the
because ‘advertisements’ and ‘economics’ are
goals set, policies and future plans of the
discussed as potent aspects capable of influencing
management”, while they make no reference to setting
the management of newspapers.
up of goals.
3. b ‘Vital’ refers to something/someone necessary or
important for something/someone to succeed or exist. 5. b The word 'afford' in the passage is used to denote
The last sentence of the passage emphasizes upon a that the heads of various departments cannot bear to
vital/important aspect of newspaper management. remain cut off from the mainstream of management
Hence, option (b) is the correct answer. Options (a) function. The other options are incorrect.
and (d) are incorrect because both ‘primary’ and
‘paramount’ are superlative adjectives meaning ‘most 6. d The entire passage focuses on the evolution of the
important’ or ‘more important than anything else’ but modern horse from the size of rabbits to the height
the adjective ‘vital’ is positive in degree. ‘Acute’ means that they have reached today. Therefore, option (d) is
‘serious’.
correct. Option (a) is incorrect because the study of
fossils only aid the research referred to. Option (b) is
4. c Option (d) is incorrect because the second paragraph
incorrect because it does not talk about horses. Option
of the passage states that “coordination” and
(c) is incorrect because the passage does not talk
communication among various departments is
about different species but evolution of horses.

VA / Exercise - 19 CEX-5339/P1BS/17 / Page 1


7. a Since the word ‘species’ in the fourth paragraph refers 16. c It is stated in the passage that “These sentiments belong
to different types of horses, option (a) would be the to one or other of two contrasting and complementary
correct answer. groups … Sentiments of Human Insignificance, and
Sentiments of Human Greatness”. Hence option (c) is
8. d Refer to the fourth paragraph where it is clearly stated, correct.
“They looked very much like a tiny modern horse”.
This clearly indicates that the horses at the end of the 17. a Refer to the passage “… the nearest and most familiar
Dawn period had a body structure similar to that of the symbol of all the astronomical horrors: but the
modern horses. Hence, option (d) is correct. astronomers who discovered those horrors of space
and time were men. The universe throws down a
9. c The first paragraph clearly state that the Dawn horse challenge to the human spirit; in spite of his
was called so “because it lived at the beginning of the insignificance and abjection, man has taken it up.” The
age of the mammals”. Hence, option (c) is correct. tone is far from abject since the author takes pride in
what has been accomplished till now by humanity.
10. d Refer to the last paragraph, “The history of the horse Hence, option (a) is the correct answer.
shows how evolution helps an animal better adapt to
its world”. Hence, option (d) is correct. All other options 18. c The contrasting and complementary sentiments are
are therefore incorrect. given as the “Sentiments of Human Insignificance”
and the “Sentiments of Human Greatness”. Thus the
11. a The very first line of the paragraph states, “The correct answer is option (c).
suggestion of the National Commission for Minority
Educational Institutions (NCMEI) to make mathematics 19. c Though the author starts out by saying that looking at
and English optional so that they do not count for the moon, man might feel insignificant or great, in the
promotion in secondary school would be music to second paragraph he mainly talks about how in spite
many an ear.” Therefore, option (a) is correct. Option of his insignificance and abjection, man has taken up
(b) can be negated by the statement, “The the challenge of the black boundlessness of space.
recommendation is not baseless.” Options (c) and (d) Hence option (c) is correct.
are incorrect because they cannot be evidenced from
the passage. 20. b In the second paragraph, the author states that man
has stood up against the challenge of black boundless
12. b Refer to the following line from the first paragraph, space and we have a right to our moods of sober
"The figures challenge the stereotype that low scores exultation. Thus, the author is more likely to agree with
are obtained by those born with low IQ." From this we what Paul Valery says.
can infer that economically weaker section score low
not because they have low IQ, but because they are 21. a The passage talks about all round reforms making
economically weak. Therefore, option (b) is correct. ‘a’ the correct option.
Options (a) and (c) are not stated in the passage.
22. d The passage talks about simplifying and reducing the
13. c Option (c) is correct because it is clearly stated in the tariff making option ‘d’ correct.
last paragraph of the passage. All other options are
incorrect for lack of evidence from the passage. 23. b The passage talks about high standards of protection
making option ‘b’ correct.
14. b Option (b) is correct because the paragraph of the
passage states, "Locality could play a major role in 24. b The entire scope has reduced considerably making
low examination scores." Option (c) is not stated in option ‘b’ the correct option.
the passage.
25. d Option (d) is correct because the author analyses the
15. a The author presents the data pertaining to education entire topic of trade and economic growth. He provides
in an objective manner. Hence, option (a) is correct. the pros and cons such as simplification of tariff and
Although in the fifth paragraph, the author sounds a anti-export bias. The author is certainly not appreciative
little ironical, his manner of presenting the facts is of the entire situation. Neither is he criticizing because
generally objective. Hence, option (c) can be ruled he provides some of the upsides as well. Hence,
out. He is certainly not appreciative. His tone is also options (a) and (b) can be ruled out. The author is
not too critical. never satirical. Option (c) is out of context.

CEX-5339/P1BS/17 / Page 2 VA / Exercise - 19


26. a The word ‘absorbed’ in the context of the passage with some quality which makes them incorrect. (a)
means engrossed in thoughts. vests it with free play, (b) as representing beauty, (d)
inner beauty.
27. c Apart from profession, careers can also mean ‘to move
swiftly and in an uncontrolled way’. The word is used 43. a The passage marks the advent and spread of
as a verb. Aesthetics and presents Kant’s views on pure
enjoyment of nature and why he thought arts to impure.
28. c As per the second paragraph, Paul’s journey on The passage traces Kant argument for appreciation
horseback reflected passion and perseverence. of pure beauty, this being the main line of thought and
thus makes (a) correct. (b) is incorrect as there is no
29. d The entire passage talks about the imagination and occurrence of any sorts, so is C, as there is no
passion of Paul. Hence, option (d) is the answer. conjecture. (d) is on the same lines and are incorrect
as the passage concentrates on Kant’s line of thought
30. c ‘Inwardly’ in the context of the sentence refers to the only.
inner self of Paul.
44. d The author employs reasoning to define for us Kant’s
31. a Refer to para one of the passage where the difference line of thought. (a) is incorrect as the tone is quite
between sadness and depression has been cited. objective, (b) is incorrect as there is no maligning and
The other options are incorrect. (c) is incorrect as the passage seeks to merely inform.
Option (d) the correct answer choice.
32. c Refer to the first two lines of para two for the answer.
45. c The passage is quite clearly about Kant’s theory of
33. b Refer to the lines - ‘In all likelihood, depression has pure beauty; the passage gradually veers towards
less to do with events that occur than with an this. All other options are incorrect as in (a) we talk
individual’s inherent vulnerability to the condition.’ about search of beauty which is not correct, (b) is
incorrect because it is more of pure beauty, (d) is
34. a The author has described the symptoms and the again incorrect because its an aspect being discussed.
causes of depression in this passage. His tone is
similar to that of a psychologist or psychotherapist. 46. c an advisory decree. The passage clearly mentions
the meaning of the phrase. (a) is not mentioned, (b) is
35. b Rest of the three options are mentioned in the passage. vague and (d) is not suitable.

36. c 2nd paragraph, inertia principle explains it. 47. d The passage clearly states that the senate had no
role in policy making, all the other options define the
37. b Moons of Jupiter were discovered not of Saturn. Refer
role of the senate.
to the second paragraph.

38. d The entire passage talks about Galileo’s observations 48. b The passage is entirely devoted to explaining the
and their impact on theory. Hence, option (d) is correct. powers vested with the senate making (b) correct.

39. a Refer to the third paragraph where the author says 49. c Option (a) is incorrect because it is rather lofty, option
that Newton developed a system of mechanics which (b) is very abrupt, option (d) is incorrect. Option (c) is
was based on Galileo’s work. the best title as it captures the essence.
50. b The last paragraph talks about the powers with the
40. d Dogma means forcing a belief on someone. ‘Request’ other authorities thus (b) forms the most logical
has nothing to do with Dogma. continuation. All the others are mentioned in the
passage and thus incorrect.
41. d The passage traces the definition of pure art as per
Kant’s theory. (a) is not correct because it Kant actually
51. d Refer to the first three lines of the passage which
believed in it, (b) is also attributed to Kant in the
passage. (c) is a part of his theory. (d) is the correct states “It still remains to speak of one of the principal
answer because according to Kant it involved free causes, which make diversity of opinion
play and was thus pure. advantageous, and will continue to do so until mankind
shall have entered a stage of intellectual advancement,
42. c According to the passage, Kant equates appreciation which at present seems at an incalculable distance.”
of beauty with scattered pebbles, unfettered and at It explains that diversity of opinion is inversely
the same time harmonious, making (c) the correct proportional to intellectual advancement.
answer. All the other options are vesting the pebbles

VA / Exercise - 19 CEX-5339/P1BS/17 / Page 3


52. b Refer to the line 7 of the passage which states centrality of the state in the administration of criminal
“….when the conflicting doctrines, instead of being justice.
one true and the other false, share the truth between
them: and the nonconforming opinion is needed to 59. b The excerpt, it appears, is a part of a biography. Option
supply the reminder of the truth, of which the received (b) is correct. Option (a) is obviously incorrect since
the author expresses his admiration for Kannabiran at
doctrine embodies only a part.” It explains that
length in the passage, which would be quite
conflicting doctrines including a non-conformist view
inappropriate if included in a legal report. Option (c) is
complement each other. incorrect since nowhere in the passage is Kannabiran
referred to as Late Kannabiran. A paean, is a song of
53. b Refer to line 10 of the passage which states “Popular praise and hence, (d) is incorrect.
opinions, on subjects not palpable to sense, are often
true, but seldom or never the whole truth.” Hence, 60. a The word ‘taboo’ refers to any prohibition resulting
option (b) is correct according to the passage. from social or other conventions. “There were no taboo
issues for Kannabiran” by this, the author means to
54. c Refer to line 19 of the passage which states “Even say that Kannabiran did not have any reservation in
progress, which other to super add for the most part deliberating any kind of issue. Hence, option (a) is
only substitutes. One partial and incomplete truth for correct.
another, incomplete truth for another, improvement
consisting chiefly in this, that the new fragment of Practice Questions
truth is more wanted, more adapted to the needs of
the time, than that which it displaces.” It explains that 1. d Statement (1) cannot be inferred because the
the progress of humanity is reflected in the substitution availability of employment opportunities has not been
of one partial truth for another less adapted to the discussed in the passage. Statement (2) also cannot
needs of time. be inferred because the ever-changing nature of the
Indian middle class has not been mentioned in the
55. a The passage clearly talks about the views of people passage. Hence the answer is option (d).
taking into considerations the extent of truth and falsity
2. c Option (a) can be deduced from the sentence “At a
in different conditions. Vox populi means the voice of
conference...not-quite middle class” given in the third
the people; popular opinion. Hence, it is an appropriate
paragraph. Option (b) can be inferred from the fourth
title for the given extract.
paragraph in which it is clearly stated that $4 is the
national poverty line in many developing countries and
56. d Option (a) can be inferred from the lines “Stressing $10 represents a level of material security. Option (c)
that there is no permanence about any definition of a is incorrect because ‘ad-hoc’ is used to describe the
crime and criminal law, he pointed to the absolute $2-$4 and $6-$10 ranges in the second paragraph.
power accorded to the state to implement the law and The $4-$10 range is discussed in the last two
the real context of abuse, misuse and vindictive paragraphs of the passage. Option (d) can be inferred
administration of the laws”. Option (b) can be inferred from the sentence “At a conference...not-quite middle
from the lines “Kannabiran’s contribution to human class” given in the third paragraph.
rights was not only in the field of criminal law”. Option
(c) can be inferred from the lines “While it is in the 3. d Refer to the last two paragraphs. Option (a) cannot
course of crime detection, investigation and be inferred even though the last paragraph makes a
apprehension of the accused that human rights reference to the $4- $10 range as being still among
violations ordinarily take place”. Thus, the correct the top 30% of Indians. In the third paragraph the $4-
answer is option (d). $10 range has been described as non-poor and not-
quite middle class. Hence, it is clear that the author
57. b The author is formally praising Kannabiran in the makes no attempt to link this range with the middle
passage, citing him as a “role model – of a person class. It is a range that has been used to understand
who through personal example became the the Indian middle-class but not to define them. Option
embodiment of the ethical and political values he (b) can be ruled out because the $4-$10 range has
advocated.” Thus, the author’s attitude towards been referred to as not unreasonable by Birdsall.
Kannabiran is eulogistic. However. the passage reveals no information on how
experts view this range. Option (c) can be ruled out
58.d The formulation refers to the Kannabiran’s formulation as the $10 limit was chosen because it represents a
mentioned in the first sentence of the passage. Thus level of material security. It does not indicate that the
“The simplicity of the formulation masked the profundity middle class, in any country, are always in danger of
of its understanding of criminal law” pertains to the falling back into poverty. Option (d) is thus the answer.

CEX-5339/P1BS/17 / Page 4 VA / Exercise - 19


4. c Refer to the second sentence of the passage for the 22. a The answer is option (a). The author mentions that the
answer. ASEAN reestablished harmonious relations and helped
close the hostility of the Cold War. Option (b) cannot
5. b Refer to the last sentence of the fourth paragraph for be inferred even though the third paragraph mentions
the answer. "festering insurgencies" this can't be juxtaposed with
the lines in the previous paragraph. Option (c) is never
6. a 7. b 8. c indicated - the author has indicated a positive
connotation when he mentions that ASEAN was in a
9. d 10. c "constructive engagement" with the rogue regime,
Myanmar. Option (d) is incorrect because the financial
11. b The answer follows directly from the 2nd paragraph, crisis took place in 1997-98 and the author says that
8th line-"The difference in perception…..mainly due to the role of ASEAN was at its optimum by early 2000s.
varying degree of access". Statement 3 is the reason
for statement 2 and one leads to the other. So (b) is 23. a The Answer follows directly from 3rd paragraph, 4th
correct. Statement 1 is not correct as it is mentioned in line-"But here again……..political-economic
the second paragraph, third line, "This perception is at hegemonies".
….. to them". This implies that there is no lack of
literature. 24. c Refer to the first and second paragraphs for the
answer.
12. a Option (a) can be inferred from the last lines of
paragraph 2. Option (b) has been given in paragraph 25. a The author is most likely a political scientist because
with relation to the documented view only. Option (c) he analyses the political developments in Southeast
has not been mentioned in the passage nor can it be Asia in great detail.
inferred from any information given in the passage.
Option (d) is incorrect and has not been stated by the 26. d 27. d 28. a
author. Answer is (a).
29. b 30. a
13. c Statement 1 can be inferred from 2nd paragraph. It is
mentioned in the second paragraph - "Common 31. c The first line of paragraph 3 mentions the three
property resources…….with respect to them". reasons (roots) for election finance reform. Statement
Statement 2 can be inferred from 4th paragraph. It is 1 and 3 have been mentioned in this paragraph.
mentioned in the fourth paragraph that-"It is common However the paragraph also refers to the public
for nomadic….other parts of semi arid India". Both concern for equal opportunity for political participation
statements 1 and 2 are correct. Hence, (c) is the and not for the polity's concern. Statement 4 has been
correct answer. mentioned as an observation of the political parties'
behaviour during election campaigns and not as a
14. a The passage talks about the property rights in India. reason for warranting a discussion on political finance
The other options are narrow in scope. reform. The answer is (c).

15. d Both the statements are incorrect. Statement 1 is 32. b In paragraph 2 the author mentions the difference in
incorrect because the passage says that pastoral the dependence on funding between the left and right
communities have grazing rights after the harvesting parties. Option (a) mentions the dependence on the "
of the monsoon crop. Refer to the fourth paragraph. collective wealth of the unions" whereas the passage
The second statement is wrong. Refer to the third mentions the "collective contribution of workers often
paragraph. channeled through affiliated trade unions". The
difference between the two is subtle with an
16. c 17. b 18. a understanding that in the first case the workers may
not make their individual contributions (as understood
19. c 20. d from the passage). Option (b) has been mentioned in
the passage and it is the answer. Option (c) is incorrect
21. b Option (a) is not within the scope of the passage. as it is not mentioned that the left parties depend on
Option (b) is the most likely answer given the contributions from the common man or that the right
information in the first and last paragraphs of the parties depend specifically on contributions from large
passage. Option (c) is also beyond the scope of the MNCs. Option (d) is incorrect because networks of
passage. Option (d) has been mentioned in the last influential individuals has not been mentioned in the
paragraph only and since it is has been stated more passage and the dependence on large business is
as a lesson it is highly unlikely the author would have also ignored in this option. The second half of the
gone on to elaborate on it. option indicates that the parties of the left depend on
contribution of the trade unions and not on individual
contribution from the workers.

VA / Exercise - 19 CEX-5339/P1BS/17 / Page 5


33. d Option (a) has not been discussed by the author and but the author has indicated that there is a way around
is beyond the scope of the passage. Option (b) and it. It also cannot be used to paraphrase the line just
(c) have both been mentioned in paragraph 2. Option after it.
(d) is incorrect because it draws away from the
central idea of the passage. The answer is (d). 43. a The correct answer is option (a) as can be inferred
from the first three paragraphs - the second paragraph
34. a In the last few lines of the first paragraph the author refers to experimental errors and the third paragraph
mentions his primary purpose - the need for political refers to self-correction. However, self-discovery is
finance reform. He does not discuss the methodology not part of the discussion put forth by the author and
to ensure political finance reform but discusses the as such is out of the scope of the passage itself. The
characteristics of the thrust of political finance reform. removal of experimental errors has not been discussed
Therefore option (b) is ruled out. The author does go and it is questionable whether this would be the
on to discuss the impact that political finance reform objective of replication. Option (b) can be ruled out.
may bring - stability of democracy and a possible The author also does not say that replication helps
reduction in the widespread cynicism about parties adjust the experimental errors and so this also cannot
and politicians. However he does not aim to discuss be inferred from the passage.
the ways to bring about this stability. Neither does he
discuss the need to reduce the widespread cynicism 44. c Both the statements are correct. Refer to the first and
about parties and politicians. The answer is option last paragraphs for the answers.
(a).
45. a Considering the tone and language, the passage is
35. a In the 4th paragraph of the passage, statements 1 and certainly an excerpt from a journal article.
2 are mentioned as the main characteristics of political
financial reform. Statement 3 is an expected outcome 46. b 47. a 48. d
of implementing a main characteristic of political financial
reform. Statement 4 presents goal of placing 49. c 50. b
expenditure limits but not the main characteristic itself.
Hence, option (a) is the correct answer. 51. c The author presents information on the book review
with factual information. He discusses De Soto's views
36. d 37. a 38. a and does not add his opinions on how the latter has
approached the topic. Option (a) and 4 are ruled out
39. d 40. c as they are negative. Option (b) is also ruled out
because an analytical style would require the author
41. b The author views both sides of the issue, however to present more information and understanding of De
with an objective to discuss his points of view with Soto's style and approach in writing the book.
the reader. There is no strong argumentation presented However, in two cases - the first line included - the
by the author and he maintains a conversational tone author supports De Soto. He then follows it up with an
throughout making the answer as option (b). The author objective view of De Soto's book. And it can then be
writes in a style that is easy to read, however, this seen that the rest of the information in the passage
cannot be termed as casual - it can also be inferred does not take away from the support that the author
that the subject matter is important to him. Option (a) has for De Soto's book. The answer is option (c).
and (c) are thus ruled out. The author also presents
his opinions on the topic and so one cannot say that 52. d The author is reviewing De Soto's book and in that
he maintains an objective tone. logical flow he presents evidence collected by De
Soto on capital accumulation and then moves on to De
42. c The correct answer is option (c) - this is given in the Soto's views on why assets in developing countries
first line of the second paragraph and the rest of the are not profitable. Option (a) is ruled out because the
paragraph as well the third paragraph explain the author does not present his own views. Option (c) is
author's point of view. Option (a) is incorrect because negative and the author has not presented a negative
the author says that time is not a factor in the inability side to De Soto's book. Option (b) is cannot be inferred.
in or avoidance of replication. Option (b) is not a correct Option (d) is the right answer.
inference from the passage. The author says that
there is a higher probability that work that is mundane
will not be replicated and one should not be alarmed at
this. Option (d) has been mentioned in the passage

CEX-5339/P1BS/17 / Page 6 VA / Exercise - 19


53. b The author mentions that De Soto conducts extensive 62. b Refer to the second paragraph. It clearly states that
fieldwork on five developing countries. He then goes economic policy is crucial because it sets the
on to present excerpts of information from the book parameters within which market forces operate.
on three of the countries mentioned. The next Statement (2) is correct. However, statement (1)
paragraph moves on to presenting De Soto's view on cannot be inferred because the paragraph never
why assets in developing countries are not profitable. implies that economic policy is more important than
The passage is a book review and as such the author's market forces (in fact the opposite is implied). Also
aim would be to present a quick analysis of De Soto's there is no implication that it is only through economic
book. It is then logical that the author would go on to policy that market forces can operate – the paragraph
talk about De Soto's analysis and not present evidence instead states that economic policy provides the
of capital accumulated by Peru and Philippines (though structure around which market forces can operate.
the book may do so) . In the first step of the analysis
(and from the last four lines of the passage) one can 63. c Refer to the last line of the second paragraph of the
logically infer that De Soto would have first presented passage. It states that if the integration strategy
evidence and then moved on to analyse the why of changes, this would have an impact on the nature of
the situation. Thus, Option (b) would fit in better than economic linkage. Statement (2) is correct. Statement
option (c). Option (a) is incorrect as it is clearly (1) is incorrect because an evaluation of an integration
mentioned that De Soto in particular conducted strategy would not affect economic linkages – the
extensive fieldwork on five developing countries and second line of this paragraph also helps cement this
not any others. understanding. It is not an evaluation per se but the
future market trends themselves that could impact
54. c Both the statements are correct. Refer to the second economic linkages. Similarly, the third last line of the
paragraph for the answer. second paragraph indicates that the closeness of fit
of the international economic strategies of East Asian
55. a The first statement is correct. Refer to the second last countries determines the trade complementarity –
sentence of the first paragraph. The second statement indicating the nature of economic linkages. Statement
is wrong. (3) is also correct. Option (c) is the answer.

56. d 57. a 58. c 64. d The last paragraph states the basis on which modes
of globalization can be distinguished from each other
59. c 60. d – on the basis of the relative emphasis it accords to
the dimensions of international linkage. Option (d) is
61. c The author starts by mentioning that there is an interplay the answer. Option (c) can be ruled out because the
of two main determinants of economic relations – distinction is based on the emphasis placed on the
market forces and economic policy. However, the rest elements of international linkage and not the elements
of the paragraph goes on to discuss global market themselves. Option (b) is not a logical answer as
forces and the key drivers that are moving these market modes of globalization and integration strategies are
forces closer to each other. Option (c) can thus be used synonymously in the paragraph. Option (a) is
inferred. Option (d) can thus be ruled out as it also incorrect as static comparative advantage is an
includes economic policy. Option (a) is incorrect as it a element of international linkage and not the main
key driver for the man point that is discussed. Option distinguishing factor.
(b) can be ruled out because it is too general –
restructuring in the global market place cannot be 65. d The author is analytical in his approach.
equated to “restructuring of global operations being
undertaken by multinational enterprises”. 66. d 67. c 68. b

69. a 70. d

VA / Exercise - 19 CEX-5339/P1BS/17 / Page 7


Verbal Ability – 20 P-1(BS)

Sentence Jumble and Para Jumble


Answers and Explanations

1 b 2 b 3 c 4 c 5 d 6 c 7 d 8 c 9 b 10 d
11 a 12 d 13 c 14 d 15 c 16 b 17 a 18 b 19 a 20 a
21 b 22 b 23 d 24 b 25 a 26 d 27 c 28 d 29 a 30 c
31 b 32 a 33 b 34 d 35 a 36 b 37 c 38 a 39 c 40 d
41 c 42 d 43 a 44 c 45 a 46 d 47 b 48 b 49 d 50 a
Practice Questions
1 c 2 d 3 a 4 b 5 b 6 d 7 b 8 a 9 d 10 c
11 d 12 a 13 b 14 d 15 b 16 b 17 c 18 b 19 a 20 a
21 c 22 a 23 d 24 c 25 b 26 c 27 b 28 c 29 c 30 b
31 a 32 c 33 c 34 d 35 d 36 a 37 b 38 b 39 d 40 a
41 a 42 d 43 c 44 b 45 c 46 d 47 a 48 d 49 a 50 c
51 a 52 c 53 d 54 a 55 b 56 c 57 b 58 c 59 a 60 b

1. b 2. b 3. c 4. c 5. d Dioysius proposed. 'Both' in statement C refers to


Taoism and Dionysius. It also explains the similarity of
6. c 7. d 8. c 9. b 10. d their purpose that is to define God. Statement D ends
the paragraph with a conclusion. 'The 'actuality of
11. a 12. d 13. c 14. d 15. c God' mentioned in C leads to the conclusion mentioned
in statement D.
16. b 17. a 18. b 19. a 20. a
25. a Statement B introduces a relatively small advantage of
21. b Statement C opens the paragraph by saying that life cloning. Statement D then suggests that there are more
after death was an enormous part of the Egyptian life. advantages. The idea of cloning can have a larger
Statement A follows statement C by giving a reason impact. Statement C mentions a problem that will occur
for their faith. Statement B is next as it tells us what in the future. Statement A provides a conclusion for
happened as a result of such faith. Statement D the problem mentioned in C.
expands upon statement B.
26. d Statement B introduces the subject of 'creation of
22. b Statement C opens the paragraph by giving a general National Investment' and how we can create it. BA is
introduction. D introduces 'Tiger Woods' who conforms a mandatory pair. The keywords which link the two
to the profile mentioned in statement C. DA is a statements are 'In other words'. Statement A decodes
mandatory pair. 'He' refers to Tiger Woods and explains what is meant by the author in B. Statement C follows
how he conquered all. B ends the paragraph by with a specific example. CD is a mandatory pair
explaining the snapshot of his victory which is followed because 'this process' links these two statements.
by a suggestion from the writer.
27. c Sentence C begins the paragraph by introducing the
23. d Statement B opens the paragraph with a general subject of India and its colorful personalities. The
statement about professionals involved in mixing 'shifting patterns' and 'kaleidoscopic images' broadens
sound or mixers. Statement A moves onto define the the point that is presented in C. This clearly suggests
situation when one is a novice. AD is a mandatory that CA is a mandatory pair. DB is also a mandatory
pair. They carry the same flow of thought forward. C pair because D introduces an example that exemplifies
explains the most important requirement that is needed the point explained earlier in the paragraph. The
when one wants to mix sound. pronoun antecedent used in B and the specific
information given in this sentence clearly suggests
24. b Statement A introduces Taoism and Dionysius. B that B will follow D.
elaborates the connection between Taoism and what

VA / Exercise - 20 CEX-5340/P1BS/17 / Page 1


28. d A careful look at the paragraph reveals that the main 32. a Statement D introduces the topic.'These' in statement
topic of the paragraph is the use of 'couch' in C refers to the 'pyramid texts' mentioned in statement
psychotherapy. Although statement C may seem to be D. Statement B says that there is evidence that the
the opener of the paragraph but it is not. Undoubtedly pyramid texts are even older. So, statement B follows
CB is a mandatory pair because statementC refers to statement C. Statement E concludes by saying that
Freud, his use of hypnosis and the importance of the these texts are therefore the oldest sacred texts
couch. StatementB, too, pertains to the same strand available. Statement A ends the paragraph by giving
of thought and explains what happened to the couch the name of the person who produced the complete
once Freud abandoned the use of hypnosis. The pair translation of the text.
CB is given in options (c) and (d). StatementC cannot
33. b Statement D introduces the topic. Statement B follows
being the paragraph because the paragraph is about
statement A by saying that the Magyar is most similar
the importance of the 'couch' in psychotherapy and
to the Finnish. Statement A follows statement B. 'Both'
Freud's example has been given because it was during
refers to 'Magyar' and 'Finnish' languages. Statement
the use of hypnosis that Freud used the couch. Also,
E follows by saying that the languages preserve the
the 'state of relaxation' explained in statementB is
root most carefully. Statement C ends the paragraph
connected with the advantage of the 'recumbent
by saying that both Gothic and Icelandic present traces
posture' mentioned in statementA that carries the same
of Finnish influence.
set of idea forward.
29. a Sentence B is the opener of the paragraph because it 34. d Sentence E begins the paragraph as it refers to the
provides the introduction to the subject matter. Neither main subject of foreign direct investment in Kenya.
sentenceA nor sentence D can begin the sequence None of the other options explicitly introduce the
because in A the comparison has no basis because subject. Sentence D follows as it explains why it was
the subject matter is not clear and in sentence D a criminal to think about this. Sentence C elaborates
particular incident has been mentioned which does further on the types of problems that investors faced
not really stand too much ground if placed in the there. Sentence B explains the effect of corrupt officials
beginning of the paragraph. In short, the ideas on investors. The paragraph concludes with sentence
presented in these lines are fragmented unless they A which presents investors realization. Also, the
are combined with sentence B. So, sentence B clearly pronoun 'they' refers to investors.
stands out from the given choices. Also, 'another major
writer from the continent' pertains to Amitav Ghosh. 35. a Sentence B begins the paragraph by introducing the
Then, sentence C reveals the name of the writer and idea of educational interactive games and their
his work. Sentence D explains a predicament that advantages. Sentence D follows as it explains thekinds
lingers on into the minds of the readers. Sentence A of games available and their benefits. Sentence E
presents a parallel between two writers (in English) explains how these affect the entire schooling of a
from the same continent and further explains the style child. Sentence A makes a mandatory pair with E
and flavour of Ghosh's fiction. because it is connected with the child's learning
through games along with his/ her education in school.
30. c According to the given options, sentences A, B or C Sentence C presents the final advantage of
are the proposed openers of the paragraph. Sentence theseprogrammes and ends the paragraph.
A presents the consequences of the credit card culture,
sentence B pertains to a specific impact on developing 36. b Sentence D begins the paragraph by introducing the
countries and sentence C presents the larger picture fact about certain powerful words. Sentence B
by suggesting that credits cards have been in the specifies what is needed when it comes to spoken
news for the wrong reasons earlier and now as well. language. AEC forms a mandatory sequence because
This sentence does not present a specific impact; it A explains what we expect when we communicate
pertains to the overall news. Therefore, sentence C is orally with someone; sentence E suggests that what
the opener. Sentence A explains 'the wrong reasons' we expect to be clear can be grossly misinterpreted
for which credit cards are in the news. BD is a and sentenceC concludes the entire issue by
mandatory pair because sentence B mentions the explaining the final effect.
'numerous cases of default' and sentence D presents
a particular example that has led to the intervention of 37. c Sentence D opens the paragraph by introducing and
RBI. defining 'adware'. Sentence C forms a mandatory pair
31. b According to the given options, either sentence A or with sentenceD. This is because the pronoun 'it' in
sentence C is the opener. Sentence C, clearly, is the sentenceD is used for 'adware'. Also, the sentence
conclusion of the paragraph. Therefore, sentence A elaborates on how adware is useful. The subject of
is the opening sentence. The 'administrators of 'advertising revenue' is elaborated further in sentence
education' given in sentence A can be linked to C that explains other ways by which companies make
'institutions' and 'best practices'. Both statements carry additional revenue. Sentence A refers to programmers
forward similar ideas. Sentence B pertains to the and distributors that gather and sell internet users'
specific goal of the National Debate which focuses on demographic data. Sentence B concludes the
the same issue. Sentence C is clearly the conclusion paragraph by explaining the overall impact of adware
of the paragraph. on the entire industry.

CEX-5340/P1BS/17 / Page 2 VA / Exercise - 20


38. a Sentence C begins the paragraph as it explains the inshallow water. II is an example of the tsunami
privacy issues with adware. Sentence E presents an disasterwhich is preceded by R that also describes
example and elaborates the kind of issues that can anotherexample of the same.
occur with adware. Sentence A makes a mandatory
45. a C obviously follows from statement I because C shows
pair with E. The keyword 'also' presents an additional
how the experience of playing in the fog can be
problem with the licensing agreement. Sentence B
unnerving. CB is a mandatory pair because B gives
elaborates on the example presented above and shows
the contrary view to the one given in C. A carries
how faulty information is collected. Sentence D ends
forward from B, showing how the experience of
the paragraph by suggesting how these security
playing in the fog can be a good one. D shows how
loopholes can be misused by a hacker.
this ‘relaxed and apathetic’ swinging can sometimes
39. c Statement C introduces the paragraph. It says that the be a comic disaster.
city was laid out in the form of a puma. Statement B
46. d Sentence R is the obvious continuation of the first line
describes the city. Statement E introduces
of the paragraph, as the pronoun 'they' in R refers to
'Sacsayhuaman' and Statement A elaborates on it. So,
'kids' mentioned in the first sentence. P furthers the
EA is a mandatory pair. Statement D follows statement
flow of the paragraph and introduces the course of
A. Statemnet A says that SAcsayhuaman is the best
action if one's kid behaves in an inappropriate way. S
monument that man has built and Statement D tells us
follows P. Q aptly pairs with II, which concludes the
what its most impressive section is.
course of action if the kid continues to misbehave.
40. d The paragraph begins with the historical aspect in
statement C and gives the reason in statement 47. b The paragraph aims to draw our attention to a fact
B.Statement A talks about the present condition. that when a relationship does not work, often the
Statement E follows by citing the change.Statement D persons sharing that relationship fight not over big
ends the paragraph by saying that the debate continues issues but over petty differences in their style of living
even today. or thinking. This idea is well reflected in the first
sentence, which is followed by S, elaborating on the
41. c The sheer extravagance of the Titanic's "demise" is sources of such differences. P gives an example to
substantiated through the superlatives mentioned in help explaining the issue discussed in I and S. Q follows
S. P follows S as it gives the other side of the fascination P. Q mentions the effect of such petty issues on love
for Titanic. RQ is a mandatory pair with R describing relationships and people's failure in accepting the fact
how the Titanic is symbolic of epic performances for that irritations are inevitable in all relationships. R6 is a
some people and Q giving an example of one such mandatory pair. R, which states that no one person
performance. It falls in place before II which also can be perfectly similar as another in his or her every
highlights another example of the performance of the quirk, habit, and preference, forms the basis for II
people on-board. Option (C) is the correct answer. which leaves us with the fundamental challenge of
figuring out how to negotiate and live with our partner's
42. d I talks about some people confusing nationalism for irritants, in a best way.
nation. R follows I as it states what nationalism exactly
is. S follows R as it talks about how one nation can 48. b Sentence I tells us that the speaker has been
have many differing views that can manifest interviewing several researchers. In sentence S 'their
themselves in the form of political differences. QP is a work', denotes the work of the several researchers
mandatory pair with Q giving a general example of mentioned in I. So, I and S become a mandatory pair.
how people view differing beliefs and P stating another RP is another mandatory pair. R mentions another
example prior to II which is more specific than Q. Option Kennedy Krieger study which is on handwriting and
(D) gives the correct sequence. autism. This makes the speaker remind of a dyslexic
friend- Lara, who is introduced in P. Q elaborates the
43. a I talks about Christianity being a religion which doesn't
discussion by stating how Lara has managed to adjust
have a homeland in this world. Q clearly follows I as it
and cope with all her difficulties caused by the disorder
concludes from here that it must be at odds with
(dyslexia) but not with her handwriting. II closes the
thenation. S comes after Q as it talks about
paragraph by mentioning that now, Lara uses a
Christianityrecognizing a distinction between the world
keyboard to help her cope with the only restrain
of Caesarand that of God. P talks about this distinction,
(handwriting) mentioned in Q.
which ismentioned in S, being ambiguous according
to Jesus.R counters it by mentioning that since the
49. d The first sentence of the paragraph introduces 'The
distinction isrecognized by Christianity, an arena exists
Lake District'. R elaborates I. Q follows R, as it mentions
for thedevelopment of the nation and fits in well before
what made the Lake District popular: its associations
II.
with the early 19th century poets and poetry. QS is a
44. c S follows I as it describes how the tsunami waves mandatory pair, as S names the poets mentioned in Q.
are formed. QP is a mandatory pair with Q talking SP6 forms another mandatory pair. P describes how
about the impact of the waves in deep water and the three poets mentioned in S, were referred to as
Ptalking about the same in shallow water. Q has the "Lake Poets." II concludes stating the connection
tocome before P as Q says the waves are of the 'Lake Poets' to 'Romanticism'.
barelynoticeable in deep water but become dangerous

VA / Exercise - 20 CEX-5340/P1BS/17 / Page 3


50. a The first line of the paragraph talks about fantasies 45. c B is the opening statement as it is a general
which many entrepreneurs assume ideal for their new comment that is explained in D by giving example of
businesses. R follows I, as it negates the unrealistic an uneducated person. A and C carry the idea
expectations mentioned in I, by mentioning the truth of forward. Therefore, option (c).
a successful business which, largely, depends on a
46. d Statement v is the opener starting with the introduction
detailed and thorough planning. Q furthers the
of Gnosticism, ii goes on to trace it further, i extends
discussion by laying importance on writing a business
it further with its definition of gnosis, iv talks about its
plan to help achieve a better financial management
further development and iii links it with Christianity.
realistically. PS is a mandatory pair, as sentences P
and S exhibit a 'Causeand- effect relationship'. P talks 47. a iii is the opener giving an idea about the state of the
about new businesses being gushy and having an pilot, v takes us on to the decision, ii takes it forward,
unclear focus; S mentions the result of this and 6 iv highlights the decision he had to make and i throws
sums up the discussion by suggesting the further light on the decision.
entrepreneurs to have a single focus with a proven
48. d v is the opener with iv forming a mandatory pair with
client base.
it, ii takes it forward, i describes the bigger picture and
Practice Questions iii closes with a description of the truly complete
shopping experience.
1. c 2. d 3. a 4. b 5. b
49. a Statement iv marks the opener as it introduces the
6. d 7. b 8. a 9. d 10. c topic, v goes on to elaborate forming a mandatory pair,
i takes it further, iii talks of his debacle and ii of his
11. d 12. a 13. b 14. d 15. b
resurgence.
16. b 17. c 18. b 19. a 20. a 50. c Option iii introduces the subject, i forms a mandatory
21. c 22. a 23. d 24. c 25. b pair with it, ii describes the subject further, v introduces
her agenda and iv closes it.
26. c 27. b 28. c 29. c 30. b
51. a Sentence C is the opener, B spells out the effect, E
31. a 32. c 33. c 34. d 35. d forms a pair with it and A takes it forward, forming a
36. a The paragraph must start with sentence D as the 'it' conclusion in D.
mentioned in sentence A is the communism 52. c E is the opener A takes it further, D & C expand on the
mentioned in D. Sentence A further elaborates the subject, B is the result.
definition of communism and B should follow option
A. Hence DABC. 53. d E is the opener, D tells us why bats detect plants, B &
C inform us about the complexities, A throws light on
37. b Sentence A defines what fascism is and D extends the developments in the field.
the definition. B and C are also a mandatory pair.
Hence ADBC. 54. a C introduces the topic, B introduces various aspects,
E & A tell us about their structure, D describes their act
38. b The 'he' mentioned in sentence D is Newton of of porpoising.
sentence A. So D should come after A. Sentence B
lays stress on work done by Newton and C 55. b Sentence B is the opening statement forms a mandatory
concludes the paragraph. Hence ADBC. pair with E, A & D elaborate the topic with C commenting
on its current status.
39. d AB and CD are the 2 mandatory pairs. Hence ABCD.
56. c Option iv is the opener, introduces the subject goes on
40. a The 'he' mentioned in the root of other sentences to describe the travails of teens, v forms a pair with it,
refer to Tagore in sentence B. B and D can be limited i takes it forward and iii expands it with ii forming the
together with the keyword 'nationalism'. A and C is a conclusion.
mandatory pair.
57. b ii introduces the subject; v goes on to tell us about his
41. a B cannot be the opening sentence (Thus...... ) nor writings, iii elaborates the ‘two forces’, i describes the
can it be D (But then.......). Between 1 & 4, B change and iv elaborates.
following A is more logical than B following C.
Therefore, option (a). 58. c iii introduces the subject of ‘Fashion cycles’, forming a
pair with ii, ‘yet’ forms the link, v describes it at length,
42. d BA is a mandatory pair. D follows A and BAD is the i reinforces the idea and iv forms the befitting
logical flow. C introduces. Therefore, option (d). conclusion.
43. c A follows B and B is the opening statement.
Similarly, D further elaborates on the statement given 59. a iii introduces the subject, i takes it forward, ii, v & iv
in option (C). Hence CD is also a mandatory pair. elaborate the idea.
Therefore, option (c).
60. b ii introduces the subject of “The notion of time”, i adds
44. b D introduces and therefore is the opening statement. to the subject, v takes it further and iii elaborates on it,
BAC is the logical flow of sequence. Therefore, iv goes on to describe the prodigious time scales.
option (b).

CEX-5340/P1BS/17 / Page 4 VA / Exercise - 20

You might also like